Applied Mathematics for Class 11th & 12th (Concepts and Questions) | ||
---|---|---|
11th | Concepts | Questions |
12th | Concepts | Questions |
Content On This Page | ||
---|---|---|
Objective Type Questions | Short Answer Type Questions | Long Answer Type Questions |
Chapter 8 Index Numbers and Time Based Data (Q & A)
Welcome to this comprehensive Question and Answer practice platform, meticulously developed for Chapter 8: Index Numbers and Time Series Analysis within the Applied Mathematics syllabus. This resource serves as a vital tool for testing, reinforcing, and deepening your understanding of these specialized statistical techniques, which are indispensable in modern economics, business analytics, and financial forecasting. Index numbers allow us to measure relative changes over time, while time series analysis helps us dissect historical data to understand trends and make predictions. This Q&A collection moves beyond theoretical knowledge, challenging you to perform accurate calculations, interpret results meaningfully, and apply these methods to practical scenarios, thereby honing essential quantitative analysis skills.
The questions featured here extensively cover the key concepts related to Index Numbers. You will find problems assessing your ability to calculate various types of indices and understand their significance:
- Simple Index Numbers: Calculating indices using the Simple Aggregative method and the Simple Average of Price Relatives method.
- Weighted Index Numbers: Recognizing the need for weights and calculating key price indices using different weighting schemes:
- Laspeyres' Index: Utilizing base period quantities as weights, $P_{01} = \frac{\sum p_1 q_0}{\sum p_0 q_0} \times 100$.
- Paasche's Index: Employing current period quantities as weights, $P_{01} = \frac{\sum p_1 q_1}{\sum p_0 q_1} \times 100$.
- Fisher's Ideal Index: Calculated as the geometric mean of Laspeyres' and Paasche's indices, $P_{01} = \sqrt{L \times P}$, often favored for satisfying statistical tests like the Time Reversal Test and Factor Reversal Test (these tests might be assessed conceptually).
- Interpretation and Application: Understanding that these indices measure percentage changes relative to a base period, and appreciating the use of important published indices like the Consumer Price Index (CPI) and Wholesale Price Index (WPI) in measuring inflation, adjusting wages (often involving $\textsf{₹}$ values), and informing economic policy.
The second major focus is on Time Series Analysis, the methodology for analyzing data collected sequentially over time. Questions will test your conceptual understanding of the different components that typically constitute a time series (Secular Trend (T), Seasonal Variation (S), Cyclical Variation (C), and Irregular Variation (I)) and the primary goal of identifying these patterns for analysis and forecasting. The main calculative emphasis is on measuring the Secular Trend. You will practice applying common techniques:
- Method of Moving Averages: Calculating trend values by averaging over specific periods (e.g., 3-yearly, 5-yearly), including handling centering for even periods, to smooth out short-term fluctuations.
- Method of Least Squares: Fitting a linear trend line of the form $Y_c = a + bX$ to the time series data. This involves calculating the constants $a$ and $b$ using the normal equations (derived from minimizing $\sum (Y - Y_c)^2$). Questions may involve finding the trend line equation and using it to estimate trend values or make short-term forecasts.
To ensure comprehensive assessment, the questions span various formats: MCQs (testing definitions, interpretations, simple calculations), Fill-in-the-Blanks, True/False statements (probing properties or concepts), and challenging Long Answer questions requiring detailed computation of weighted indices, moving averages, or fitting trend lines via least squares, followed by interpretation. The provided answers are meticulously detailed, showcasing the necessary tabular calculations for index numbers and time series methods, clear step-by-step application of formulas, and contextual interpretations of the calculated values, ensuring you gain both computational proficiency and analytical insight.
Objective Type Questions
Question 1. A time series is a set of observations recorded in:
(A) Any random order
(B) Ascending order of magnitude
(C) Descending order of magnitude
(D) Chronological order
Answer:
Explanation:
A time series is a sequence of data points collected or recorded in successive order over a period of time. The key characteristic of a time series is that the observations are ordered chronologically. This ordering is essential for analyzing trends, seasonality, and other patterns that evolve over time.
Therefore, a set of observations recorded in chronological order is called a time series.
The correct answer is (D) Chronological order.
Question 2. The component of a time series that describes the long-term overall movement is called:
(A) Seasonal Variation
(B) Cyclical Variation
(C) Irregular Variation
(D) Secular Trend
Answer:
Explanation:
A time series can be decomposed into several components that help describe its behavior over time. The major components are:
- Seasonal Variation: Fluctuations that occur at regular intervals, usually within a year (e.g., sales increasing during holidays).
- Cyclical Variation: Long-term oscillations or swings around the trend line, typically lasting longer than a year and not necessarily following a fixed pattern.
- Irregular Variation: Unpredictable fluctuations caused by random or unforeseen events (e.g., strikes, natural disasters).
- Secular Trend: The smooth, general long-term movement of the series over a significant period of time. It reflects the underlying growth or decline of the variable.
The component that specifically describes the long-term overall movement or direction of the time series is the Secular Trend.
The correct answer is (D) Secular Trend.
Question 3. Variations in a time series data that occur regularly within a period of one year or less are known as:
(A) Secular Trend
(B) Cyclical Variation
(C) Seasonal Variation
(D) Irregular Variation
Answer:
Explanation:
Time series analysis involves identifying and understanding different types of variations present in the data:
- Secular Trend: Describes the long-term direction or movement of the data over an extended period, usually several years.
- Cyclical Variation: Refers to fluctuations that occur over periods longer than a year, often associated with business cycles (prosperity, recession, depression, recovery). These are not necessarily regular in timing or amplitude.
- Irregular Variation: These are unpredictable, random fluctuations caused by unforeseen events like natural disasters, strikes, or sudden policy changes.
- Seasonal Variation: These are patterns that repeat over a fixed period of time, usually within a year. Examples include quarterly sales figures peaking in the fourth quarter, or temperature variations across the seasons.
The variations that occur regularly within a period of one year or less are the defining characteristic of Seasonal Variation.
The correct answer is (C) Seasonal Variation.
Question 4. The boom and depression phases in a business cycle are examples of:
(A) Seasonal Variation
(B) Cyclical Variation
(C) Secular Trend
(D) Irregular Variation
Answer:
Explanation:
Let's review the components of a time series:
- Seasonal Variation: Regular fluctuations occurring within a year (e.g., monthly, quarterly).
- Secular Trend: The long-term, smooth movement of the series over many years.
- Irregular Variation: Unpredictable, random fluctuations caused by sudden events.
- Cyclical Variation: Long-term oscillations or swings around the trend line, typically lasting longer than a year and representing phases like boom, recession, depression, and recovery. These are associated with business cycles and are not necessarily regular in their periodicity or amplitude.
The different phases of a business cycle, such as boom, recession, depression, and recovery, are characteristic examples of Cyclical Variation because they represent long-term fluctuations around the general trend and are related to the overall economic activity.
The correct answer is (B) Cyclical Variation.
Question 5. Variations in a time series caused by unpredictable events like earthquakes, floods, or strikes are called:
(A) Seasonal Variation
(B) Cyclical Variation
(C) Secular Trend
(D) Irregular Variation
Answer:
Explanation:
The variations in a time series are categorized based on their nature and duration:
- Seasonal Variation: Regular fluctuations that repeat over a period of one year or less.
- Cyclical Variation: Long-term oscillations around the trend line, lasting longer than a year, often associated with business cycles.
- Secular Trend: The smooth, long-term direction or movement of the series over a significant period.
- Irregular Variation: These are random, unpredictable fluctuations caused by sudden and unforeseen events that do not follow any systematic pattern. Examples include natural disasters, wars, strikes, epidemics, or sudden political changes.
Variations caused by unpredictable events such as earthquakes, floods, or strikes fall under the category of Irregular Variation.
The correct answer is (D) Irregular Variation.
Question 6. Which component of a time series is generally smooth and represents the fundamental direction of the data over a long period?
(A) Seasonal Variation
(B) Cyclical Variation
(C) Secular Trend
(D) Irregular Variation
Answer:
Explanation:
Let's consider the nature of each component:
- Seasonal Variation: Characterized by regular peaks and troughs occurring at fixed intervals within a year. It is not smooth over a long period.
- Cyclical Variation: Involves long-term oscillations around the trend, often associated with business cycles. While long-term, it is not necessarily smooth and represents swings, not the fundamental direction.
- Irregular Variation: Consists of unpredictable, random fluctuations caused by unforeseen events. It is not smooth or indicative of a long-term direction.
- Secular Trend: Represents the underlying, smooth, long-term movement or direction of the time series data over an extended period. It captures the fundamental growth or decline pattern, abstracting from shorter-term fluctuations.
The component that is generally smooth and represents the fundamental direction of the data over a long period is the Secular Trend.
The correct answer is (C) Secular Trend.
Question 7. Which method is suitable for measuring trend when the trend is approximately linear and the data has significant short-term fluctuations?
(A) Method of Least Squares
(B) Moving Average Method
(C) Graphical Method
(D) All of the above
Answer:
Explanation:
Let's examine the suitability of each method for measuring trend when the trend is approximately linear and significant short-term fluctuations are present:
- Method of Least Squares: This method fits a specific mathematical function (like a straight line, $y = a + bx$, for a linear trend) to the data by minimizing the sum of the squared differences between the actual data points and the fitted line. It provides the best fitting line according to the least squares criterion and is suitable for fitting linear trends. However, significant short-term fluctuations can influence the position of the fitted line and the method doesn't inherently 'smooth' the data.
- Moving Average Method: This method involves calculating the average of data points over a specified period and plotting these averages. The purpose is to smooth out short-term fluctuations (seasonal and irregular variations) to reveal the underlying trend and cyclical components. When the data has significant short-term fluctuations, the moving average is highly effective in filtering out this noise, making the underlying trend (which is approximately linear in this case) much clearer.
- Graphical Method: This method involves plotting the time series data and drawing a freehand curve that appears to follow the general direction of the points. While it provides a visual estimate of the trend, it is subjective and less precise, especially when fluctuations are significant, as it can be challenging to accurately draw a line that represents the true underlying trend.
Considering the presence of significant short-term fluctuations, the Moving Average Method is particularly suitable because its primary function is to smooth out such fluctuations, making the underlying approximately linear trend more evident and easier to measure or describe.
The correct answer is (B) Moving Average Method.
Question 8. In the Method of Least Squares for fitting a straight line trend ($Y_c = a + bX$), the equations to solve for $a$ and $b$ are:
(A) $\sum Y = na + b \sum X$ and $\sum XY = a \sum X + b \sum X^2$
(B) $\sum Y = na + b \sum X$ and $\sum XY = a \sum X + b \sum X^2$
(C) $\sum X = na + b \sum Y$ and $\sum XY = a \sum Y + b \sum Y^2$
(D) $\sum Y = a \sum X + nb$ and $\sum XY = a \sum X^2 + b \sum X$
Answer:
Explanation:
The Method of Least Squares is used to find the best-fitting line ($Y_c = a + bX$) that minimizes the sum of the squared differences between the actual values ($Y$) and the predicted values ($Y_c$). Here, $Y_c$ represents the calculated trend value, $X$ represents the time period (often coded), $a$ is the Y-intercept, and $b$ is the slope of the trend line.
To find the values of $a$ and $b$ that minimize $\sum (Y - Y_c)^2$, we differentiate this sum with respect to $a$ and $b$ and set the derivatives equal to zero. This process yields the following two normal equations:
$\sum Y = na + b \sum X$
$\sum XY = a \sum X + b \sum X^2$
where $n$ is the number of data points.
These equations are solved simultaneously to find the values of $a$ and $b$.
Comparing these with the given options, the set of equations that matches the normal equations for fitting a straight line trend using the Method of Least Squares is given in options (A) and (B).
The correct answer is (A) $\sum Y = na + b \sum X$ and $\sum XY = a \sum X + b \sum X^2$.
Question 9. If the time variable $X$ is coded such that $\sum X = 0$, the normal equations for the Method of Least Squares (linear trend) become:
(A) $\sum Y = na$ and $\sum XY = b \sum X^2$
(B) $\sum Y = nb$ and $\sum XY = a \sum X^2$
(C) $\sum Y = na$ and $\sum XY = a \sum X^2$
(D) $\sum Y = nb$ and $\sum XY = b \sum X^2$
Answer:
Explanation:
The normal equations for fitting a straight line trend ($Y_c = a + bX$) using the Method of Least Squares are:
$\sum Y = na + b \sum X$
... (1)
$\sum XY = a \sum X + b \sum X^2$
... (2)
where $n$ is the number of data points, $Y$ are the actual values, and $X$ are the coded time periods.
When the time variable $X$ is coded such that the sum of the coded values is zero, i.e., $\sum X = 0$.
Substitute $\sum X = 0$ into the normal equations:
From equation (1):
$\sum Y = na + b (0)$
$\sum Y = na$
From equation (2):
$\sum XY = a (0) + b \sum X^2$
$\sum XY = b \sum X^2$
Thus, when $\sum X = 0$, the normal equations simplify to:
$\sum Y = na$
$\sum XY = b \sum X^2$
The correct answer is (A) $\sum Y = na$ and $\sum XY = b \sum X^2$.
Question 10. The moving average method is used to:
(A) Isolate the seasonal component
(B) Smooth out short-term fluctuations and reveal the trend
(C) Identify irregular variations
(D) Forecast cyclical variations
Answer:
Explanation:
The Moving Average Method is a technique used in time series analysis to calculate the average of data points over a specified period. This calculation window moves through the data, producing a series of averages.
The primary effect of calculating a moving average is to smooth out the data by averaging away the short-term fluctuations. These short-term fluctuations are typically the seasonal and irregular components of a time series. By removing or reducing the impact of these rapid variations, the underlying longer-term patterns, particularly the Secular Trend and sometimes the Cyclical component, become more apparent.
- Option (A) is incorrect because moving average removes seasonality, it does not isolate it. Isolating seasonality often involves removing the trend first and then analyzing the remaining fluctuations.
- Option (C) is incorrect because moving average smooths out irregular variations; it does not identify them specifically. Irregular variations are often seen as the residuals after removing trend and seasonal components.
- Option (D) is incorrect. While smoothing can help visualize cyclical patterns, the moving average method itself is primarily for trend identification or smoothing, not for forecasting the complex and non-periodic nature of cyclical variations.
Therefore, the main purpose of the moving average method is to smooth out short-term fluctuations and reveal the trend.
The correct answer is (B) Smooth out short-term fluctuations and reveal the trend.
Question 11. A 3-year moving average will eliminate:
(A) Secular Trend
(B) Cyclical Variation
(C) Seasonal Variation of period 3 years
(D) Short-term fluctuations (seasonal and irregular) over a 3-year period
Answer:
Explanation:
The purpose of calculating a moving average is to smooth out fluctuations in a time series. A $k$-period moving average calculates the average of the data over $k$ consecutive periods.
A key property of a moving average of period $k$ is that it will completely eliminate any perfectly periodic component in the time series that has a period of exactly $k$.
- Secular Trend: A moving average follows the trend; it does not eliminate it. It helps to make the trend clearer by smoothing out shorter-term fluctuations.
- Cyclical Variation: Cyclical variations typically have periods longer than a few years and are not always perfectly regular. A 3-year moving average would smooth cyclical variations but would not eliminate them unless they happened to have a perfect 3-year cycle.
- Irregular Variation: Irregular variations are random. A moving average smooths these fluctuations by averaging them out over the period, but it does not eliminate them entirely unless their sum within each moving window is zero.
- Seasonal Variation of period 3 years: If there is a component in the time series that repeats perfectly every 3 years (i.e., has a period of 3 years), a 3-year moving average will exactly eliminate this component because the sum of values over one complete cycle of this variation, when averaged over 3 years, will be constant, effectively removing the variation. While typical seasonality refers to patterns within a year, the question specifically mentions "Seasonal Variation of period 3 years".
Therefore, a 3-year moving average is specifically designed to eliminate a periodic component with a period of 3 years.
The correct answer is (C) Seasonal Variation of period 3 years.
Question 12. Which of the following is a disadvantage of the moving average method?
(A) It is easy to calculate.
(B) It smooths the data.
(C) It results in loss of data at the beginning and end of the time series.
(D) It helps in identifying the trend.
Answer:
Explanation:
Let's evaluate each option:
- (A) It is easy to calculate: This is generally considered an advantage of the moving average method, especially for simple moving averages.
- (B) It smooths the data: This is the primary purpose and advantage of the moving average method. It filters out short-term fluctuations to reveal underlying patterns.
- (C) It results in loss of data at the beginning and end of the time series: To calculate a $k$-period moving average, you need $k$ data points. This means you cannot calculate a moving average for the first $(k-1)$ periods and the last $(k-1)$ periods (or similarly depending on centering). This leads to a shorter smoothed series than the original data, representing a disadvantage as information is lost at the boundaries.
- (D) It helps in identifying the trend: By smoothing out fluctuations, the moving average makes the underlying trend more visible and easier to identify. This is an advantage.
The inability to calculate moving averages for the data points at the start and end of the series is a notable drawback of this method.
The correct answer is (C) It results in loss of data at the beginning and end of the time series.
Question 13. In the multiplicative model of time series, the components are represented as $Y = T \times S \times C \times I$. What does $Y$ represent?
(A) Trend component
(B) Original time series value
(C) Seasonal component
(D) Cyclical component
Answer:
Explanation:
Time series analysis often involves decomposing the observed data into several components. Two common models for this decomposition are the additive model and the multiplicative model.
The multiplicative model is represented as:
$Y = T \times S \times C \times I$
where:
- $Y$ represents the actual or original observed value of the time series at a given point in time.
- $T$ represents the Trend component, reflecting the long-term direction.
- $S$ represents the Seasonal component, representing regular fluctuations within a year.
- $C$ represents the Cyclical component, representing long-term oscillations around the trend.
- $I$ represents the Irregular component, representing random, unpredictable variations.
In this model, the components are multiplied together to get the original value. This is often used when the magnitude of the seasonal and irregular variations is proportional to the level of the trend.
Therefore, in the multiplicative model $Y = T \times S \times C \times I$, $Y$ represents the Original time series value.
The correct answer is (B) Original time series value.
Question 14. Assertion (A): Secular trend always shows an increasing movement over time.
Reason (R): Secular trend represents the long-term direction of the time series, which can be increasing, decreasing, or stagnant.
(A) Both A and R are true and R is the correct explanation of A.
(B) Both A and R are true but R is not the correct explanation of A.
(C) A is true but R is false.
(D) A is false but R is true.
Answer:
Explanation:
Let's analyze the Assertion (A) and the Reason (R).
Assertion (A): Secular trend always shows an increasing movement over time.
The secular trend is the long-term underlying direction or movement of a time series. While many economic or population series show an increasing trend over long periods, the trend is not necessarily always increasing. A time series can exhibit a decreasing trend (e.g., the number of landline phone users over recent decades) or a relatively constant/stagnant trend.
Therefore, Assertion (A) is False.
Reason (R): Secular trend represents the long-term direction of the time series, which can be increasing, decreasing, or stagnant.
This statement accurately defines the nature of the secular trend. It captures the overall long-term movement, irrespective of whether that movement is upward, downward, or horizontal (stagnant).
Therefore, Reason (R) is True.
Since Assertion (A) is false and Reason (R) is true, the correct option is (D).
The correct answer is (D) A is false but R is true.
Question 15. Which method of measuring trend assumes a specific mathematical relationship between the time variable and the time series variable?
(A) Moving Average Method
(B) Method of Least Squares
(C) Graphical Method
(D) All methods are free from such assumptions
Answer:
Explanation:
Let's consider how each method approaches the trend:
- Moving Average Method: This method smooths the data by averaging. While it reveals the underlying trend, it does not assume that the trend follows any specific mathematical equation (like a straight line or a parabola).
- Method of Least Squares: This method involves fitting a specific curve to the data. You must first decide whether you want to fit a linear trend ($Y_c = a + bX$), a quadratic trend ($Y_c = a + bX + cX^2$), an exponential trend ($Y_c = ab^X$ or $\log Y_c = \log a + X \log b$), or some other mathematical function. The method then finds the parameters of that chosen function that best fit the data according to the least squares criterion. Thus, it assumes a specific mathematical relationship between the time variable ($X$) and the time series variable ($Y$).
- Graphical Method: This method involves plotting the data and drawing a freehand line or curve that seems to follow the trend. It is subjective and does not assume a specific mathematical formula for the trend.
The Method of Least Squares explicitly requires the assumption of a predefined mathematical form for the trend line or curve.
The correct answer is (B) Method of Least Squares.
Question 16. A price relative is calculated as:
(A) $\frac{\text{Price in current period}}{\text{Price in base period}} \times 100$
(B) $\frac{\text{Price in base period}}{\text{Price in current period}} \times 100$
(C) $\frac{\text{Price in current period} - \text{Price in base period}}{\text{Price in base period}} \times 100$
(D) $\frac{\text{Price in current period}}{\text{Price in base period}}$
Answer:
Explanation:
A price relative is a measure of the price of a single commodity in a given period relative to its price in a base period. It is essentially a simple index number for one item.
The formula for a price relative, often expressed as a percentage, is:
Price Relative $= \frac{\text{Price in current period}}{\text{Price in base period}} \times 100$
Let $P_1$ be the price in the current period and $P_0$ be the price in the base period. The price relative is given by:
Price Relative $= \frac{P_1}{P_0} \times 100$
Comparing this formula with the given options, option (A) matches the standard definition of a price relative expressed as a percentage.
The correct answer is (A) $\frac{\text{Price in current period}}{\text{Price in base period}} \times 100$.
Question 17. Match the time series component with its characteristic duration:
(i) Secular Trend
(ii) Seasonal Variation
(iii) Cyclical Variation
(iv) Irregular Variation
(a) Long period (many years)
(b) Unpredictable, short duration
(c) Short period (within a year)
(d) Moderate period (several years, but less than trend)
(A) (i)-(a), (ii)-(c), (iii)-(d), (iv)-(b)
(B) (i)-(a), (ii)-(d), (iii)-(c), (iv)-(b)
(C) (i)-(a), (ii)-(c), (iii)-(b), (iv)-(d)
(D) (i)-(b), (ii)-(c), (iii)-(d), (iv)-(a)
Answer:
Explanation:
Let's match each time series component with its typical duration:
- (i) Secular Trend: This represents the long-term, smooth movement or direction of the series. It extends over many years and is considered the longest-duration component. This matches with (a) Long period (many years).
- (ii) Seasonal Variation: These are regular fluctuations that repeat over a fixed period, usually within one year (e.g., monthly, quarterly). This matches with (c) Short period (within a year).
- (iii) Cyclical Variation: These are oscillations or swings around the trend that occur over periods longer than a year, often related to business cycles. They are typically of moderate duration, lasting several years, but are generally shorter than the overall secular trend period and are not necessarily regular in length. This matches with (d) Moderate period (several years, but less than trend).
- (iv) Irregular Variation: These are unpredictable fluctuations caused by random or unforeseen events. They are typically of short and irregular duration. This matches with (b) Unpredictable, short duration.
The correct matching is therefore:
(i) - (a)
(ii) - (c)
(iii) - (d)
(iv) - (b)
Comparing this with the given options, option (A) matches this mapping.
The correct answer is (A) (i)-(a), (ii)-(c), (iii)-(d), (iv)-(b).
Question 18. In the Method of Least Squares, the line of best fit minimizes the sum of the squares of the differences between the actual values and the:
(A) Mean values
(B) Median values
(C) Trend values
(D) Moving average values
Answer:
Explanation:
The Method of Least Squares is a statistical technique used to find the line or curve that best fits a set of data points. When applying this method to time series analysis to find the trend, we fit a function (like a straight line $Y_c = a + bX$) to the observed data points ($Y$).
The principle of least squares is to find the parameters of the fitted function (e.g., $a$ and $b$ for a straight line) such that the sum of the squares of the vertical distances between the observed values ($Y$) and the values calculated from the fitted function ($Y_c$) is minimized.
In the context of trend analysis using the Method of Least Squares, $Y_c$ represents the calculated trend values for the corresponding time periods $X$. The method minimizes $\sum (Y - Y_c)^2$, where $Y$ are the actual values and $Y_c$ are the trend values predicted by the fitted model.
Therefore, the Method of Least Squares minimizes the sum of the squares of the differences between the actual values and the Trend values.
The correct answer is (C) Trend values.
Question 19. Which of the following methods does NOT provide a mathematical equation for the trend line?
(A) Method of Least Squares (Linear)
(B) Method of Least Squares (Parabolic)
(C) Moving Average Method
(D) All provide an equation
Answer:
Explanation:
Let's examine the nature of the output from each method:
- Method of Least Squares (Linear): This method fits a straight line to the data, yielding an equation of the form $Y_c = a + bX$, where $Y_c$ is the calculated trend value, $X$ is the time variable, and $a$ and $b$ are constants determined by the method. This is a specific mathematical equation for the trend.
- Method of Least Squares (Parabolic): This method fits a parabolic or quadratic curve to the data, resulting in an equation of the form $Y_c = a + bX + cX^2$. This is also a specific mathematical equation for the trend.
- Moving Average Method: This method calculates a series of average values by smoothing the data over a defined period. The result is a new set of smoothed data points that represent the trend. It does not produce a single algebraic equation that can be used to calculate the trend value for any given time period $X$ by plugging $X$ into a formula. You get a specific trend value for each period for which the moving average is calculated, but not a general function.
Therefore, the Moving Average Method is the one that does NOT provide a mathematical equation for the trend line; instead, it provides a series of calculated trend values.
The correct answer is (C) Moving Average Method.
Question 20. Case Study: The annual sales (in $\textsf{₹}$ lakhs) of a company for 5 years are given below:
Year | 2018 | 2019 | 2020 | 2021 | 2022 |
---|---|---|---|---|---|
Sales ($\textsf{₹}$ lakhs) | 10 | 12 | 15 | 13 | 18 |
Use the Method of Least Squares to fit a linear trend line $Y_c = a + bX$, taking 2020 as the origin for $X$.
Based on this data and method, answer the following questions:
What are the coded values of $X$ for the years 2018, 2019, 2020, 2021, and 2022 respectively?
(A) -2, -1, 0, 1, 2
(B) 18, 19, 20, 21, 22
(C) 0, 1, 2, 3, 4
(D) -2.5, -1.5, 0, 1.5, 2.5
Answer:
Explanation:
When fitting a trend line using the Method of Least Squares, it is often convenient to code the time variable ($X$) to simplify calculations, especially when using an odd number of periods. Coding involves setting a specific period as the origin and expressing other periods as deviations from the origin.
In this case, the year 2020 is taken as the origin. This means the coded value of $X$ for the year 2020 is 0.
For other years, the coded value is the difference between the year and the origin year (2020).
- For the year 2018: $X = 2018 - 2020 = -2$
- For the year 2019: $X = 2019 - 2020 = -1$
- For the year 2020: $X = 2020 - 2020 = 0$
- For the year 2021: $X = 2021 - 2020 = 1$
- For the year 2022: $X = 2022 - 2020 = 2$
Thus, the coded values of $X$ for the years 2018, 2019, 2020, 2021, and 2022 are -2, -1, 0, 1, and 2 respectively.
The correct answer is (A) -2, -1, 0, 1, 2.
Question 21. (Continuing from Question 20) Calculate $\sum Y$ and $\sum X$ using the coded values of $X$ from Question 20.
(A) $\sum Y = 68, \sum X = 0$
(B) $\sum Y = 68, \sum X = 100$
(C) $\sum Y = 68, \sum X = 5$
(D) $\sum Y = 68, \sum X = -1$
Answer:
Explanation:
From Question 20, we have the following data and coded X values:
Year | Sales (Y) | Coded X
-----|-----------|--------
2018 | 10 | -2
2019 | 12 | -1
2020 | 15 | 0
2021 | 13 | 1
2022 | 18 | 2
Now, we calculate the sum of the sales values ($\sum Y$) and the sum of the coded X values ($\sum X$).
$\sum Y = 10 + 12 + 15 + 13 + 18$
$\sum Y = 68$
$\sum X = (-2) + (-1) + 0 + 1 + 2$
$\sum X = -2 - 1 + 0 + 1 + 2$
$\sum X = -3 + 3$
$\sum X = 0$
Thus, $\sum Y = 68$ and $\sum X = 0$.
The correct answer is (A) $\sum Y = 68, \sum X = 0$.
Question 22. (Continuing from Question 20) Calculate $\sum XY$ and $\sum X^2$ using the coded values of $X$ from Question 20.
Year | 2018 | 2019 | 2020 | 2021 | 2022 | Total |
---|---|---|---|---|---|---|
Sales (Y) | 10 | 12 | 15 | 13 | 18 | 68 |
Coded X | -2 | -1 | 0 | 1 | 2 | 0 |
XY | -20 | -12 | 0 | 13 | 36 | 17 |
X$^2$ | 4 | 1 | 0 | 1 | 4 | 10 |
(A) $\sum XY = 17, \sum X^2 = 10$
(B) $\sum XY = 68, \sum X^2 = 0$
(C) $\sum XY = 10, \sum X^2 = 17$
(D) $\sum XY = 0, \sum X^2 = 10$
Answer:
Explanation:
We need to calculate the sum of the products of Y and coded X ($\sum XY$) and the sum of the squares of coded X ($\sum X^2$). The table provided in the question already lists these values for each year and their totals.
From the table:
The sum of XY values is:
$\sum XY = (-20) + (-12) + 0 + 13 + 36$
$\sum XY = -32 + 49$
$\sum XY = 17$
The sum of X$^2$ values is:
$\sum X^2 = 4 + 1 + 0 + 1 + 4$
$\sum X^2 = 10$
Thus, $\sum XY = 17$ and $\sum X^2 = 10$.
The correct answer is (A) $\sum XY = 17, \sum X^2 = 10$.
Question 23. (Continuing from Question 20) Using the normal equations from Question 9 and the calculated sums, find the values of $a$ and $b$. ($n=5$)
(A) $a = 13.6, b = 1.7$
(B) $a = 68, b = 17$
(C) $a = 17, b = 13.6$
(D) $a = 1.7, b = 13.6$
Answer:
Explanation:
From Question 20, the number of data points is $n = 5$.
From Question 21, the sum of Y values is $\sum Y = 68$, and the sum of coded X values is $\sum X = 0$.
From Question 22, the sum of XY values is $\sum XY = 17$, and the sum of X$^2$ values is $\sum X^2 = 10$.
Since the time variable $X$ is coded such that $\sum X = 0$, we can use the simplified normal equations for fitting a linear trend ($Y_c = a + bX$) from Question 9:
$\sum Y = na$
... (1)
$\sum XY = b \sum X^2$
... (2)
Now, we substitute the calculated sums and $n=5$ into these equations:
From equation (1):
$68 = 5 \times a$
To find $a$, divide both sides by 5:
$a = \frac{68}{5}$
$a = 13.6$
From equation (2):
$17 = b \times 10$
To find $b$, divide both sides by 10:
$b = \frac{17}{10}$
$b = 1.7$
So, the values for the trend line equation $Y_c = a + bX$ are $a = 13.6$ and $b = 1.7$.
The correct answer is (A) $a = 13.6, b = 1.7$.
Question 24. (Continuing from Question 20) The trend line equation with origin 2020 and unit 1 year for $X$ is:
(A) $Y_c = 1.7 + 13.6 X$
(B) $Y_c = 13.6 + 1.7 X$
(C) $Y_c = 68 + 17 X$
(D) $Y_c = 13.6 - 1.7 X$
Answer:
Explanation:
The general form of a linear trend line equation is given by $Y_c = a + bX$, where:
$Y_c$ is the calculated trend value.
$X$ is the time variable (coded in this case).
$a$ is the intercept (trend value at the origin).
$b$ is the slope (average change in Y per unit change in X).
From the solution to Question 23, using the Method of Least Squares with the origin at 2020 and the coded time variable $X$, we found the values for $a$ and $b$ to be:
$a = 13.6$
$b = 1.7$
Substituting these values into the linear trend equation $Y_c = a + bX$, we get:
$Y_c = 13.6 + 1.7X$
This equation represents the linear trend line for the annual sales data, with the origin at the year 2020 and the unit of $X$ being 1 year.
The correct answer is (B) $Y_c = 13.6 + 1.7 X$.
Question 25. Completion Question: The increase in sales of ice cream during summer months is an example of ____ variation in a time series.
(A) Secular
(B) Cyclical
(C) Seasonal
(D) Irregular
Answer:
Explanation:
We need to identify which component of a time series is characterized by fluctuations that repeat regularly within a period of one year.
- Secular Trend: Long-term, smooth movement over many years.
- Cyclical Variation: Long-term oscillations around the trend, lasting longer than a year, often related to business cycles.
- Irregular Variation: Unpredictable, random fluctuations caused by sudden events.
- Seasonal Variation: Regular fluctuations that repeat within a year, driven by seasons, holidays, or other calendar-related factors.
The increase in ice cream sales during summer months is a classic example of a pattern that repeats every year during the same period (summer). This regular, within-a-year pattern is the definition of Seasonal Variation.
The correct answer is (C) Seasonal.
Question 26. Which of the following is NOT a component of time series data?
(A) Random Component
(B) Population Component
(C) Cyclical Component
(D) Seasonal Component
Answer:
Explanation:
Standard time series decomposition models (like additive or multiplicative) typically identify four main components:
- Trend Component (T) or Secular Trend: Represents the long-term underlying direction.
- Seasonal Component (S): Represents regular fluctuations within a year.
- Cyclical Component (C): Represents long-term oscillations or swings around the trend, lasting longer than a year.
- Irregular Component (I) or Random Component: Represents unpredictable, random fluctuations.
Looking at the options:
- (A) Random Component is a recognized component (same as Irregular Component).
- (B) Population Component is not a standard component in time series decomposition. While population growth might contribute to the secular trend in some time series (e.g., birth rates), "Population Component" itself is not one of the generally accepted components of a time series.
- (C) Cyclical Component is a standard component.
- (D) Seasonal Component is a standard component.
Therefore, Population Component is not a standard component of time series data in the context of typical decomposition methods.
The correct answer is (B) Population Component.
Question 27. The base period for an index number is the period against which comparisons are made. The index number for the base period is always taken as:
(A) 0
(B) 1
(C) 100
(D) Equal to the current period index
Answer:
Explanation:
An index number is a statistical measure designed to show changes in a variable or group of related variables over time, relative to a base period.
The formula for a simple index number is typically:
Index Number $= \frac{\text{Value in Current Period}}{\text{Value in Base Period}} \times \text{Base Index Value}$
The base period is chosen as a reference point, and its value is set to a specific index value. By convention, the index number for the base period is usually set to 100.
If we apply the formula to the base period, the "Value in Current Period" is the same as the "Value in Base Period". So, if the Base Index Value is 100:
Index Number for Base Period $= \frac{\text{Value in Base Period}}{\text{Value in Base Period}} \times 100 = 1 \times 100 = 100$
Thus, the index number for the base period is always taken as 100.
The correct answer is (C) 100.
Question 28. If the price of a commodity was $\textsf{₹}50$ in the base period and is $\textsf{₹}60$ in the current period, the price relative for the current period is:
(A) 120
(B) 83.33
(C) 100
(D) 20
Answer:
Explanation:
A price relative for a given period measures the price in that period as a percentage of the price in a base period. The formula is:
Price Relative $= \frac{\text{Price in current period}}{\text{Price in base period}} \times 100$
Given:
Price in base period = $\textsf{₹}50$
Price in current period = $\textsf{₹}60$
Substituting these values into the formula:
Price Relative $= \frac{60}{50} \times 100$
Price Relative $= 1.2 \times 100$
Price Relative $= 120$
The price relative for the current period is 120.
The correct answer is (A) 120.
Question 29. Data for which only one variable is recorded over time (e.g., population over years) is called:
(A) Bivariate data
(B) Multivariate data
(C) Univariate data
(D) Cross-sectional data
Answer:
Explanation:
Statistical data can be classified based on the number of variables being considered:
- Univariate data: Data where observations are recorded for only one variable. For example, recording the heights of students in a class, or the population of a country over time (the variable is population, recorded at different time points).
- Bivariate data: Data where observations are recorded for two variables for each unit. For example, recording both the height and weight of students in a class.
- Multivariate data: Data where observations are recorded for more than two variables for each unit. For example, recording height, weight, age, and income for individuals.
- Cross-sectional data: Data collected at a single point in time from multiple subjects or entities. For example, the sales figures for different companies in a specific year. This contrasts with time series data, which tracks a variable over time.
Data for which only one variable (like population, sales, temperature, etc.) is recorded over different points in time is a characteristic example of Univariate data.
The correct answer is (C) Univariate data.
Question 30. Which method for measuring trend is considered more objective and mathematically sound?
(A) Graphical Method
(B) Moving Average Method
(C) Method of Least Squares
(D) All methods are equally objective
Answer:
Explanation:
Let's evaluate the objectivity and mathematical basis of each method for measuring trend:
- Graphical Method: This method involves plotting the data and drawing a trend line or curve freehand. It is highly subjective as the exact placement and shape of the line depend entirely on the individual analyst's interpretation and drawing. It is not based on a precise mathematical formula or optimization criterion.
- Moving Average Method: This method calculates averages over a specific period to smooth the data. While the calculation itself is mathematical, the choice of the period length for the moving average can be somewhat subjective, and the method produces a series of smoothed values rather than a single mathematical equation for the trend line. It doesn't fit a predefined mathematical curve.
- Method of Least Squares: This method involves fitting a specific mathematical function (like a straight line $Y_c = a + bX$) to the data by minimizing the sum of the squared differences between the actual values and the values predicted by the function. This is based on a clearly defined mathematical principle (minimizing $\sum (Y - Y_c)^2$) and results in a unique set of parameters for the chosen function. Once the functional form is chosen, the calculation of the trend equation is objective and mathematically sound.
Therefore, the Method of Least Squares is considered more objective and mathematically sound among the given options because it relies on a specific mathematical criterion to determine the best-fitting trend line or curve, eliminating subjective judgment from the fitting process.
The correct answer is (C) Method of Least Squares.
Question 31. If a 5-year moving average is calculated for a time series of 10 years, how many moving average values will be obtained?
(A) 10
(B) 5
(C) 6
(D) 7
Answer:
Explanation:
When calculating a $k$-period moving average for a time series with $n$ observations, you lose data points at the beginning and end of the series because you need $k$ consecutive data points to compute each average.
The number of $k$-period moving average values that can be calculated from a time series of length $n$ is given by the formula:
Number of moving average values $= n - k + 1$
In this question, the total number of years (observations) is $n = 10$, and the period of the moving average is $k = 5$ years.
Using the formula:
Number of moving average values $= 10 - 5 + 1$
Number of moving average values $= 5 + 1$
Number of moving average values $= 6$
Thus, 6 moving average values will be obtained from a 10-year time series using a 5-year moving average.
The correct answer is (C) 6.
Question 32. When the number of periods in a moving average is even (e.g., 4-year moving average), centering is required. Centering involves calculating a further average of:
(A) The original data points
(B) The uncentered moving averages
(C) The trend values
(D) The seasonal indices
Answer:
Explanation:
When you calculate an even-period moving average (e.g., 4-year, 6-year), the calculated average value falls between two time periods. For example, a 4-year moving average of years 1, 2, 3, and 4 is typically placed between year 2 and year 3. For many applications (like finding the trend value corresponding to a specific year), you need the average to align with a specific time point (like the middle of a year).
To align the moving average with a specific time period, a process called centering is used. Centering involves calculating a 2-period moving average of the previously calculated uncentered moving averages. For a 4-year moving average, this means averaging the first two 4-year moving averages, then the second and third, and so on.
For example, if you have uncentered 4-year moving averages for periods 2.5 (average of years 1-4) and 3.5 (average of years 2-5), centering involves averaging these two values: $\frac{\text{MA}_{1-4} + \text{MA}_{2-5}}{2}$. This result is then centered at period 3.
Therefore, centering for an even-period moving average involves calculating a further average of the uncentered moving averages.
The correct answer is (B) The uncentered moving averages.
Question 33. Assertion (A): The method of least squares can be used to fit both linear and non-linear trends.
Reason (R): The method minimizes the sum of squared errors, and the form of the trend equation ($Y_c = a+bX$, $Y_c = a+bX+cX^2$, etc.) can be chosen based on the observed pattern in the data.
(A) Both A and R are true and R is the correct explanation of A.
(B) Both A and R are true but R is not the correct explanation of A.
(C) A is true but R is false.
(D) A is false but R is true.
Answer:
Explanation:
Let's analyze the Assertion (A) and the Reason (R).
Assertion (A): The method of least squares can be used to fit both linear and non-linear trends.
This statement is True. The Method of Least Squares is a general regression technique. While it is often used to fit a straight line ($Y_c = a + bX$), it can also be used to fit non-linear relationships by choosing an appropriate functional form, such as a quadratic equation ($Y_c = a + bX + cX^2$), an exponential curve (often by transforming the data, like $\log Y_c = \log a + X \log b$), or other mathematical models.
Reason (R): The method minimizes the sum of squared errors, and the form of the trend equation ($Y_c = a+bX$, $Y_c = a+bX+cX^2$, etc.) can be chosen based on the observed pattern in the data.
This statement is also True. The fundamental principle of the Method of Least Squares is to find the curve (of a chosen mathematical form) that minimizes the sum of the squared vertical distances between the actual data points and the points on the fitted curve. The choice of the mathematical form of the trend equation (linear, quadratic, etc.) is indeed made by the analyst, often by examining a scatter plot of the time series to discern the underlying pattern.
The reason (R) explains why Assertion (A) is true. Because the method's objective is to minimize squared errors for a *chosen* functional form, and we can *choose* linear or various non-linear forms based on the data pattern, the method is applicable to fitting both types of trends.
Therefore, both Assertion (A) and Reason (R) are true, and Reason (R) is the correct explanation of Assertion (A).
The correct answer is (A) Both A and R are true and R is the correct explanation of A.
Question 34. In the Method of Least Squares, fitting a parabolic trend ($Y_c = a + bX + cX^2$) is suitable when the time series shows a trend that is:
(A) Strictly linear
(B) Curved (either upwards or downwards)
(C) Highly seasonal
(D) Purely random
Answer:
Explanation:
The equation $Y_c = a + bX + cX^2$ represents a parabola. A parabolic curve can be U-shaped (opening upwards) or inverted U-shaped (opening downwards), depending on the sign of the coefficient $c$. This shape allows the trend to accelerate or decelerate over time.
- Strictly linear: A linear trend is best fitted by a straight line equation, $Y_c = a + bX$.
- Curved (either upwards or downwards): If the time series data, after accounting for seasonal and irregular fluctuations, shows a systematic curvature over the long term, a parabolic trend is a suitable model to capture this non-linear movement. For example, growth that is initially slow but accelerates, or growth that starts rapidly and then slows down.
- Highly seasonal: Seasonality refers to within-year patterns and is a separate component from the trend. Fitting a trend line addresses the long-term movement, not the seasonal fluctuations.
- Purely random: A purely random time series has no discernible trend, seasonality, or cyclical components. Fitting any trend line (linear or parabolic) would not be appropriate.
Therefore, fitting a parabolic trend using the Method of Least Squares is suitable when the underlying long-term movement of the time series is curved (either upwards or downwards) rather than strictly linear.
The correct answer is (B) Curved (either upwards or downwards).
Question 35. The primary purpose of analyzing a time series is:
(A) To describe the past behavior of the data.
(B) To understand the underlying patterns and forces affecting the data.
(C) To forecast future values.
(D) All of the above.
Answer:
Explanation:
Time series analysis is a statistical method used to analyze data collected sequentially over time. Its objectives are typically multifaceted:
- To describe the past behavior: By examining historical data, we can visualize and summarize the trends, seasonal patterns, and irregular fluctuations that have occurred.
- To understand the underlying patterns and forces: Decomposing a time series into its components (Trend, Seasonality, Cyclical, Irregular) helps in identifying the systematic patterns and understanding the factors that influence the variable over time. This understanding is crucial for making informed decisions.
- To forecast future values: Based on the identified patterns and trends in the past and present data, time series models can be used to predict future values of the variable. This is one of the most significant applications of time series analysis in various fields like economics, business, and science.
All the options listed are valid purposes of time series analysis. Describing past behavior and understanding underlying patterns are essential steps that lead to the ultimate goal of forecasting or making decisions based on the time series data.
Therefore, the primary purpose encompasses all these aspects.
The correct answer is (D) All of the above.
Question 36. If the trend equation is $Y_c = 50 + 2X$ (with origin Year 2010, unit 1 year), what is the trend value for the year 2015?
(A) 50
(B) $50 + 2(5) = 60$
(C) $50 + 2(10) = 70$
(D) $50 + 2(2015 - 2010) = 50 + 2(5) = 60$
Answer:
The given trend equation is $Y_c = 50 + 2X$.
The origin of the trend equation is the Year 2010, and the unit is 1 year. This means that $X$ represents the number of years from 2010.
We need to find the trend value for the year 2015.
To find the value of $X$ for the year 2015, we subtract the origin year from the target year:
$X = 2015 - 2010$
... (i)
From (i), we get $X = 5$.
Now, substitute the value of $X$ into the trend equation:
$Y_c = 50 + 2(5)$
... (ii)
Calculate the trend value:
$Y_c = 50 + 10$
... (iii)
$Y_c = 60$
... (iv)
Comparing this result with the given options, we find that option (D) correctly shows the calculation and the result.
The correct option is (D).
Question 37. The Method of Least Squares is preferred over the Graphical Method because:
(A) It is easier to apply.
(B) It gives a unique and objective trend line.
(C) It requires less data.
(D) It can only fit linear trends.
Answer:
The Method of Least Squares is preferred over the Graphical Method because it provides a unique and objective trend line.
Let's analyze the options:
(A) It is easier to apply: The Graphical Method is generally considered easier to apply and understand, especially for quick estimations, whereas the Method of Least Squares involves calculations.
(B) It gives a unique and objective trend line: The Method of Least Squares uses a mathematical formula to find the best-fitting line, which ensures that for a given dataset, the resulting trend line is always the same and is not influenced by personal judgment. This makes it objective and unique.
(C) It requires less data: Both methods require data to fit a trend line. The amount of data required is not a distinguishing factor between these two methods in terms of preference.
(D) It can only fit linear trends: While the Method of Least Squares is commonly used for linear trends, it can also be extended to fit non-linear trends (e.g., polynomial trends) by modifying the equation.
Therefore, the primary reason for preferring the Method of Least Squares is its objectivity and the uniqueness of the derived trend line.
The correct option is (B).
Question 38. Data Interpretation: The population (in thousands) of a town for 7 years is given below:
Year | 2016 | 2017 | 2018 | 2019 | 2020 | 2021 | 2022 |
---|---|---|---|---|---|---|---|
Population (Y) | 10 | 12 | 13 | 15 | 17 | 18 | 20 |
Use the 3-year moving average method to find the trend values.
Based on this data and method, answer the following questions:
What is the 3-year moving average for the year 2017?
(A) $(10+12+13)/3 = 35/3 \approx 11.67$
(B) $(12+13+15)/3 = 40/3 \approx 13.33$
(C) $(13+15+17)/3 = 45/3 = 15$
(D) 12
Answer:
To find the 3-year moving average for a specific year, we need to average the population of that year and the two adjacent years.
The 3-year moving average is calculated for the middle year of the three consecutive years. The first 3-year moving average can be calculated for the year 2017, using the population data from 2016, 2017, and 2018.
The formula for the 3-year moving average centered at Year $t$ is:
$MA_t = \frac{Y_{t-1} + Y_t + Y_{t+1}}{3}$
... (i)
Here, we want to find the moving average for the year 2017. So, $t = 2017$.
The population values are:
For Year 2016 ($Y_{t-1}$): 10 (in thousands)
For Year 2017 ($Y_t$): 12 (in thousands)
For Year 2018 ($Y_{t+1}$): 13 (in thousands)
Substitute these values into the formula (i):
$MA_{2017} = \frac{Y_{2016} + Y_{2017} + Y_{2018}}{3}$
... (ii)
$MA_{2017} = \frac{10 + 12 + 13}{3}$
... (iii)
$MA_{2017} = \frac{35}{3}$
... (iv)
$MA_{2017} \approx 11.67$
... (v)
Comparing this with the given options, we see that option (A) matches our calculation.
The correct option is (A).
Question 39. (Continuing from Question 38) What is the 3-year moving average for the year 2021?
(A) $(17+18+20)/3 = 55/3 \approx 18.33$
(B) $(15+17+18)/3 = 50/3 \approx 16.67$
(C) $(18+20)/2 = 19$
(D) 18
Answer:
We need to find the 3-year moving average for the year 2021. According to the 3-year moving average method, this will be the average of the population for the years 2020, 2021, and 2022.
The population values are:
For Year 2020 ($Y_{t-1}$): 17 (in thousands)
For Year 2021 ($Y_t$): 18 (in thousands)
For Year 2022 ($Y_{t+1}$): 20 (in thousands)
Using the formula for the 3-year moving average:
$MA_{2021} = \frac{Y_{2020} + Y_{2021} + Y_{2022}}{3}$
... (i)
Substitute the population values:
$MA_{2021} = \frac{17 + 18 + 20}{3}$
... (ii)
$MA_{2021} = \frac{55}{3}$
... (iii)
$MA_{2021} \approx 18.33$
... (iv)
Comparing this result with the given options, we find that option (A) matches our calculation.
The correct option is (A).
Question 40. (Continuing from Question 38) How many trend values are obtained using the 3-year moving average method for this 7-year data?
(A) 7
(B) 5
(C) 3
(D) 4
Answer:
In the 3-year moving average method, we calculate the average of three consecutive data points. For a dataset of $n$ data points, the number of moving averages that can be calculated is $n - k + 1$, where $k$ is the period of the moving average (in this case, $k=3$).
The given data has $n=7$ years (from 2016 to 2022).
The formula for the number of moving averages is:
Number of Trend Values = $n - k + 1$
... (i)
Here, $n = 7$ and $k = 3$.
Substituting these values into the formula:
Number of Trend Values = $7 - 3 + 1$
... (ii)
Number of Trend Values = $5$
... (iii)
Alternatively, we can list the years for which we can calculate the 3-year moving average:
- To calculate the first moving average, we need data from 2016, 2017, and 2018. This average is centered at 2017.
- The subsequent moving averages will be centered at 2018, 2019, 2020, and 2021.
- To calculate the last moving average, we need data from 2020, 2021, and 2022. This average is centered at 2021.
The years for which we can calculate the 3-year moving average are 2017, 2018, 2019, 2020, and 2021. This gives us a total of 5 trend values.
Comparing this with the given options, we find that option (B) matches our result.
The correct option is (B).
Question 41. Which component of time series is easiest to predict?
(A) Irregular Variation
(B) Seasonal Variation
(C) Cyclical Variation
(D) Secular Trend
Answer:
The component of a time series that is generally easiest to predict is the Secular Trend.
Let's consider each component:
- Secular Trend: This represents the long-term movement or direction of the data, such as an overall increase or decrease over many years. Trends are often relatively smooth and can be identified and projected forward using various statistical methods like moving averages or regression analysis.
- Seasonal Variation: This refers to patterns that repeat over a fixed period, usually within a year (e.g., sales increasing during holidays). While predictable within its cycle, forecasting the exact magnitude can be influenced by other factors.
- Cyclical Variation: These are longer-term fluctuations that occur over periods longer than one year, often related to business cycles. They are less predictable than seasonal variations because their timing and amplitude are not fixed.
- Irregular Variation (or Random Variation): This component represents unpredictable, random fluctuations that are not explained by the other components. These are inherently difficult, if not impossible, to predict.
Because secular trends represent a long-term, often smooth movement, they are generally the most predictable component of a time series.
The correct option is (D).
Question 42. A major technological advancement leading to increased production efficiency over several years would primarily affect which component of a time series of production output?
(A) Seasonal Variation
(B) Cyclical Variation
(C) Secular Trend
(D) Irregular Variation
Answer:
A major technological advancement that leads to increased production efficiency over several years would primarily affect the Secular Trend component of a time series of production output.
Let's break down why:
- Secular Trend: This component represents the long-term direction or movement of a time series. A significant technological advancement that boosts efficiency over an extended period directly contributes to an upward long-term movement or trend in production output.
- Seasonal Variation: This component relates to patterns that repeat within a year (e.g., increased production during peak demand seasons). While the advancement might indirectly influence seasonal patterns, its primary impact is on the overall long-term growth.
- Cyclical Variation: This component refers to fluctuations that occur over periods longer than a year, often related to economic cycles. Technological advancements can influence business cycles, but the advancement itself directly impacts the production capacity, which is a trend.
- Irregular Variation: This component accounts for random, unpredictable fluctuations. While a technological advancement might cause a temporary surge or dip due to its implementation, its sustained impact on efficiency is a trend, not a random event.
Therefore, the sustained increase in production efficiency due to a technological advancement over several years directly shapes the long-term direction of production output, which is the secular trend.
The correct option is (C).
Question 43. If the trend equation is $Y_c = 100 + 5X$ (with origin 2000, unit 1 year), the trend value for July 2001 is calculated using $X$ value as:
(A) 1
(B) 1.5
(C) 2001
(D) 100 + 5(1.5)
Answer:
The trend equation is given as $Y_c = 100 + 5X$, with the origin at the year 2000 and a unit of 1 year.
This means that $X$ represents the number of years that have passed since the origin year, 2000.
We need to determine the value of $X$ for July 2001.
The year 2000 corresponds to $X=0$.
The year 2001 starts after the completion of the year 2000, so the beginning of 2001 corresponds to $X=1$.
July is the seventh month of the year. Since the unit is 1 year, we need to consider the fraction of the year that has passed by July 2001.
The year 2001 is 1 full year after the origin year 2000.
Within the year 2001, July is the 7th month. There are 12 months in a year.
So, the fraction of the year 2001 that has passed by July is $\frac{7}{12}$.
Therefore, the value of $X$ for July 2001 is 1 (for the full year passed since the origin) plus the fraction of the current year that has passed.
$X = 1 + \frac{7}{12}$
... (i)
Calculating the value:
$X = 1 + 0.5833...$
... (ii)
$X \approx 1.5833$
... (iii)
However, looking at the options, it seems the question is simplified, or the options are presented in a way that suggests an approximation or a specific interpretation of how to handle months.
Option (A) 1 would represent the start of the year 2001.
Option (B) 1.5 suggests that July is considered as halfway through the year, which is $\frac{7}{12} \approx 0.58$, so 1.5 is a close approximation for $1 + \frac{6}{12}$ or the midpoint of the year.
Option (C) 2001 is the actual year, not the value of $X$.
Option (D) $100 + 5(1.5)$ is the calculation itself, not the value of $X$.
Given the options, the most plausible interpretation for the $X$ value to calculate the trend for July 2001, considering the unit is 1 year, is to represent 1 full year plus half of the current year, which is 1.5.
The correct option for the $X$ value used in the calculation is (B).
Question 44. In fitting a parabolic trend $Y_c = a + bX + cX^2$ using the Method of Least Squares, when is it advantageous to code the time variable $X$ such that $\sum X = 0$ and $\sum X^3 = 0$?
(A) When the number of years is odd.
(B) When the number of years is even.
(C) Always.
(D) Never.
Answer:
It is advantageous to code the time variable $X$ such that $\sum X = 0$ and $\sum X^3 = 0$ when the number of years in the data set is odd.
Let's understand why:
When fitting a parabolic trend $Y_c = a + bX + cX^2$, the normal equations derived from the Method of Least Squares involve sums of powers of $X$ (i.e., $\sum Y$, $\sum X$, $\sum X^2$, $\sum X^3$, $\sum X^4$, $\sum XY$, $\sum X^2Y$).
If we code the time variable $X$ for an odd number of years such that the middle year is assigned $X=0$, then the subsequent years are $X=1, 2, 3, \dots$ and the preceding years are $X=-1, -2, -3, \dots$.
For such a coding scheme with an odd number of observations:
- The sum of $X$ terms will be zero because for every positive value of $X$, there is a corresponding negative value, and they cancel each other out. For example, with 5 years, $X$ values would be -2, -1, 0, 1, 2, and $\sum X = -2 + (-1) + 0 + 1 + 2 = 0$.
- The sum of odd powers of $X$ (like $X^3$) will also be zero. This is because $(-X)^3 = -X^3$, so the terms will cancel out. For example, $(-2)^3 + (-1)^3 + 0^3 + 1^3 + 2^3 = -8 - 1 + 0 + 1 + 8 = 0$.
When $\sum X = 0$ and $\sum X^3 = 0$, the normal equations simplify significantly, making the calculation of the coefficients $a$, $b$, and $c$ much easier.
For example, one of the normal equations is $\sum XY = b \sum X^2 + c \sum X^3$. If $\sum X^3 = 0$, this equation simplifies to $\sum XY = b \sum X^2$.
If the number of years is even, it's not possible to assign $X=0$ to a specific year and have the sums of odd powers of $X$ be zero with integers. Typically, for an even number of years, the coding involves assigning values like $\pm 1, \pm 3, \pm 5, \dots$ to make the sums of odd powers zero, but this is a different coding strategy. The simplest and most advantageous coding leading to $\sum X=0$ and $\sum X^3=0$ is when the number of years is odd and the central year is $X=0$.
Therefore, it is advantageous to code the time variable $X$ such that $\sum X = 0$ and $\sum X^3 = 0$ when the number of years is odd.
The correct option is (A).
Question 45. The period of cyclical variation is generally:
(A) Less than one year
(B) Exactly one year
(C) More than one year but less than 10 years
(D) More than one year and can vary from cycle to cycle
Answer:
The period of cyclical variation is generally more than one year and can vary from cycle to cycle.
Let's analyze the components of a time series:
- Secular Trend: Long-term movement, typically increasing or decreasing over many years.
- Seasonal Variation: Patterns that repeat at fixed intervals, usually within a year (e.g., daily, weekly, monthly, quarterly). The period is fixed, e.g., 1 year.
- Cyclical Variation: Fluctuations that occur over periods longer than one year, often related to economic business cycles (e.g., periods of boom and recession). The duration of these cycles is not fixed and can vary. For example, a business cycle might last 5 years one time and 8 years another time.
- Irregular Variation: Random, unpredictable fluctuations.
Option (A) describes seasonal variation.
Option (B) describes seasonal variation.
Option (C) is a possible range for some cycles, but it doesn't capture the variability in duration.
Option (D) accurately describes the nature of cyclical variations, which are longer-term fluctuations with a period that is not fixed and can vary between cycles.
The correct option is (D).
Question 46. Which of the following is NOT a method for measuring Secular Trend?
(A) Method of Semi-Averages
(B) Method of Least Squares
(C) Ratio to Trend Method
(D) Moving Average Method
Answer:
The method that is NOT primarily used for measuring Secular Trend among the given options is the Ratio to Trend Method.
Let's examine each method:
- Method of Semi-Averages: This method involves dividing the time series data into two equal halves, calculating the average of each half, and then fitting a straight line through these two average points. This is a method to determine the secular trend.
- Method of Least Squares: This is a widely used statistical method for fitting a trend line (linear or non-linear) by minimizing the sum of the squared differences between the actual data points and the trend line. It is a primary method for measuring secular trend.
- Ratio to Trend Method: This method is used to measure the *seasonal component* of a time series. It involves calculating the ratio of the actual value to the trend value for each period. These ratios are then averaged to find the seasonal indices.
- Moving Average Method: This method calculates a series of averages of subsets of the data. A moving average smooths out short-term fluctuations and irregular variations, thereby revealing the underlying trend. It is a common method for estimating secular trend.
Since the Ratio to Trend Method is used to isolate and measure seasonal variations after the trend has already been established, it is not a method for measuring the secular trend itself.
The correct option is (C).
Question 47. If a time series represents quarterly sales data, what moving average period would be appropriate to eliminate seasonal variation?
(A) 3-period moving average
(B) 4-period moving average
(C) 5-period moving average
(D) 12-period moving average
Answer:
If a time series represents quarterly sales data, a 4-period moving average would be appropriate to eliminate seasonal variation.
Here's why:
- Quarterly Data: Quarterly data means there are 4 data points within a year.
- Seasonal Variation: Seasonal variation refers to patterns that repeat within a year. For quarterly data, the seasonal cycle completes every 4 quarters.
- Moving Average for Seasonal Elimination: To eliminate or smooth out seasonal variations, we use a moving average with a period equal to the length of the seasonal cycle. This is because when you average over a full seasonal cycle, the seasonal effects within that cycle tend to cancel each other out, leaving the underlying trend and cyclical components.
Applying this to quarterly data:
- A 4-period moving average will average data from four consecutive quarters. For example, for the first moving average point, you might average quarters 1, 2, 3, and 4 of a year. The next moving average point would average quarters 2, 3, 4, and 1 of the next year, and so on. This process effectively averages out the seasonal effects that occur within each year.
Let's consider the other options:
- (A) 3-period moving average: This would smooth out fluctuations but would not specifically eliminate a 4-quarter seasonal pattern.
- (C) 5-period moving average: This period does not align with the quarterly seasonal cycle.
- (D) 12-period moving average: This would be appropriate for monthly data to eliminate annual seasonal variation.
Therefore, for quarterly data, a 4-period moving average is the correct choice to remove seasonal effects.
The correct option is (B).
Question 48. Assertion (A): The Method of Least Squares is used to fit a trend line that best represents the underlying pattern in the data.
Reason (R): The principle of least squares states that the best fitting line is the one that minimizes the sum of the absolute differences between the observed and fitted values.
(A) Both A and R are true and R is the correct explanation of A.
(B) Both A and R are true but R is not the correct explanation of A.
(C) A is true but R is false.
(D) A is false but R is true.
Answer:
Let's evaluate the Assertion (A) and the Reason (R).
Assertion (A): The Method of Least Squares is used to fit a trend line that best represents the underlying pattern in the data.
This statement is True. The primary goal of the Method of Least Squares is to find a line (or curve) that fits the data points as closely as possible, thereby representing the underlying pattern or trend.
Reason (R): The principle of least squares states that the best fitting line is the one that minimizes the sum of the absolute differences between the observed and fitted values.
This statement is False. The principle of least squares states that the best fitting line is the one that minimizes the sum of the squared differences (also known as residuals) between the observed and fitted values, not the absolute differences.
Since Assertion (A) is true and Reason (R) is false, the correct option is (C).
The correct option is (C).
Question 49. If the trend equation for annual data is $Y_c = 200 + 10X$ (Origin 2010, X unit 1 year), what is the trend value for the first half of 2016?
(A) $200 + 10(6) = 260$
(B) $200 + 10(5.5) = 255$
(C) $200 + 10(6.5) = 265$
(D) $200 + 10(2016 - 2010.5) = 200 + 10(5.5) = 255$
Answer:
The trend equation is $Y_c = 200 + 10X$, with the origin at the year 2010 and the unit of $X$ being 1 year.
We need to find the trend value for the first half of 2016.
The origin year 2010 corresponds to $X=0$.
The year 2011 corresponds to $X=1$.
The year 2012 corresponds to $X=2$.
The year 2013 corresponds to $X=3$.
The year 2014 corresponds to $X=4$.
The year 2015 corresponds to $X=5$.
The year 2016 corresponds to $X=6$.
The first half of 2016 means the midpoint of the year 2016.
If $X=5$ represents the *entire* year 2015, then the start of 2016 (i.e., January 1st, 2016) would be at $X = 5 + \frac{6}{12} = 5.5$ if we consider the origin to be the beginning of 2010. However, the origin is usually considered the start of the year.
Let's clarify the $X$ value for the *first half* of 2016. The year 2015 is represented by $X=5$. The year 2016 starts after 2015. The "first half of 2016" means that $5.5$ years have passed since the origin (beginning of 2010).
So, the $X$ value for the first half of 2016 is $2016 - 2010.5 = 5.5$.
This can also be seen as completing 5 full years (2010, 2011, 2012, 2013, 2014, 2015) which brings us to the end of 2015, corresponding to $X=5$. Then, the first half of 2016 means we add 0.5 years. So, $X = 5 + 0.5 = 5.5$.
Now, substitute $X=5.5$ into the trend equation:
$Y_c = 200 + 10(5.5)$
... (i)
Calculate the value:
$Y_c = 200 + 55$
... (ii)
$Y_c = 255$
... (iii)
Comparing this with the options, both option (B) and (D) present the correct calculation and result. However, option (D) explicitly shows the calculation of $X$ as $(2016 - 2010.5)$, which is the correct way to determine the $X$ value for the midpoint of a year when the origin is the beginning of a year.
The correct option is (D).
Question 50. A time series plot shows consistent peaks in sales during the festive season every year. This pattern is attributable to:
(A) Secular Trend
(B) Cyclical Variation
(C) Seasonal Variation
(D) Irregular Variation
Answer:
The consistent peaks in sales during the festive season every year are attributable to Seasonal Variation.
Let's analyze why:
- Seasonal Variation: This component of a time series refers to patterns that repeat over a fixed period, typically within a year. Festive seasons occur at predictable times each year (e.g., holidays like Christmas, Diwali, etc.), leading to increased sales during those periods. The consistency "every year" is a key characteristic of seasonality.
- Secular Trend: This is the long-term upward or downward movement of the data, not specific peaks within a year.
- Cyclical Variation: These are longer-term fluctuations (more than a year) related to economic cycles, not the predictable yearly festive season.
- Irregular Variation: These are random, unpredictable fluctuations that do not follow a pattern. The consistent peaks during the festive season are predictable, so they are not irregular.
Therefore, the phenomenon described is a clear example of seasonal variation.
The correct option is (C).
Question 51. If the trend line equation with origin 2010 and X unit 1 year is $Y_c = 150 + 8X$, what is the equation if the origin is shifted to 2015?
(A) $Y_c = 150 + 8(X+5) = 190 + 8X$
(B) $Y_c = 150 + 8(X-5) = 110 + 8X$
(C) $Y_c = 150 + 8X$ (equation remains the same)
(D) $Y_c = (150+5) + 8X = 155 + 8X$
Answer:
The original trend equation is $Y_c = 150 + 8X$, where the origin is 2010 and the unit of $X$ is 1 year. This means that for any year $Y$, the value of $X$ is $Y - 2010$.
We want to shift the origin to 2015. Let the new time variable be $X'$.
If the new origin is 2015, then for any year $Y$, the value of $X'$ is $Y - 2015$.
We need to express the original $X$ in terms of the new $X'$.
We know:
$X = Y - 2010$
... (i)
And
$X' = Y - 2015$
... (ii)
From equation (ii), we can express $Y$ as:
$Y = X' + 2015$
... (iii)
Now, substitute equation (iii) into equation (i) to express $X$ in terms of $X'$:
$X = (X' + 2015) - 2010$
... (iv)
$X = X' + 5$
... (v)
Now, substitute this expression for $X$ into the original trend equation:
$Y_c = 150 + 8(X)$
... (vi)
$Y_c = 150 + 8(X' + 5)$
... (vii)
Expand and simplify the equation:
$Y_c = 150 + 8X' + 40$
... (viii)
$Y_c = 190 + 8X'$
... (ix)
This new equation represents the trend with the origin shifted to 2015. Comparing this result with the given options, option (A) matches our derived equation.
The correct option is (A).
Question 52. Data Interpretation: Annual production (in thousand units) of a factory for 6 years is given:
Year | 2017 | 2018 | 2019 | 2020 | 2021 | 2022 |
---|---|---|---|---|---|---|
Production (Y) | 50 | 55 | 62 | 60 | 68 | 75 |
Use the Method of Least Squares to fit a linear trend $Y_c = a+bX$, taking the origin between 2019 and 2020.
Based on this data and method, answer the following questions:
What are the coded values of $X$ for the years 2017 to 2022 respectively?
(A) -3, -2, -1, 1, 2, 3
(B) -2.5, -1.5, -0.5, 0.5, 1.5, 2.5
(C) -5, -3, -1, 1, 3, 5
(D) -2, -1, 0, 1, 2, 3
Answer:
The problem states that the origin is taken between 2019 and 2020. This means we are setting the point exactly halfway between these two years as $X=0$.
When the origin is placed between two consecutive years (for an even number of observations), we typically assign values of $\pm 0.5, \pm 1.5, \pm 2.5$, and so on, with a unit of 1 year.
Since the origin is between 2019 and 2020, let's assign $X=0$ to this point.
- The year 2019 is half a year before the origin. So, its $X$ value will be $-0.5$.
- The year 2020 is half a year after the origin. So, its $X$ value will be $+0.5$.
Following this pattern for the other years, with a unit of 1 year:
- For 2019: $X = -0.5$
- For 2020: $X = +0.5$
- For 2018 (one year before 2019): $X = -0.5 - 1 = -1.5$
- For 2021 (one year after 2020): $X = +0.5 + 1 = +1.5$
- For 2017 (one year before 2018): $X = -1.5 - 1 = -2.5$
- For 2022 (one year after 2021): $X = +1.5 + 1 = +2.5$
So, the coded values of $X$ for the years 2017 to 2022 are:
2017: $-2.5$
2018: $-1.5$
2019: $-0.5$
2020: $+0.5$
2021: $+1.5$
2022: $+2.5$
This sequence matches option (B).
The correct option is (B).
Question 53. (Continuing from Question 52) Calculate $\sum Y$ and $\sum X$ using the coded values of $X$ from Question 52 (using option (C) for coding, i.e., -5, -3, -1, 1, 3, 5).
(A) $\sum Y = 370, \sum X = 0$
(B) $\sum Y = 370, \sum X = 6$
(C) $\sum Y = 370, \sum X = -5$
(D) $\sum Y = 370, \sum X = 1$
Answer:
First, let's calculate $\sum Y$ using the given production data:
Years: 2017, 2018, 2019, 2020, 2021, 2022
Production (Y): 50, 55, 62, 60, 68, 75
$\sum Y = 50 + 55 + 62 + 60 + 68 + 75$
$\sum Y = 370$
... (i)
Next, we need to calculate $\sum X$ using the coded values of $X$ provided in option (C): -5, -3, -1, 1, 3, 5.
The coded values of $X$ correspond to the years 2017 to 2022 as per this specific coding scheme (where the interval between consecutive $X$ values is 2). Let's confirm the order:
Year 2017: $X = -5$
Year 2018: $X = -3$
Year 2019: $X = -1$
Year 2020: $X = 1$
Year 2021: $X = 3$
Year 2022: $X = 5$
Now, calculate the sum of these $X$ values:
$\sum X = -5 + (-3) + (-1) + 1 + 3 + 5$
Notice that the positive and negative values cancel each other out:
$\sum X = (-5 + 5) + (-3 + 3) + (-1 + 1)$
$\sum X = 0 + 0 + 0$
... (ii)
$\sum X = 0$
... (iii)
So, we have $\sum Y = 370$ and $\sum X = 0$.
Comparing this with the given options, option (A) matches our results.
The correct option is (A).
Question 54. (Continuing from Question 52) Calculate $\sum XY$ and $\sum X^2$ using the coded values of $X$ from Question 52 (using option (C)).
Year | 2017 | 2018 | 2019 | 2020 | 2021 | 2022 | Total |
---|---|---|---|---|---|---|---|
Production (Y) | 50 | 55 | 62 | 60 | 68 | 75 | 370 |
Coded X | -5 | -3 | -1 | 1 | 3 | 5 | 0 |
XY | -250 | -165 | -62 | 60 | 204 | 375 | 162 |
X$^2$ | 25 | 9 | 1 | 1 | 9 | 25 | 70 |
(A) $\sum XY = 162, \sum X^2 = 70$
(B) $\sum XY = 370, \sum X^2 = 0$
(C) $\sum XY = 0, \sum X^2 = 70$
(D) $\sum XY = 70, \sum X^2 = 162$
Answer:
We are given the following data and coded $X$ values from Question 52, option (C):
Years: 2017, 2018, 2019, 2020, 2021, 2022
Production (Y): 50, 55, 62, 60, 68, 75
Coded X: -5, -3, -1, 1, 3, 5
First, we need to calculate $\sum XY$. This is done by multiplying the $Y$ value for each year by its corresponding $X$ value and then summing these products.
- For 2017: $XY = 50 \times (-5) = -250$
- For 2018: $XY = 55 \times (-3) = -165$
- For 2019: $XY = 62 \times (-1) = -62$
- For 2020: $XY = 60 \times 1 = 60$
- For 2021: $XY = 68 \times 3 = 204$
- For 2022: $XY = 75 \times 5 = 375$
Now, sum these products:
$\sum XY = -250 + (-165) + (-62) + 60 + 204 + 375$
$\sum XY = -477 + 639$
$\sum XY = 162$
... (i)
Next, we need to calculate $\sum X^2$. This is done by squaring each $X$ value and then summing these squares.
- For 2017: $X^2 = (-5)^2 = 25$
- For 2018: $X^2 = (-3)^2 = 9$
- For 2019: $X^2 = (-1)^2 = 1$
- For 2020: $X^2 = (1)^2 = 1$
- For 2021: $X^2 = (3)^2 = 9$
- For 2022: $X^2 = (5)^2 = 25$
Now, sum these squares:
$\sum X^2 = 25 + 9 + 1 + 1 + 9 + 25$
$\sum X^2 = 70$
... (ii)
So, we have calculated $\sum XY = 162$ and $\sum X^2 = 70$. Comparing these results with the given options, option (A) matches.
The correct option is (A).
Question 55. (Continuing from Question 52) Using the normal equations and the calculated sums, find the values of $a$ and $b$. ($n=6$)
(A) $a = 370/6 \approx 61.67, b = 162/70 \approx 2.31$
(B) $a = 162/6 = 27, b = 370/70 \approx 5.29$
(C) $a = 370, b = 162$
(D) $a = 61.67, b = 5.29$
Answer:
We are fitting a linear trend $Y_c = a + bX$. The normal equations for Method of Least Squares are:
1. $\sum Y = na + b \sum X$
2. $\sum XY = a \sum X + b \sum X^2$
From the previous questions, we have:
$n = 6$ (number of years)
$\sum Y = 370$
$\sum X = 0$ (using the coding -5, -3, -1, 1, 3, 5)
$\sum XY = 162$
$\sum X^2 = 70$
Let's use these values in the normal equations.
Substitute $\sum X = 0$ into the first equation:
$\sum Y = na + b(0)$
... (i)
$\sum Y = na$
... (ii)
Now, solve for $a$:
$a = \frac{\sum Y}{n}$
... (iii)
Substitute the values:
$a = \frac{370}{6}$
... (iv)
$a \approx 61.67$
... (v)
Now, substitute the values into the second normal equation:
$\sum XY = a \sum X + b \sum X^2$
... (vi)
Since $\sum X = 0$, the equation simplifies to:
$\sum XY = a(0) + b \sum X^2$
... (vii)
$\sum XY = b \sum X^2$
... (viii)
Now, solve for $b$:
$b = \frac{\sum XY}{\sum X^2}$
... (ix)
Substitute the values:
$b = \frac{162}{70}$
... (x)
$b \approx 2.314$
... (xi)
So, $a \approx 61.67$ and $b \approx 2.31$.
Looking at the options, option (A) provides these values:
$a = 370/6 \approx 61.67$
$b = 162/70 \approx 2.31$
The correct option is (A).
Question 56. (Continuing from Question 52) The trend line equation with origin between 2019 and 2020 and X unit 6 months is approximately:
(A) $Y_c = 61.67 + 2.31 X$
(B) $Y_c = 61.67 + 5.29 X$
(C) $Y_c = 2.31 + 61.67 X$
(D) $Y_c = 5.29 + 61.67 X$
Answer:
From Question 55, we found $a \approx 61.67$ and $b \approx 2.31$.
The general form of the trend line is $Y_c = a + bX$.
Substituting the calculated values:
$Y_c = 61.67 + 2.31X$
This matches option (A). The phrasing "X unit 6 months" means that each unit of $X$ represents a 6-month period. The calculated $b \approx 2.31$ represents the change per year, assuming the coding used in previous steps implied yearly intervals. If $X$ is now in 6-month units, and $b$ is the annual change, the coefficient for $X$ would be $b/2$. However, this is not an option. Thus, we assume the equation uses the calculated $a$ and $b$ directly.
The correct option is (A).
Question 57. The term 'deseasonalizing' a time series refers to the process of removing the effect of which component?
(A) Secular Trend
(B) Cyclical Variation
(C) Seasonal Variation
(D) Irregular Variation
Answer:
The term 'deseasonalizing' a time series refers to the process of removing the effect of Seasonal Variation.
Explanation:
- Deseasonalizing: This is a statistical technique used to remove the seasonal component from a time series. The goal is to reveal the underlying trend and cyclical patterns more clearly by eliminating the predictable, short-term fluctuations that occur regularly within a year.
- Seasonal Variation: This is the component of a time series that represents patterns that repeat over fixed periods within a year (e.g., daily, weekly, monthly, quarterly).
- Secular Trend: This is the long-term movement or direction of the data. Deseasonalizing helps to see the trend more clearly.
- Cyclical Variation: These are longer-term fluctuations (more than a year) related to economic cycles. Deseasonalizing can also help to better observe cyclical patterns.
- Irregular Variation: These are random, unpredictable fluctuations. Deseasonalizing does not remove irregular variations.
Therefore, deseasonalizing specifically targets and removes the seasonal component.
The correct option is (C).
Question 58. Which method of measuring trend involves dividing the data into two equal halves and calculating the average for each half?
(A) Graphical Method
(B) Moving Average Method
(C) Method of Semi-Averages
(D) Method of Least Squares
Answer:
The method of measuring trend that involves dividing the data into two equal halves and calculating the average for each half is the Method of Semi-Averages.
Explanation of the methods:
- Graphical Method: This involves plotting the data on a graph and drawing a trend line by eye, which is subjective.
- Moving Average Method: This method smooths out short-term fluctuations by calculating a series of averages over consecutive periods.
- Method of Semi-Averages: In this method, the time series data is divided into two roughly equal halves. The average of the first half (called the first semi-average) and the average of the second half (called the second semi-average) are calculated. A trend line is then fitted by connecting the points representing these two semi-averages.
- Method of Least Squares: This is a mathematical method that fits a trend line by minimizing the sum of the squares of the deviations of the observed values from the trend line.
The description provided in the question precisely matches the procedure for the Method of Semi-Averages.
The correct option is (C).
Question 59. If the number of years in the data is odd (e.g., 7 years), the middle year is taken as the origin when coding $X$ for the Method of Least Squares. The coded values will be:
(A) ... -2, -1, 0, 1, 2 ...
(B) ... -3, -2, -1, 0, 1, 2, 3 ...
(C) ... -2.5, -1.5, -0.5, 0.5, 1.5, 2.5 ...
(D) 1, 2, 3, 4, 5, 6, 7
Answer:
When the number of years in the data is odd, and the middle year is taken as the origin ($X=0$) for coding the time variable in the Method of Least Squares, the coded values for $X$ will be consecutive integers centered around zero.
For a dataset of 7 years, the middle year is the 4th year. If this 4th year is assigned $X=0$, then:
- The year before the middle year (3rd year) would be $X=-1$.
- The year before that (2nd year) would be $X=-2$.
- The first year would be $X=-3$.
- The year after the middle year (5th year) would be $X=1$.
- The year after that (6th year) would be $X=2$.
- The 7th year would be $X=3$.
So, the coded values for 7 years with the middle year as origin are: -3, -2, -1, 0, 1, 2, 3.
Let's examine the options:
- (A) ... -2, -1, 0, 1, 2 ... : This represents a sequence for 5 years (with the middle year as origin).
- (B) ... -3, -2, -1, 0, 1, 2, 3 ... : This represents a sequence for 7 years, centered at 0, which perfectly matches our deduction.
- (C) ... -2.5, -1.5, -0.5, 0.5, 1.5, 2.5 ... : This type of coding (using halves) is typically used when the origin is placed *between* two years (for an even number of observations), not for an odd number of years with the middle year as the origin.
- (D) 1, 2, 3, 4, 5, 6, 7 : This coding starts from 1 and does not have the origin at 0, which simplifies calculations differently and does not center the data around zero.
Therefore, the correct coded values for 7 years when the middle year is the origin ($X=0$) are -3, -2, -1, 0, 1, 2, 3.
The correct option is (B).
Question 60. If the trend equation for monthly data is $Y_c = 500 + 10X$ (Origin July 2020, X unit 1 month), what is the trend value for September 2020?
(A) $500 + 10(2) = 520$
(B) $500 + 10(-2) = 480$
(C) $500 + 10(0) = 500$
(D) $500 + 10(8) = 580$
Answer:
The trend equation is given as $Y_c = 500 + 10X$, with the origin at July 2020 and the unit of $X$ being 1 month.
This means that for any month, the value of $X$ represents the number of months that have passed since July 2020.
We need to find the trend value for September 2020.
Let's determine the $X$ value for September 2020:
- The origin is July 2020, which corresponds to $X=0$.
- August 2020 is 1 month after July 2020. So, August 2020 corresponds to $X=1$.
- September 2020 is 2 months after July 2020. So, September 2020 corresponds to $X=2$.
Now, substitute $X=2$ into the trend equation:
$Y_c = 500 + 10(2)$
... (i)
Calculate the trend value:
$Y_c = 500 + 20$
... (ii)
$Y_c = 520$
... (iii)
Comparing this result with the given options, option (A) matches our calculation.
The correct option is (A).
Question 61. Assertion (A): The Method of Least Squares always fits a linear trend.
Reason (R): By changing the equation form ($Y_c = a+bX+cX^2$, etc.), the method of least squares can fit non-linear trends as well.
(A) Both A and R are true and R is the correct explanation of A.
(B) Both A and R are true but R is not the correct explanation of A.
(C) A is true but R is false.
(D) A is false but R is true.
Answer:
Let's analyze the Assertion (A) and Reason (R).
Assertion (A): The Method of Least Squares always fits a linear trend.
This statement is False. While the Method of Least Squares is commonly used to fit linear trends (of the form $Y_c = a + bX$), it is a versatile method that can be applied to fit various types of trends, including non-linear ones.
Reason (R): By changing the equation form ($Y_c = a+bX+cX^2$, etc.), the method of least squares can fit non-linear trends as well.
This statement is True. The principle of least squares involves minimizing the sum of squared errors. This principle can be applied to any functional form (linear, quadratic, exponential, etc.) by setting up the appropriate system of normal equations. For example, to fit a parabolic trend $Y_c = a + bX + cX^2$, the method of least squares is used to find the values of $a$, $b$, and $c$ that minimize the sum of squared residuals.
Since Assertion (A) is false and Reason (R) is true, the correct option is (D).
The correct option is (D).
Question 62. In the additive model of time series, the components are represented as $Y = T + S + C + I$. This model is typically used when the magnitude of fluctuations is independent of the level of the trend. Is this statement true or false?
(A) True
(B) False
(C) It depends on the data.
(D) It's only true for seasonal variation.
Answer:
The statement is True.
Explanation:
The additive model of a time series is represented as $Y = T + S + C + I$, where:
- $Y$ is the actual value
- $T$ is the Secular Trend
- $S$ is the Seasonal Component
- $C$ is the Cyclical Component
- $I$ is the Irregular Component
The key characteristic of the additive model is that the magnitudes of the seasonal ($S$), cyclical ($C$), and irregular ($I$) components are assumed to be constant regardless of the level of the trend ($T$). In other words, the variations are additive, meaning that if the trend level increases, the absolute size of the seasonal or cyclical fluctuations does not necessarily increase with it.
This assumption holds true when the fluctuations (seasonal, cyclical, irregular) are relatively stable in absolute terms. For example, if sales increase by 100 units during a festive season, this model assumes that the increase is always 100 units, whether the overall trend is at 1000 units or 5000 units.
If the magnitude of fluctuations *increases* as the trend level increases (e.g., sales increase by 10% during a festive season), then the multiplicative model ($Y = T \times S \times C \times I$) is more appropriate.
Therefore, the statement that the additive model is typically used when the magnitude of fluctuations is independent of the level of the trend is correct.
The correct option is (A).
Question 63. If you calculate a 4-quarter centered moving average, how many original data points are effectively used to calculate the moving average value centered at a specific point in time?
(A) 4
(B) 5
(C) 8
(D) 2
Answer:
When calculating a 4-quarter centered moving average, the moving average value centered at a specific point in time is obtained by averaging four consecutive data points.
For quarterly data, a 4-quarter moving average involves summing up the values for four consecutive quarters and dividing by 4.
To center this average at a specific point in time (e.g., to represent a particular quarter's trend), the process is as follows:
- Take four consecutive data points (e.g., Q1, Q2, Q3, Q4 of a year).
- Calculate their average.
- This average represents a value that is midway between the second and third quarters. For an even number of periods (like 4), the average is often considered to be centered between the two middle periods. However, when specifically asked about the "number of original data points used to calculate the moving average value centered at a specific point in time," it directly refers to the number of terms in the average calculation.
In a 4-period moving average, you are always summing and averaging 4 original data points to get one moving average value.
For example, to get a moving average value associated with a certain point in time, you would take the data for that period, the period before, and the two periods after (or similar combinations that result in an average of 4 points). The direct calculation of a 4-period moving average inherently uses 4 original data points.
The correct option is (A).
Question 64. The value of $\sum X$ for the coded time variable in the Method of Least Squares is made equal to zero by selecting the:
(A) First year as origin
(B) Last year as origin
(C) Middle year or mid-point between two middle years as origin
(D) Year with highest value as origin
Answer:
The value of $\sum X$ for the coded time variable in the Method of Least Squares is made equal to zero by selecting the middle year or mid-point between two middle years as origin.
Explanation:
When fitting a trend line using the Method of Least Squares, coding the time variable $X$ with the origin at the middle year (for an odd number of observations) or at the mid-point between the two middle years (for an even number of observations) is a standard practice. This coding ensures that the positive and negative values of $X$ are symmetrically distributed around zero.
- Odd number of observations: If there are $n$ (odd) years, the middle year is assigned $X=0$. The years before it get negative values (e.g., -1, -2, ...), and the years after it get positive values (e.g., 1, 2, ...). The sum of these symmetrically distributed positive and negative integers (and zero) is always zero ($\sum X = 0$).
- Even number of observations: If there are $n$ (even) years, the origin is placed between the two middle years. The coded values are typically $\pm 0.5, \pm 1.5, \pm 2.5, \dots$. Again, these values are symmetrically distributed around zero, so their sum is zero ($\sum X = 0$).
When $\sum X = 0$, the normal equations for the Method of Least Squares simplify considerably, making the calculation of the coefficients ($a$ and $b$ for a linear trend) easier. Specifically, the first normal equation $\sum Y = na + b\sum X$ becomes $\sum Y = na$ (since $\sum X = 0$), which directly gives $a = (\sum Y)/n$.
Options (A), (B), and (D) do not lead to $\sum X = 0$. For example, if the first year is the origin ($X=0$), all subsequent $X$ values will be positive, resulting in $\sum X > 0$. Similarly, if the last year is the origin or the year with the highest value, $\sum X$ will not be zero.
The correct option is (C).
Question 65. The Method of Semi-Averages is simpler to calculate than the Method of Least Squares but is generally considered:
(A) More accurate
(B) Less accurate
(C) Equally accurate
(D) Only suitable for non-linear trends
Answer:
The Method of Semi-Averages is simpler to calculate than the Method of Least Squares but is generally considered Less accurate.
Explanation:
- Method of Least Squares: This method uses all data points and the principle of minimizing the sum of squared errors. This statistical approach is mathematically robust and provides a unique "best-fitting" line in a least-squares sense, making it generally the most accurate method for fitting a linear trend.
- Method of Semi-Averages: This method is simpler because it only uses the averages of two halves of the data. However, by only using these two averages and discarding the information from all other data points, it can be heavily influenced by outliers or deviations in those halves. It does not consider the variability of the data points within each half, nor does it minimize errors across the entire dataset.
Therefore, while easier, the Method of Semi-Averages is less accurate because it relies on fewer data points and a less sophisticated mathematical principle compared to the Method of Least Squares.
The correct option is (B).
Question 66. If the trend equation is $Y_c = 80 + 3X$, and the original data for a specific period is $Y = 85$, what is the deviation from trend for that period? (Assume the $X$ value for that period resulted in a trend value of 80).
(A) $85 - 80 = 5$
(B) $80 - 85 = -5$
(C) 3
(D) $85/80$
Answer:
The deviation from trend for a specific period is calculated as the difference between the actual value ($Y$) and the trend value ($Y_c$) for that period.
Deviation = $Y - Y_c$
We are given:
- Original data value for a specific period, $Y = 85$.
- The trend value for that same period is given as $Y_c = 80$. (The question states "Assume the $X$ value for that period resulted in a trend value of 80", implying $Y_c = 80$ for this period, even though the trend equation is $Y_c = 80 + 3X$. This phrasing is a bit confusing. If the trend equation is $Y_c = 80+3X$, then $Y_c$ cannot be 80 unless $X=0$. Let's assume the question means that for this specific period, the *calculated* trend value is 80.)
Using the formula for deviation:
Deviation = $Y - Y_c$
Deviation = $85 - 80$
Deviation = $5$
... (i)
This calculation matches option (A).
Let's consider the context of the trend equation $Y_c = 80 + 3X$. If the trend value ($Y_c$) for a specific period is 80, it implies that for that period, $X=0$. If $X=0$, then the deviation calculation is indeed $85 - (80 + 3 \times 0) = 85 - 80 = 5$. The wording of the question supports this interpretation.
The correct option is (A).
Question 67. Case Study: The table shows the average monthly temperature ($^\circ\text{C}$) in a city over 4 quarters of a year.
Quarter | 1 (Jan-Mar) | 2 (Apr-Jun) | 3 (Jul-Sep) | 4 (Oct-Dec) |
---|---|---|---|---|
Avg Temp ($^\circ\text{C}$) | 20 | 35 | 30 | 25 |
Which time series component is most evident in this data?
(A) Secular Trend
(B) Cyclical Variation
(C) Seasonal Variation
(D) Irregular Variation
Answer:
The time series component most evident in this data is Seasonal Variation.
Explanation:
- The data represents average temperatures over 4 quarters of a year.
- We observe a pattern of temperature changes: lower in winter (Q1), rising to a peak in summer (Q2), then decreasing in autumn/winter (Q3, Q4).
- This pattern of increase and decrease that repeats within a 12-month cycle is characteristic of seasonal variation. The temperatures are higher in the warmer months (spring/summer) and lower in the colder months (autumn/winter), a predictable pattern for a given location within a year.
- Secular Trend: This would represent a long-term change in temperature over many years, which cannot be determined from data covering only one year.
- Cyclical Variation: These are longer-term fluctuations (more than a year) related to economic cycles, which are not relevant to temperature changes within a year.
- Irregular Variation: While there might be some day-to-day or week-to-week fluctuations, the overall quarterly averages show a consistent pattern that is not irregular.
The observed pattern of temperature rise and fall over the four quarters of a single year is a classic example of seasonal variation.
The correct option is (C).
Question 68. The process of fitting a trend line to a time series is often referred to as:
(A) Decomposition
(B) Forecasting
(C) Smoothing
(D) Regression analysis (in the context of time as the independent variable)
Answer:
The process of fitting a trend line to a time series is often referred to as Regression analysis (in the context of time as the independent variable).
Explanation:
- Decomposition: This is a broader process of breaking down a time series into its constituent components (trend, seasonal, cyclical, irregular). Fitting a trend line is a part of decomposition, but decomposition itself is not just fitting the trend line.
- Forecasting: This is the process of predicting future values based on past data. While fitting a trend line is often a step in forecasting, it is not the entire process.
- Smoothing: Methods like moving averages are used for smoothing, which aims to reduce random fluctuations and reveal underlying patterns, including the trend. While closely related to fitting a trend, "smoothing" primarily focuses on reducing noise rather than explicitly defining a functional form for the trend.
- Regression analysis: When time ($t$ or $X$) is treated as the independent variable and the time series values ($Y$) are treated as the dependent variable, fitting a line (linear regression) or a curve (non-linear regression) to the data is precisely what regression analysis does. This is the most direct and common description of fitting a trend line. For example, fitting $Y_c = a + bX$ is a linear regression.
While smoothing is a related technique and decomposition involves fitting a trend, regression analysis is the most accurate description of the process of mathematically defining a trend line based on the data where time is the independent variable.
The correct option is (D).
Question 69. If a time series consists only of the trend component, the graph would appear as a:
(A) Jagged line with peaks and troughs
(B) Smooth curve or straight line
(C) Repeating pattern within a year
(D) Series of random points
Answer:
If a time series consists only of the trend component, the graph would appear as a smooth curve or straight line.
Explanation:
- Trend Component: The trend component represents the long-term direction or movement in the data. It describes the general tendency of the data to increase, decrease, or remain relatively stable over extended periods. By its nature, the trend is a smooth, underlying movement that captures the overall pattern without the short-term fluctuations.
- Jagged line with peaks and troughs: This describes a time series that includes cyclical and/or irregular variations.
- Repeating pattern within a year: This describes seasonal variation.
- Series of random points: This describes irregular variation.
Since the trend component is about the long-term, smooth movement, a time series composed solely of the trend would exhibit this smooth, continuous pattern, which can be either a straight line (for a linear trend) or a curve (for a non-linear trend).
The correct option is (B).
Question 70. A major political event causing sudden changes in economic indicators is an example of:
(A) Seasonal Variation
(B) Cyclical Variation
(C) Secular Trend
(D) Irregular Variation
Answer:
A major political event causing sudden changes in economic indicators is an example of Irregular Variation.
Explanation:
- Irregular Variation (or Random Variation): This component of a time series consists of unpredictable, random fluctuations that are not explained by the other components (trend, seasonal, or cyclical). Major unforeseen events, such as significant political changes, natural disasters, or unexpected economic policies, often lead to sudden, erratic movements in data that fit this definition.
- Seasonal Variation: This refers to patterns that repeat at regular intervals within a year (e.g., monthly, quarterly). A political event is not a regular, annual occurrence.
- Cyclical Variation: These are longer-term fluctuations, often tied to economic business cycles, which typically last several years and have a somewhat predictable rhythm. A sudden political event's impact is usually more abrupt and less cyclical.
- Secular Trend: This is the long-term upward or downward movement in data over many years. While a major political event might influence the long-term trend, its immediate, sudden impact is classified as irregular.
The sudden and unpredictable nature of the change caused by a major political event makes it an irregular variation.
The correct option is (D).
Question 71. The trend equation for annual data is $Y_c = 300 + 15X$ (Origin 2005, X unit 1 year). Convert this equation to have the origin at 2010.
(A) $Y_c = 300 + 15(X-5) = 225 + 15X$
(B) $Y_c = 300 + 15(X+5) = 375 + 15X$
(C) $Y_c = (300+15 \times 5) + 15X = 375 + 15X$
(D) Both (B) and (C)
Answer:
The original trend equation is $Y_c = 300 + 15X$, where the origin is 2005 and the unit of $X$ is 1 year. This means for any year $Y$, $X = Y - 2005$.
We want to shift the origin to 2010. Let the new time variable be $X'$. Then, $X' = Y - 2010$.
We need to express the original $X$ in terms of the new $X'$.
From $X' = Y - 2010$, we get $Y = X' + 2010$.
Substitute this into the original $X$ definition:
$X = (X' + 2010) - 2005$
$X = X' + 5$
Now, substitute this expression for $X$ into the original trend equation:
$Y_c = 300 + 15X$
$Y_c = 300 + 15(X' + 5)$
Let's simplify this equation:
$Y_c = 300 + 15X' + 15 \times 5$
$Y_c = 300 + 15X' + 75$
$Y_c = 375 + 15X'$
This is the new trend equation with the origin shifted to 2010, where $X'$ is the new time variable.
Now let's look at the options:
- (A) $Y_c = 300 + 15(X-5) = 225 + 15X$. This would be for shifting the origin forward by 5 years (e.g., from 2005 to 2010, if X was based on 2005, then X-5 would be for origin 2010). Let's check the relationship between variables: If original X is Y-2005, and new X' is Y-2010, then X = X'+5. Option A uses X-5. If the new variable is called X, then $Y_c = 300 + 15(X_{new}-5)$. This is incorrect, as $X_{original} = X_{new}+5$.
- (B) $Y_c = 300 + 15(X+5) = 375 + 15X$. Here, $X$ is treated as the new variable. So, $Y_c = 300 + 15(X_{new}+5) = 300 + 15X_{new} + 75 = 375 + 15X_{new}$. This matches our derived equation, assuming $X$ in the option refers to the new time variable.
- (C) $Y_c = (300+15 \times 5) + 15X = 375 + 15X$. This is simply an algebraic rearrangement of the correct equation, showing that the new intercept is the original intercept plus the shift in the origin times the slope. This also matches our derivation, assuming $X$ in the option refers to the new time variable.
- (D) Both (B) and (C).
Both options (B) and (C) represent the correct conversion and algebraic simplification. Option (B) shows the substitution and then the simplified form, while option (C) shows the calculation of the new intercept and then the simplified form. Both are valid ways of expressing the converted equation.
The correct option is (D).
Question 72. If the trend equation for annual data is $Y_c = 500 + 20X$ (Origin 2015, X unit 1 year), convert this to have the origin at July 1, 2015, and X unit 6 months.
(A) $Y_c = 500 + 20(X/2) = 500 + 10X$
(B) $Y_c = 500 + 20(2X) = 500 + 40X$
(C) $Y_c = (500 + 20 \times 0) + 20X$
(D) $Y_c = 500 + 20X$ (equation remains the same)
Answer:
Original equation: $Y_c = 500 + 20X$, where $X$ is in years from Jan 1, 2015.
New requirements: Origin at July 1, 2015. New $X$ ($X_{new}$) in 6-month units.
1. Unit Conversion: If original $X$ is in years and new $X_{new}$ is in 6-month units, then $X = X_{new}/2$ (since 1 year = 2 six-month periods). Or, $X_{old} = 0.5 X_{new}$.
2. Origin Shift: The new origin (July 1, 2015) is 0.5 years from the old origin (Jan 1, 2015). The original intercept (500) applies to Jan 1, 2015. The value at the new origin (July 1, 2015) in the old equation would be $500 + 20(0.5) = 510$. So, the new intercept should be 510.
3. New Slope: The original slope is 20 per year. The new unit is 0.5 years. So, the new slope is $20 \times 0.5 = 10$ per 6-month period.
The correct transformed equation should be $Y_c = 510 + 10X_{new}$.
None of the options precisely match this derived equation. However, Option (A) $Y_c = 500 + 20(X/2) = 500 + 10X$ correctly changes the slope from 20 (per year) to 10 (per 6-month period) by replacing $X$ with $X/2$ (implicitly meaning $X_{old} = X_{new}/2$ for unit conversion). It retains the original intercept of 500, ignoring the origin shift's impact on the intercept.
Given the options, Option (A) is the most plausible choice if the question prioritizes unit conversion over the origin shift in the intercept calculation.
The correct option is (A).
Question 73. The primary objective of decomposing a time series is to:
(A) Combine all components into a single value.
(B) Isolate and understand the individual components that influence the series.
(C) Eliminate all variations from the data.
(D) Only identify the irregular component.
Answer:
The primary objective of decomposing a time series is to isolate and understand the individual components that influence the series.
Explanation:
- Time Series Decomposition: This is a statistical method that breaks down a time series into several key components:
- Trend (T): The long-term direction of the series.
- Seasonal (S): The regular, periodic patterns that repeat within a year.
- Cyclical (C): Fluctuations that occur over longer periods (more than a year), often related to business cycles.
- Irregular/Random (I): Unpredictable, short-term fluctuations.
- By separating these components, analysts can gain a better understanding of the underlying patterns driving the time series. This understanding helps in explaining past behavior, forecasting future values, and making informed decisions.
- Option (A) is incorrect because decomposition separates components, it doesn't combine them into a single value.
- Option (C) is incorrect; while smoothing might reduce variations, decomposition aims to understand them, not eliminate all of them.
- Option (D) is incorrect; while the irregular component is identified, it's only one part of the decomposition, and the primary goal is a holistic understanding of all components.
The correct option is (B).
Question 74. In the Method of Semi-Averages, if the number of years is odd, the middle year is:
(A) Included in the first half.
(B) Included in the second half.
(C) Excluded from both halves.
(D) Divided equally between the two halves.
Answer:
In the Method of Semi-Averages, if the number of years in the data is odd, the middle year is typically excluded from both halves to ensure that the two halves are as equal as possible in size.
Explanation:
Let's consider an example with 7 years of data:
Years: $Y_1, Y_2, Y_3, Y_4, Y_5, Y_6, Y_7$
The middle year is $Y_4$.
To divide the data into two equal halves, we take the first $(n-1)/2$ years for the first half and the last $(n-1)/2$ years for the second half, where $n$ is the total number of years.
For $n=7$, $(n-1)/2 = (7-1)/2 = 3$.
- First half: $Y_1, Y_2, Y_3$
- Middle year (excluded): $Y_4$
- Second half: $Y_5, Y_6, Y_7$
Averages are then calculated for the first half and the second half. The line is fitted through these two averages.
If the middle year were included in either half, the halves would not be of equal size, which would bias the calculation. For instance, including $Y_4$ in the first half would give 4 years in the first half and 3 in the second, or vice-versa.
Option (D) "Divided equally between the two halves" is incorrect because a single year cannot be divided in this context for averaging purposes to create two equal halves of data points.
The correct option is (C).
Question 75. Which characteristic of a time series is least likely to be predictable?
(A) Trend
(B) Seasonality
(C) Cyclicality
(D) Randomness
Answer:
The characteristic of a time series that is least likely to be predictable is Randomness.
Explanation:
- Trend: Represents the long-term direction of the data. It is generally predictable, as it describes a sustained upward, downward, or stable movement over time. Methods like linear regression or moving averages can project the trend into the future.
- Seasonality: Refers to patterns that repeat over fixed periods within a year (e.g., daily, weekly, monthly, quarterly). These patterns are predictable because they occur at regular intervals.
- Cyclicality: Describes fluctuations that occur over periods longer than a year, often related to business cycles. While the timing and amplitude of cycles can vary, their general occurrence and direction are often predictable to some extent, especially in economic contexts.
- Randomness (Irregular Variation): This component consists of unpredictable, random fluctuations that are not explained by the other components. Major political events, natural disasters, or unforeseen individual occurrences fall into this category. By definition, these variations are random and therefore the least predictable.
Predictability decreases in the order: Seasonality > Trend > Cyclicality > Randomness.
The correct option is (D).
Question 76. If the trend equation is $Y_c = a + bX$, the value of $b$ represents the average change in $Y$ per unit change in $X$. This value is also known as the:
(A) Intercept
(B) Slope
(C) Mean
(D) Standard deviation
Answer:
In the trend equation $Y_c = a + bX$, the value of $b$ represents the average change in $Y$ per unit change in $X$. This value is also known as the Slope.
Explanation:
- In the equation of a straight line, $y = mx + c$, where $m$ is the slope and $c$ is the y-intercept.
- In the context of trend analysis, $Y_c = a + bX$, the term $b$ is analogous to $m$ and represents the slope of the trend line. The slope indicates how much $Y_c$ changes for a one-unit increase in $X$.
- The term $a$ is the y-intercept, which represents the value of $Y_c$ when $X=0$.
- Mean and standard deviation are statistical measures related to the distribution of data but are not directly represented by $b$ in the trend equation.
The correct option is (B).
Question 77. The Method of Moving Averages is particularly effective in smoothing out:
(A) Long-term trend
(B) Cycles of long duration
(C) Short-term random fluctuations and seasonal variations
(D) Only linear trends
Answer:
The Method of Moving Averages is particularly effective in smoothing out Short-term random fluctuations and seasonal variations.
Explanation:
- Moving Average Method: This technique involves calculating a series of averages for different subsets of the data. For instance, a 3-period moving average averages three consecutive data points.
- Smoothing Effect: By averaging consecutive data points, the method effectively dampens or "smooths out" the impact of short-term fluctuations, including random variations and predictable seasonal patterns. When the window of the moving average (e.g., 4 quarters for seasonal data) covers a full cycle, the seasonal effects within that window tend to cancel out, revealing the underlying trend and cyclical components.
- Long-term Trend and Cycles: While moving averages can help in identifying the trend and cycles, their primary effectiveness lies in their ability to smooth out the more erratic short-term noise. They don't inherently predict or quantify the long-term trend or long cycles as effectively as other methods like regression analysis or specialized cycle analysis techniques.
- Linear Trends: Moving averages can be used to estimate linear trends, but their effectiveness in smoothing is not limited to only linear trends; they smooth all types of short-term variations.
Therefore, the main strength of the moving average method is its ability to reduce the impact of noise, making underlying patterns more visible.
The correct option is (C).
Question 78. Which of the following represents a Price Relative of 150?
(A) Current price is 150% of the base price.
(B) Current price is 50% higher than the base price.
(C) Current price is 1.5 times the base price.
(D) All of the above.
Answer:
A Price Relative is calculated as:
Price Relative = $\frac{\text{Current Price}}{\text{Base Price}} \times 100$
If the Price Relative is 150, it means:
$150 = \frac{\text{Current Price}}{\text{Base Price}} \times 100$
Dividing both sides by 100:
$1.50 = \frac{\text{Current Price}}{\text{Base Price}}$
This implies:
- Current Price = 1.50 $\times$ Base Price
Let's analyze the given options based on this relationship:
(A) Current price is 150% of the base price.
This means Current Price = 150% $\times$ Base Price = $1.50 \times$ Base Price. This is true.
(B) Current price is 50% higher than the base price.
This means Current Price = Base Price + 50% of Base Price = Base Price + $0.50 \times$ Base Price = $1.50 \times$ Base Price. This is true.
(C) Current price is 1.5 times the base price.
This means Current Price = $1.5 \times$ Base Price. This is true.
Since all three statements (A), (B), and (C) correctly represent the meaning of a Price Relative of 150, option (D) is the correct answer.
The correct option is (D).
Question 79. If the Method of Least Squares linear trend equation is used for forecasting, the forecast will assume that:
(A) Only trend will continue in the future.
(B) All components (Trend, Seasonal, Cyclical, Irregular) will continue with their past patterns.
(C) Only seasonal variation will continue.
(D) There will be no changes in the future.
Answer:
If the Method of Least Squares linear trend equation is used for forecasting, the forecast will assume that only the trend will continue in the future.
Explanation:
- Method of Least Squares Linear Trend: This method specifically models and projects the long-term, smooth movement (the trend) of the data. The equation $Y_c = a + bX$ represents this linear trend.
- Forecasting with Trend: When this trend line is used for forecasting, it assumes that the underlying linear trend observed in the past will continue into the future.
- Exclusion of Other Components: A simple linear trend model does not explicitly account for or project seasonal, cyclical, or irregular variations. It assumes these other components are either negligible or will not persist in their past patterns in a way that can be projected. Therefore, the forecast represents an extrapolation of the trend only.
- Option (B) is incorrect because a simple linear trend model does not forecast seasonality, cyclicality, or randomness.
- Option (C) is incorrect because a linear trend model does not specifically forecast seasonal variation.
- Option (D) is incorrect; while it assumes the trend continues, it doesn't assume *no* changes in other components, but rather that those other components are not part of the forecast. The forecast is based on the *continuation* of the trend, not a cessation of all change.
The correct option is (A).
Question 80. The primary objective of fitting a trend line to a time series is to:
(A) Eliminate seasonal and irregular variations.
(B) Predict the exact future values.
(C) Estimate the systematic long-term movement.
(D) Identify short-term fluctuations.
Answer:
The primary objective of fitting a trend line to a time series is to estimate the systematic long-term movement.
Explanation:
- Trend Line: A trend line represents the underlying direction or movement in a time series over a long period. It smooths out short-term fluctuations like seasonal, cyclical, and irregular variations to reveal this long-term pattern.
- Objective of Fitting: By fitting a trend line, we aim to quantify and understand this systematic long-term change (whether it's increasing, decreasing, or stable). This understanding is crucial for various analyses, including forecasting, comparison, and understanding underlying growth or decline patterns.
- Option (A) is incorrect; fitting a trend line helps in identifying it, but its primary goal isn't to eliminate other variations, although it's a step in doing so for decomposition.
- Option (B) is incorrect; while trend estimation is a step towards forecasting, a trend line alone does not predict exact future values as it ignores other components.
- Option (D) is incorrect; fitting a trend line aims to capture the long-term movement, not the short-term fluctuations.
The correct option is (C).
Question 81. A period of prosperity followed by recession, depression, and recovery in economic activity is characteristic of:
(A) Secular Trend
(B) Seasonal Variation
(C) Cyclical Variation
(D) Irregular Variation
Answer:
A period of prosperity followed by recession, depression, and recovery in economic activity is characteristic of Cyclical Variation.
Explanation:
- Cyclical Variation: This component of a time series refers to fluctuations that occur over periods longer than one year, typically related to economic business cycles. These cycles involve phases of expansion (prosperity), contraction (recession/depression), and recovery. The duration of these cycles is not fixed and can vary.
- Secular Trend: This is the long-term, underlying movement of the data, which may be upward, downward, or stable over many years. It does not describe the fluctuations within those years.
- Seasonal Variation: This refers to patterns that repeat within a fixed period, usually a year (e.g., sales peaking during holidays). The economic cycle described is much longer and less predictable in its timing than seasonal patterns.
- Irregular Variation: These are random, unpredictable fluctuations. While specific events can cause sudden changes, the described pattern of prosperity-recession-depression-recovery is a systematic, recurring cycle, not random noise.
The description of economic activity phases directly aligns with the definition of cyclical variation.
The correct option is (C).
Question 82. When coding the time variable $X$ for the Method of Least Squares, if the time unit is 6 months and the origin is a specific year, the coded value for a year half a year before the origin year would typically be:
(A) -0.5
(B) -1
(C) -2
(D) 0.5
Answer:
If the time unit is 6 months, and the origin is a specific year (let's assume the start of the origin year, e.g., Jan 1 of the origin year), then:
- The origin year itself (starting point) corresponds to $X=0$.
- A period half a year *before* the origin year means we are at the start of the year that precedes the origin year.
- Since the unit is 6 months, half a year is equivalent to $0.5 \text{ years} / 0.5 \text{ years/unit} = 1$ unit of $X$.
Therefore, a period half a year before the origin year would be 1 unit of $X$ before the origin.
If the origin is $X=0$, then 1 unit before the origin is $X = 0 - 1 = -1$.
Let's clarify with an example:
Suppose the origin year is 2020. The origin $X=0$ corresponds to Jan 1, 2020.
The time unit is 6 months.
- Jan 1, 2020 is $X=0$.
- July 1, 2020 is 6 months after Jan 1, 2020, so $X=1$.
- Jan 1, 2020 is 6 months after Jan 1, 2019, so Jan 1, 2019 is $X=-1$.
The question asks for "a year half a year before the origin year". This phrasing can be interpreted in two ways:
- Half a year before the *start* of the origin year: If the origin year is 2020, and the origin is Jan 1, 2020 ($X=0$), then half a year before this is July 1, 2019. Since the unit is 6 months, July 1, 2019 is 6 months before Jan 1, 2020, which is 1 unit of $X$ before $X=0$. So, $X = -1$.
- The year that starts half a year before the origin year: This interpretation is less likely given the standard way time variables are coded.
Assuming the first interpretation, where we are looking at a point in time half a year prior to the origin point in time:
Origin Year (e.g., 2020), Origin Point: Jan 1, 2020 ($X=0$)
Time point: July 1, 2019 (half a year before Jan 1, 2020)
Unit: 6 months (0.5 years)
Number of units from origin = (July 1, 2019 - Jan 1, 2020) / 0.5 years
= (-0.5 years) / 0.5 years = -1 unit of $X$.
Let's check the options.
If the time unit is 6 months, then half a year is exactly 1 unit of $X$. If the origin is $X=0$, then half a year *before* the origin is $X=-1$.
The correct option is (B).
Question 83. If a 4-year moving average is calculated, the first moving average value will be centered between which two years?
(A) Between year 1 and year 2
(B) Between year 2 and year 3
(C) At year 2
(D) At year 3
Answer:
If a 4-year moving average is calculated, the first moving average value will be centered between year 2 and year 3.
Explanation:
Let the annual data be $Y_1, Y_2, Y_3, Y_4, Y_5, \dots$.
The first 4-year moving average is calculated using the first four data points: $Y_1, Y_2, Y_3, Y_4$.
The average is $\frac{Y_1 + Y_2 + Y_3 + Y_4}{4}$.
For an even number of periods (like 4), the moving average value is considered to be centered between the two middle periods. In this case, the two middle periods are $Y_2$ and $Y_3$.
Therefore, the first moving average value is centered between year 2 and year 3.
The correct option is (B).
Question 84. The method of fitting a trend line by eye is called the:
(A) Moving Average Method
(B) Method of Semi-Averages
(C) Graphical Method
(D) Method of Least Squares
Answer:
The method of fitting a trend line by eye is called the Graphical Method.
Explanation:
- Graphical Method: In this method, the time series data is plotted on a graph, and a trend line is drawn by visual inspection, aiming to represent the general tendency of the data. It is subjective and depends on the user's judgment.
- Moving Average Method: This involves calculating averages of subsets of data to smooth out fluctuations and reveal the trend. It's a computational method.
- Method of Semi-Averages: This method involves averaging two halves of the data to determine the trend line. It relies on calculations, not fitting by eye.
- Method of Least Squares: This is a mathematical method that minimizes the sum of squared errors to find the best-fitting line. It is highly objective and computational.
Therefore, the method described as fitting by eye is the Graphical Method.
The correct option is (C).
Question 85. The most unpredictable component of a time series is:
(A) Trend
(B) Seasonal
(C) Cyclical
(D) Irregular
Answer:
The most unpredictable component of a time series is the Irregular component.
Explanation:
- Trend: This represents the long-term direction and is generally predictable, showing a sustained increase or decrease.
- Seasonal: This component involves patterns that repeat at regular intervals within a year (e.g., monthly, quarterly). These are highly predictable due to their cyclical nature.
- Cyclical: This refers to fluctuations that occur over longer periods, often related to economic cycles. While their exact timing and magnitude can vary, they exhibit a pattern that is more predictable than random fluctuations.
- Irregular (or Random): This component represents unpredictable, random variations caused by unforeseen events, unique occurrences, or errors. By definition, it is the least predictable component of a time series.
The correct option is (D).
Question 86. If the trend equation is $Y_c = 400 - 5X$ (Origin 2018, X unit 1 year), what is the trend value for the year 2025?
(A) $400 - 5(7) = 400 - 35 = 365$
(B) $400 - 5(2025 - 2018) = 400 - 5(7) = 365$
(C) $400 - 5(2025) = 400 - 10125 = -9725$
(D) Both (A) and (B)
Answer:
The trend equation is given as $Y_c = 400 - 5X$.
The origin is the year 2018, and the unit of $X$ is 1 year. This means $X$ represents the number of years passed since 2018.
We need to find the trend value for the year 2025.
To find the value of $X$ for the year 2025, we calculate the difference between the target year and the origin year:
$X = \text{Target Year} - \text{Origin Year}$
$X = 2025 - 2018$
$X = 7$
... (i)
Now, substitute the value of $X=7$ into the trend equation:
$Y_c = 400 - 5X$
$Y_c = 400 - 5(7)$
Calculate the trend value:
$Y_c = 400 - 35$
$Y_c = 365$
... (ii)
Let's examine the options:
- (A) $400 - 5(7) = 400 - 35 = 365$: This correctly shows the substitution and the final result.
- (B) $400 - 5(2025 - 2018) = 400 - 5(7) = 365$: This option explicitly shows the calculation of $X$ and then the substitution, which is also correct.
- (C) $400 - 5(2025) = 400 - 10125 = -9725$: This incorrectly uses the year 2025 directly as $X$.
- (D) Both (A) and (B).
Since both options (A) and (B) correctly demonstrate the calculation and arrive at the correct trend value, option (D) is the most appropriate answer.
The correct option is (D).
Short Answer Type Questions
Question 1. Define a Time Series. Give two examples of time series data collected in India.
Answer:
Definition of a Time Series
A time series is a sequence of data points or observations collected at successive points in time. These observations are ordered chronologically, usually at equally spaced time intervals (e.g., hourly, daily, weekly, monthly, annually). Mathematically, a time series is a set of observations $y_t$, where the subscript $t$ represents the time at which the observation was made.
The primary objective of analyzing a time series is to understand its underlying structure, identify patterns (like trends, seasonality, and cycles), and use this understanding to forecast future values.
Two Examples of Time Series Data Collected in India
1. Annual Population of India: The Census of India provides population data at regular intervals. The sequence of population figures recorded over the years (e.g., population in 1981, 1991, 2001, 2011, and subsequent estimates) forms a time series. This data is fundamental for government planning, resource allocation, and policy-making.
2. Monthly Consumer Price Index (CPI): The National Statistical Office (NSO) of India releases data on the Consumer Price Index on a monthly basis. The CPI measures the average change over time in the prices paid by urban consumers for a market basket of consumer goods and services. The sequence of monthly CPI values (e.g., CPI for Jan 2022, Feb 2022, Mar 2022, ...) is a classic example of a time series used to measure inflation and guide monetary policy.
Question 2. What are the four main components of a time series?
Answer:
A time series is traditionally decomposed into four main components. These components represent the underlying patterns in the data and are used to analyze its behavior and make forecasts. The four components are:
1. Secular Trend or Trend ($T_t$):
This refers to the long-term, general tendency of the data to increase, decrease, or remain stable over a significant period. It shows the smooth, underlying movement of the series, ignoring short-term fluctuations. For example, the trend in India's GDP over the last few decades has been generally upward.
2. Seasonal Variation ($S_t$):
These are periodic, repetitive, and predictable fluctuations that occur within a year. The period of variation is less than or equal to one year (e.g., quarterly, monthly, weekly). They are often caused by factors like weather, climate, and social customs or festivals. For example, the sale of air conditioners typically peaks during the summer months each year.
3. Cyclical Variation ($C_t$):
These are oscillatory movements in a time series with a period of more than one year. They are long-term swings around the trend line or curve. These cycles are not strictly periodic and can vary in length and amplitude. They are often associated with business or economic cycles, such as periods of prosperity, recession, depression, and recovery.
4. Irregular or Random Variation ($I_t$):
This component represents the erratic, unpredictable, and random fluctuations in the time series. These variations are not accounted for by trend, seasonal, or cyclical movements. They are caused by unforeseen and non-recurring events such as strikes, wars, floods, earthquakes, or policy changes. They are purely random in nature.
These four components are combined to model the observed value ($Y_t$) of the time series. There are two primary models for this combination:
a) Additive Model: This model is used when the magnitude of the seasonal and cyclical fluctuations does not vary with the level of the series. It assumes the components are independent of each other.
$Y_t = T_t + S_t + C_t + I_t$
(Additive Model) ... (i)
b) Multiplicative Model: This model is more common in practice and is used when the magnitude of the seasonal and cyclical fluctuations varies with the level of the trend. It assumes the components are dependent on each other.
$Y_t = T_t \times S_t \times C_t \times I_t$
(Multiplicative Model) ... (ii)
Question 3. Explain the concept of Secular Trend in a time series. Give an example.
Answer:
Concept of Secular Trend in a Time Series
The Secular Trend, often referred to simply as the trend, is the fundamental component of a time series that represents its smooth, long-term, general direction or tendency. It describes whether the data, over a significant period (usually several years), is generally increasing, decreasing, or remaining stable. The term "secular" originates from the Latin word saeculum, which means a long period or an age, highlighting the long-term nature of this component.
The trend component, denoted by $T_t$, captures the underlying movement of the series by filtering out the short-term fluctuations caused by seasonal, cyclical, and irregular variations. It reflects the influence of persistent and fundamental forces that gradually affect the data over time. These forces can include:
- Changes in population (growth or decline).
- Technological advancements and innovation.
- Shifts in consumer tastes, preferences, and social habits.
- Gradual economic development and changes in government policies.
- Capital formation and infrastructure development.
Identifying and analyzing the secular trend is crucial for understanding the historical growth or decline of a phenomenon and for making long-range forecasts.
Example of Secular Trend
A classic example of a secular trend is the literacy rate in India since its independence in 1947.
If we plot the literacy rate recorded in each decennial census (1951, 1961, 1971, and so on), we will observe some fluctuations from one decade to the next. However, the overall, long-term movement of the data points shows a clear and consistent upward trend.
The key characteristics of this example are:
- Long-Term Movement: The analysis spans over 70 years, which is a sufficiently long period to establish a trend.
- General Direction: The literacy rate has consistently increased from 18.33% in 1951 to 74.04% in 2011 and continues to rise.
- Underlying Forces: This upward trend is not accidental. It is driven by persistent, long-term factors such as sustained government programs and policies promoting education (like the Sarva Shiksha Abhiyan), increased public awareness about the importance of education, and overall socio-economic development.
By studying this secular trend, policymakers can evaluate the effectiveness of past policies and make informed plans and forecasts for achieving universal literacy in the future.
Question 4. Define Seasonal Variation. Provide an example related to economic activity in India.
Answer:
Definition of Seasonal Variation
Seasonal Variation, denoted as $S_t$, is a component of a time series that represents periodic, repetitive, and predictable fluctuations that occur within a period of one year or less. These variations are generally regular in their timing, direction, and amplitude and complete their full cycle within a 12-month period (e.g., quarterly, monthly, or weekly data).
The primary causes of seasonal variation are:
- Natural Factors: These are related to the seasons of the year, climate, and weather conditions. For example, the demand for woolen clothes increases in winter and decreases in summer.
- Man-made Conventions: These are related to customs, traditions, and habits of people. For instance, sales volumes tend to increase significantly during festival seasons like Diwali or Christmas.
Analyzing seasonal variation is essential for short-term planning, helping businesses to manage inventory, staffing, and marketing campaigns effectively.
Example Related to Economic Activity in India
A prominent example of seasonal variation in India's economic activity is the sale of consumer goods during the festival season.
Specifically, the sales of automobiles, electronics (like smartphones and televisions), apparel, and gold show a marked and predictable increase during the third and fourth quarters of the calendar year. This period typically encompasses major Indian festivals such as Dussehra, Dhanteras, and Diwali.
- Pattern: There is a sharp spike in consumer spending and retail sales from approximately October to December each year.
- Cause: This surge is driven by cultural and social conventions. Consumers consider it an auspicious time to make significant purchases, and many working individuals receive annual bonuses during this period. Companies also fuel this demand with festival-specific discounts, offers, and extensive marketing campaigns.
- Periodicity: This pattern of a sales peak in the festival quarter, followed by a relative lull in the subsequent quarter, repeats reliably every year.
This seasonal pattern is closely monitored by economists, retailers, and manufacturers for inventory planning, sales forecasting, and assessing the overall health of consumer sentiment in the economy.
Question 5. What is meant by Cyclical Variation? How is it different from Seasonal Variation?
Answer:
Cyclical Variation
Cyclical variation refers to the long-term, oscillatory movements or swings that occur in a time series. These variations are often associated with the phases of a business cycle, such as prosperity, recession, depression, and recovery. They typically extend over periods longer than one year and do not necessarily follow a fixed or regular pattern in terms of duration or amplitude. The causes of cyclical variations are complex and can include economic factors like demand-supply gaps, changes in government policies, technological shifts, and investor confidence.
Seasonal Variation
Seasonal variation refers to the fluctuations in a time series that occur at regular intervals, usually within a calendar year. These variations repeat year after year with a predictable pattern. They are typically caused by factors that are directly or indirectly related to the seasons, climate, calendar events (like holidays), or social customs. Examples include increased sales of woollens in winter, higher tourism during specific months, or peak production in agriculture during harvesting seasons.
Difference between Cyclical Variation and Seasonal Variation
The main differences between cyclical and seasonal variations lie in their duration, regularity, and causes:
1. Duration: Cyclical variations are long-term fluctuations, typically lasting for more than a year, often ranging from 2 to 10 years or more. Seasonal variations are short-term fluctuations that complete within a year and repeat annually.
2. Regularity: Seasonal variations are regular and periodic, repeating almost identically each year. Their pattern is predictable. Cyclical variations are less regular; their duration and amplitude can vary significantly from one cycle to another, making them less predictable than seasonal variations.
3. Cause: Seasonal variations are caused by seasonal factors, climate, calendar, or social customs. Cyclical variations are generally caused by economic factors related to business cycles, demand, supply, technological changes, etc.
4. Nature of Pattern: Seasonal patterns are typically a fixed wave-like pattern recurring annually. Cyclical patterns are also wave-like (ups and downs) but are irregular in their length and intensity.
Question 6. Explain Irregular Variation in a time series. Give an example of an event that would cause irregular variation.
Answer:
Irregular Variation (Random Variation)
Irregular variation, also known as random variation or erratic variation, refers to the fluctuations in a time series that are unpredictable, erratic, and do not follow any discernible pattern. These variations are caused by unforeseen and random events that are usually of short duration and have a sudden impact on the time series. They are essentially residual fluctuations after accounting for trend, seasonal, and cyclical components.
Irregular variations are typically caused by unique, one-off events that are external to the normal working of the system or process being measured. They are considered random because their occurrence and impact cannot be predicted in advance using historical data or known patterns.
Example of an event that would cause irregular variation:
A sudden natural disaster, such as an earthquake or a flood, occurring in a major production area. This event would disrupt production, supply chains, and demand, causing an unpredictable and sudden change in sales figures or production output for that period, which would be an irregular variation in the time series data for sales or production.
Other examples could include a major political event (like a sudden policy change or civil unrest), a large-scale strike in a key industry, a terrorist attack, or a sudden severe weather event like a blizzard affecting retail sales significantly for a short period.
Question 7. What is the primary objective of analyzing a time series?
Answer:
The primary objective of analyzing a time series is to understand the underlying patterns, components, and behavior of the data over time in order to make informed decisions and predictions about future values.
This typically involves identifying and separating the different components of the time series, such as trend, seasonality, cyclical variations, and irregular fluctuations. By understanding these components, analysts can:
- Describe past behavior
- Explain the causes of variations
- Predict future values (forecasting)
- Control or adjust the process based on the patterns
Therefore, the core aim is to gain insights into the temporal dynamics of the data for purposes like forecasting, planning, and decision-making.
Question 8. Name two methods used for measuring the Secular Trend in a time series.
Answer:
Two common methods used for measuring the Secular Trend in a time series are:
1. Method of Least Squares: This is a mathematical method that fits a straight line or a curve to the time series data in such a way that the sum of the squares of the vertical deviations of the actual data points from the fitted line/curve is minimized. It provides an objective measure of the trend.
2. Method of Moving Averages: This method calculates the average of the data points over a fixed period (e.g., 3-year moving average, 5-year moving average). These averages are then plotted to smooth out short-term fluctuations (seasonal, cyclical, irregular) and reveal the underlying trend. The length of the moving average period is crucial and is often chosen to eliminate seasonal or cyclical variations.
Question 9. What is a Moving Average? How does it help in smoothing a time series?
Answer:
What is a Moving Average?
A Moving Average is a statistical method used in time series analysis to calculate the average of a data set over a specified period, called the "window" or "period" of the moving average. This window moves through the time series data, with each average being calculated for a consecutive subset of the data.
For example, a 3-period moving average at a particular time point would be the average of the data values at that time point, the previous time point, and the time point before that.
How does it help in smoothing a time series?
The primary purpose of using a Moving Average is to smooth out the short-term fluctuations or noise in a time series, such as seasonal variations, irregular variations, and minor cyclical variations. By averaging the data points over a period, the effect of extreme values or random spikes is reduced.
When a Moving Average is plotted, the resulting line tends to be less jagged and more representative of the underlying trend and longer-term movements in the data, rather than the temporary ups and downs. The longer the period of the moving average, the smoother the resulting line will be, as more data points are included in each average calculation, thus dampening the effect of individual fluctuations more significantly.
Effectively, it isolates the longer-term components (like trend and major cycles) by averaging away the shorter-term components (like seasonality and irregularity).
Question 10. Calculate the 3-year moving averages for the following data: 20, 25, 22, 28, 24, 30.
Answer:
The given data points are: 20, 25, 22, 28, 24, 30.
We need to calculate the 3-year moving averages. A 3-year moving average is the average of three consecutive data points. The average is typically placed at the center of the period.
Calculations:
1. Moving Average for the period covering the 1st, 2nd, and 3rd data points (centered at 2nd data point):
$\frac{20 + 25 + 22}{3} = \frac{67}{3}$
$\approx 22.33$
2. Moving Average for the period covering the 2nd, 3rd, and 4th data points (centered at 3rd data point):
$\frac{25 + 22 + 28}{3} = \frac{75}{3}$
$= 25$
3. Moving Average for the period covering the 3rd, 4th, and 5th data points (centered at 4th data point):
$\frac{22 + 28 + 24}{3} = \frac{74}{3}$
$\approx 24.67$
4. Moving Average for the period covering the 4th, 5th, and 6th data points (centered at 5th data point):
$\frac{28 + 24 + 30}{3} = \frac{82}{3}$
$\approx 27.33$
Summary Table:
Year/Period | Data Value | 3-Year Moving Total | 3-Year Moving Average (Centered) |
1 | 20 | - | - |
2 | 25 | 67 (20+25+22) | $\frac{67}{3} \approx 22.33$ |
3 | 22 | 75 (25+22+28) | $\frac{75}{3} = 25.00$ |
4 | 28 | 74 (22+28+24) | $\frac{74}{3} \approx 24.67$ |
5 | 24 | 82 (28+24+30) | $\frac{82}{3} \approx 27.33$ |
6 | 30 | - | - |
The 3-year moving averages for the given data are approximately 22.33, 25.00, 24.67, and 27.33, centered at the 2nd, 3rd, 4th, and 5th data points respectively.
Question 11. What is the purpose of 'centering' in the Method of Moving Averages when the period is even?
Answer:
When the period of a moving average is an even number (e.g., 2, 4, 6, 12), the calculated moving average value does not align perfectly with any specific time point in the original series. Instead, it is centered between two time points.
The purpose of 'centering' in the Method of Moving Averages when the period is even is to align the calculated average value with the original time periods or points. This is typically done by calculating a 2-period moving average of the initial moving average results.
For example, if you calculate a 4-period moving average, the first average is for periods 1-4, centered between period 2 and 3. The second average is for periods 2-5, centered between period 3 and 4. By taking a 2-period average of these two results, the final value is centered at period 3, aligning it with an actual time point in the original data series.
This centering is important because:
- It makes the smoothed series directly comparable with the original series at corresponding time points.
- It is necessary for subsequent steps in time series decomposition, such as calculating seasonal indices, which require deviations of the original data from the trend (represented by the centered moving average) at each specific time point.
Question 12. Explain the fundamental principle of the Method of Least Squares as applied to fitting a trend line.
Answer:
The fundamental principle of the Method of Least Squares, when applied to fitting a trend line to a time series, is to determine the line (or curve) that best represents the overall direction of the data by minimizing the discrepancies between the actual observed data points and the values predicted by the fitted line.
More specifically, the principle states that the "best-fitting" line is the one for which the sum of the squares of the vertical deviations (or errors) of the actual data points from the fitted line is the minimum.
Let $y_i$ be the actual observed value at time period $i$, and let $\hat{y}_i$ be the value predicted by the fitted trend line at the same time period $i$. The deviation or error for the $i$-th data point is the difference between the observed value and the predicted value, i.e., $e_i = y_i - \hat{y}_i$.
The Method of Least Squares does not minimize the sum of the errors ($\sum e_i$) because positive and negative errors would cancel each other out. Instead, it minimizes the sum of the squares of these errors ($\sum e_i^2$):
Minimize $\sum_{i=1}^{n} (y_i - \hat{y}_i)^2$
where $n$ is the number of data points.
For a linear trend line, the predicted value $\hat{y}_i$ is typically given by the equation $\hat{y}_i = a + b x_i$, where $x_i$ represents the time period (often coded as 1, 2, 3, ... or centered around zero) and $a$ and $b$ are the parameters of the line (intercept and slope) that need to be determined. The method involves finding the values of $a$ and $b$ that satisfy the minimum condition for the sum of squared errors.
By minimizing the sum of squares, the method ensures that the fitted line is as close as possible to all data points overall, giving more weight to larger deviations, thus providing a statistically objective way to fit a trend line to the data.
Question 13. If the trend line equation obtained by the Method of Least Squares is $Y_t = 150 + 5t$, where $t$ represents the year number (with $t=1$ for the first year), what is the estimated trend value for the 6th year?
Answer:
Given:
The trend line equation obtained by the Method of Least Squares is $Y_t = 150 + 5t$, where $t$ represents the year number and $t=1$ corresponds to the first year.
To Find:
The estimated trend value for the 6th year.
Solution:
The trend equation is given by:
${Y_t} = 150 + 5t$
We need to find the estimated trend value for the 6th year. This corresponds to $t=6$.
Substitute $t=6$ into the equation:
${Y_6} = 150 + 5(6)$
Calculate the value:
${Y_6} = 150 + 30$
${Y_6} = 180$
Answer:
The estimated trend value for the 6th year is 180.
Question 14. In the trend line equation $Y_t = a + bt$, what does the coefficient 'b' represent? What does its sign (positive or negative) indicate?
Answer:
In the linear trend line equation $Y_t = a + bt$, where $Y_t$ is the estimated trend value at time period $t$, and $a$ and $b$ are coefficients:
The coefficient 'b' represents the slope of the trend line. It indicates the average rate of change in the time series per unit of time.
In simpler terms, 'b' tells us by how much the trend value is expected to increase or decrease on average with each passing time period (e.g., per year, per quarter, per month), assuming a linear relationship.
The sign of the coefficient 'b' indicates the direction of the trend:
- If $b$ is positive ($b > 0$), it indicates an upward trend. This means that the time series values are increasing on average over time. The trend line slopes upwards.
- If $b$ is negative ($b < 0$), it indicates a downward trend. This means that the time series values are decreasing on average over time. The trend line slopes downwards.
- If $b$ is close to zero ($b \approx 0$), it suggests that there is little or no significant linear trend in the time series.
Therefore, 'b' quantifies the magnitude and direction of the linear trend component in the time series.
Question 15. State the normal equations for fitting a linear trend line $Y_t = a + bt$ using the Method of Least Squares.
Answer:
The Method of Least Squares aims to find the values of the coefficients $a$ and $b$ in the linear trend equation $Y_t = a + bt$ that minimize the sum of the squared differences between the observed values ($y_t$) and the estimated trend values ($Y_t$).
This minimization process leads to a system of two linear equations with two unknowns ($a$ and $b$), known as the Normal Equations.
Let $n$ be the number of data points (time periods). The normal equations are:
Normal Equation 1:
$\sum y = na + b \sum t$
Normal Equation 2:
$\sum ty = a \sum t + b \sum t^2$
Where:
- $\sum y$ is the sum of the actual observed values of the time series.
- $n$ is the number of time periods (observations).
- $\sum t$ is the sum of the time period indices (e.g., if $t=1, 2, ..., n$).
- $\sum ty$ is the sum of the product of the time period index and the observed value for each period.
- $\sum t^2$ is the sum of the squares of the time period indices.
- $a$ and $b$ are the coefficients of the trend line to be determined by solving these equations.
These two equations can be solved simultaneously to find the unique values of $a$ and $b$ that define the linear trend line that best fits the data in the least-squares sense.
Question 16. What is the main disadvantage of using the Method of Moving Averages compared to the Method of Least Squares?
Answer:
The main disadvantage of using the Method of Moving Averages compared to the Method of Least Squares is its inability to provide a mathematical equation for the trend line.
Because the Moving Average method only produces a smoothed series of values, it does not yield a function ($Y_t = a + bt$ or similar) that explicitly describes the relationship between the time variable ($t$) and the trend value ($Y_t$).
This lack of an explicit equation makes it difficult or impossible to forecast or extrapolate the trend value into future time periods directly using the moving average method alone. The Method of Least Squares, on the other hand, provides the equation $Y_t = a + bt$, which can be easily used to calculate the estimated trend value for any future time period $t$ by simply substituting the value of $t$ into the equation.
Question 17. How can a trend line fitted by the Method of Least Squares be used for forecasting?
Answer:
A trend line fitted by the Method of Least Squares provides a mathematical model for the long-term movement in a time series. This model, typically a linear equation $Y_t = a + bt$ (where $Y_t$ is the estimated trend value and $t$ is the time period), quantifies the relationship between time and the variable being measured.
To use this trend line for forecasting, you simply need to substitute the value of the future time period for which you want to forecast into the fitted trend equation.
For example, if the fitted linear trend equation is:
${Y_t} = a + bt$
and you want to forecast the trend value for a future time period, say $t_{future}$, you would calculate:
Estimated Trend Value at $t_{future} = a + b \times t_{future}$
By calculating the value of the dependent variable ($Y_t$) for future values of the independent variable ($t$), you are projecting the established historical trend forward in time. This provides an estimate of the expected value of the time series variable in the future, assuming the underlying trend continues.
It is important to note that this method forecasts only the trend component. For a complete forecast, especially in time series with significant seasonality or cyclicality, the forecasts for these other components would also need to be calculated and combined with the trend forecast.
Question 18. Give an example of how Seasonal Variation might affect the sales data of an ice cream vendor in India.
Answer:
Seasonal variation would significantly affect the sales data of an ice cream vendor in India due to the distinct climate patterns throughout the year.
Here's an example:
During the hot summer months (typically March to June), the demand for ice cream is very high. People seek cold refreshments to cope with the heat, leading to a surge in sales. This period would represent the peak in the ice cream sales time series for the year.
As the monsoon season arrives (typically July to September), sales might see a decline compared to summer, as the weather becomes cooler and wetter, and some people might avoid cold items. This would represent a relative dip in sales.
During the winter months (typically November to February, especially in northern and central India), the weather is cold, and the demand for ice cream plummets significantly. Sales would be at their lowest point during this period.
This pattern of high sales in summer, moderate sales during transitional periods, and low sales in winter is a predictable annual cycle. This consistent, within-year fluctuation, driven by the seasonal climate, is a clear example of seasonal variation impacting the ice cream vendor's sales data.
Question 19. What is univariate time series data?
Answer:
Univariate Time Series Data
Univariate time series data refers to a sequence of observations or measurements recorded for a single variable at successive points or intervals in time. The data consists of values of just one characteristic, item, or measure, collected sequentially over a period.
In essence, it's data that has two main attributes: a measurement and a time stamp. The focus is solely on how this single variable changes over time.
Example:
Monthly sales figures of a single product recorded over several years would be univariate time series data. Here, the single variable is "sales figures", and the data is recorded at successive time points (each month).
Other examples include daily closing stock prices of a specific company, annual rainfall in a particular region, or hourly temperature readings at one location.
This is contrasted with multivariate time series data, which involves recording measurements for multiple variables simultaneously over time (e.g., monthly sales figures for multiple products recorded at the same time points).
Question 20. Describe the graphical method of estimating trend. What is its main limitation?
Answer:
Graphical Method of Estimating Trend
The graphical method is one of the simplest ways to estimate the trend component of a time series. The process involves the following steps:
1. Plotting the Data: The time series data is plotted on a graph paper or using graphing software. Time is plotted on the x-axis (horizontal axis), and the observed values of the variable are plotted on the y-axis (vertical axis).
2. Drawing the Trend Line: A freehand, straight line is drawn through the plotted points. The goal is to draw a line that visually appears to pass through the middle of the points, capturing the general upward or downward movement of the series, while smoothing out the short-term fluctuations (seasonal, cyclical, irregular). The line should ideally have approximately an equal number of points above and below it.
The line drawn represents the estimated trend line. The values on this line at different time points are considered the estimated trend values.
Main Limitation:
The main limitation of the graphical method is its subjectivity. Since the trend line is drawn by freehand, different individuals analyzing the same data may draw slightly different lines, leading to varying estimates of the trend. There is no objective criterion or mathematical formula to ensure that the "best" possible line is drawn. This lack of objectivity makes the results less reliable and non-reproducible compared to mathematical methods like the Method of Least Squares or Moving Averages.
Question 21. Calculate the 4-year moving total for the first few periods of the following data: 100, 110, 105, 112, 108, 115.
Answer:
The given data points are: 100, 110, 105, 112, 108, 115.
We need to calculate the 4-year moving totals for the first few periods.
Calculations:
1. First 4-year moving total: Sum of the first 4 data points (100, 110, 105, 112)
Sum = $100 + 110 + 105 + 112$
Sum = $427$
This total covers periods 1 to 4.
2. Second 4-year moving total: Sum of the next 4 consecutive data points (110, 105, 112, 108)
Sum = $110 + 105 + 112 + 108$
Sum = $435$
This total covers periods 2 to 5.
3. Third 4-year moving total: Sum of the next 4 consecutive data points (105, 112, 108, 115)
Sum = $105 + 112 + 108 + 115$
Sum = $440$
This total covers periods 3 to 6.
Summary Table:
Year/Period | Data Value | 4-Year Moving Total |
1 | 100 | - |
2 | 110 | 427 (100+110+105+112) |
3 | 105 | 435 (110+105+112+108) |
4 | 112 | 440 (105+112+108+115) |
5 | 108 | - |
6 | 115 | - |
The 4-year moving totals for the first few periods are 427, 435, and 440.
Question 22. If the trend line for annual production (in tonnes) is $Y_t = 500 + 10t$, where $t=0$ in 2015, what is the estimated production in 2020?
Answer:
Given:
The trend line equation for annual production is $Y_t = 500 + 10t$.
The time index $t=0$ corresponds to the year 2015.
$Y_t$ represents the estimated production in tonnes.
To Find:
The estimated production in the year 2020.
Solution:
First, we need to determine the value of $t$ that corresponds to the year 2020.
The base year is 2015, where $t=0$.
The number of years from the base year to 2020 is $2020 - 2015 = 5$ years.
So, the year 2020 corresponds to $t=5$.
Now, substitute $t=5$ into the given trend line equation:
${Y_t} = 500 + 10t$
${Y_5} = 500 + 10(5)$
Calculate the value:
${Y_5} = 500 + 50$
${Y_5} = 550$
Answer:
The estimated production in the year 2020 is 550 tonnes.
Question 23. What type of time series component would typically be removed by using a moving average with a period equal to the length of the cycle?
Answer:
When a moving average is used with a period equal to the length of the seasonal cycle (e.g., 12 for monthly data, 4 for quarterly data), the time series component that is typically removed or smoothed out is the Seasonal Variation.
This happens because each moving average calculation includes exactly one complete cycle of the seasonal pattern. By averaging over the full seasonal period, the high and low values within the season tend to cancel each other out, effectively isolating the non-seasonal components (Trend and Cyclical) and smoothing out the Irregular fluctuations as well.
While a moving average also smooths out irregular variations, the specific design of using a period equal to the 'cycle' length is primarily aimed at neutralizing the effect of the recurring, fixed-period fluctuations, which are the seasonal variations.
Question 24. Give one advantage of the Method of Moving Averages.
Answer:
One advantage of the Method of Moving Averages is its simplicity and ease of calculation and understanding.
Compared to mathematical methods like the Method of Least Squares, calculating a moving average is straightforward. It only requires summing values over a defined period and dividing by the number of periods. This makes it accessible and easy to implement, even without advanced statistical software, and the concept of averaging to smooth out variations is intuitive.
Additionally, it is very effective at visually smoothing the time series, making the underlying trend and major cycles more apparent by removing short-term noise (seasonal and irregular fluctuations).
Question 25. What are residuals in the context of fitting a trend line using Least Squares?
Answer:
In the context of fitting a trend line using the Method of Least Squares, residuals (also known as errors or deviations) are the differences between the actual observed values of the time series and the corresponding values predicted by the fitted trend line at each time point.
For a time series with observed values $y_1, y_2, ..., y_n$ and a fitted trend line equation providing estimated trend values $\hat{y}_1, \hat{y}_2, ..., \hat{y}_n$, the residual for the $i$-th time period is calculated as:
${e_i} = y_i - \hat{y}_i$
Where:
- $y_i$ is the actual observed value at time $i$.
- $\hat{y}_i$ is the value on the fitted trend line at time $i$.
- $e_i$ is the residual at time $i$.
These residuals represent the part of the variation in the time series data that is *not* explained by the linear trend. In time series decomposition models, these residuals, after the trend has been accounted for, contain the combined effects of seasonal variation, cyclical variation, and irregular variation.
The Method of Least Squares specifically works by minimizing the sum of the squares of these residuals ($\sum e_i^2$) to find the best-fitting trend line.
Question 26. The quarterly sales (in $\textsf{₹}$ lakhs) of a product for two years are: Q1 10, Q2 12, Q3 15, Q4 8, Q1 11, Q2 13, Q3 16, Q4 9. Calculate the 4-quarter centered moving average for the available quarters.
Answer:
Given Data:
Year | Quarter | Sales ($\textsf{₹}$ lakhs) | Time Index ($t$) |
1 | Q1 | 10 | 1 |
1 | Q2 | 12 | 2 |
1 | Q3 | 15 | 3 |
1 | Q4 | 8 | 4 |
2 | Q1 | 11 | 5 |
2 | Q2 | 13 | 6 |
2 | Q3 | 16 | 7 |
2 | Q4 | 9 | 8 |
We need to calculate the 4-quarter centered moving average.
Step 1: Calculate 4-Quarter Moving Totals
Sum of every 4 consecutive quarters.
- Total 1 (t=1 to 4): $10 + 12 + 15 + 8 = 45$ (Centered between t=2 and t=3)
- Total 2 (t=2 to 5): $12 + 15 + 8 + 11 = 46$ (Centered between t=3 and t=4)
- Total 3 (t=3 to 6): $15 + 8 + 11 + 13 = 47$ (Centered between t=4 and t=5)
- Total 4 (t=4 to 7): $8 + 11 + 13 + 16 = 48$ (Centered between t=5 and t=6)
- Total 5 (t=5 to 8): $11 + 13 + 16 + 9 = 49$ (Centered between t=6 and t=7)
Step 2: Calculate 4-Quarter Moving Averages
Divide each moving total by 4.
- Average 1 (centered between t=2 & 3): $\frac{45}{4} = 11.25$
- Average 2 (centered between t=3 & 4): $\frac{46}{4} = 11.50$
- Average 3 (centered between t=4 & 5): $\frac{47}{4} = 11.75$
- Average 4 (centered between t=5 & 6): $\frac{48}{4} = 12.00$
- Average 5 (centered between t=6 & 7): $\frac{49}{4} = 12.25$
These averages are centered between time periods.
Step 3: Calculate 2-Period Moving Average of 4-Quarter Moving Averages (Centered Moving Average)
Take a 2-period average of the results from Step 2. This centers the average on the actual time periods.
- Centered MA 1 (centered at t=3, 1Y Q3): $\frac{11.25 + 11.50}{2} = \frac{22.75}{2} = 11.375$
- Centered MA 2 (centered at t=4, 1Y Q4): $\frac{11.50 + 11.75}{2} = \frac{23.25}{2} = 11.625$
- Centered MA 3 (centered at t=5, 2Y Q1): $\frac{11.75 + 12.00}{2} = \frac{23.75}{2} = 11.875$
- Centered MA 4 (centered at t=6, 2Y Q2): $\frac{12.00 + 12.25}{2} = \frac{24.25}{2} = 12.125$
Summary Table:
Year | Quarter | Sales (y) | 4-Qtr Moving Total | 4-Qtr Moving Average | 4-Qtr Centered Moving Average |
1 | Q1 | 10 | - | - | - |
1 | Q2 | 12 | 45 | 11.25 | - |
1 | Q3 | 15 | 46 | 11.50 | $\frac{11.25+11.50}{2} = 11.375$ |
1 | Q4 | 8 | 47 | 11.75 | $\frac{11.50+11.75}{2} = 11.625$ |
2 | Q1 | 11 | 48 | 12.00 | $\frac{11.75+12.00}{2} = 11.875$ |
2 | Q2 | 13 | 49 | 12.25 | $\frac{12.00+12.25}{2} = 12.125$ |
2 | Q3 | 16 | - | - | - |
2 | Q4 | 9 | - | - | - |
The 4-quarter centered moving averages are 11.375, 11.625, 11.875, and 12.125 for 1Y Q3, 1Y Q4, 2Y Q1, and 2Y Q2, respectively.
Question 27. If the trend line equation is $Y_t = 200 - 3t$, where $t=1$ for Year 2010, what does this indicate about the trend of the data?
Answer:
The given trend line equation is $Y_t = 200 - 3t$, where $t=1$ represents the Year 2010.
In the linear trend equation $Y_t = a + bt$, the coefficient 'b' represents the slope of the trend line and indicates the average rate of change per unit of time.
In this equation, $a = 200$ and $b = -3$.
What this indicates about the trend of the data is:
1. Direction of Trend: Since the coefficient 'b' is negative ($b = -3$), it indicates a downward trend in the data over time.
2. Rate of Change: The value of 'b' being 3 means that the trend value of the time series is decreasing by an average of 3 units per time period (which is one year, as $t$ represents the year number).
So, the trend indicates that the variable represented by $Y_t$ is, on average, declining by 3 units each year starting from the base period (represented by $t=1$ in 2010).
Question 28. Why is it important to analyze the components of a time series separately?
Answer:
Analyzing the components of a time series (Trend, Seasonal, Cyclical, and Irregular) separately is important for several key reasons:
1. Understanding Underlying Factors: Each component represents different types of influences acting on the time series. Trend reflects long-term fundamental changes, seasonality reflects regular, repeating patterns (often calendar or climate related), cyclical variation reflects business cycle type oscillations, and irregular variation reflects random or unusual events. Separating them helps in understanding the specific drivers behind the observed fluctuations.
2. Improved Forecasting Accuracy: Different components exhibit different characteristics and require different forecasting techniques. By analyzing and modeling each component separately, more appropriate methods can be applied to each, leading to more accurate forecasts when the components are recombined.
3. Policy Formulation and Decision Making: Identifying the strength and pattern of each component provides valuable insights for planning and decision-making. For example, understanding seasonal patterns is crucial for inventory management and staffing. Recognizing a cyclical downturn can inform strategic responses to economic conditions. Identifying the underlying trend helps in long-term planning.
4. Isolation of Irregularity: By removing the predictable components (Trend, Seasonal, Cyclical), the remaining irregular component is isolated. This helps in identifying and investigating unusual or unexpected events that caused significant deviations from the expected pattern.
In summary, separate analysis of time series components allows for a deeper understanding of the data's behavior, facilitates better modeling and forecasting, and provides actionable insights for management and policy decisions.
Question 29. Calculate the 5-year moving average for the following production data (in tonnes): 500, 520, 510, 530, 525, 540, 535, 550.
Answer:
Given Data:
The production data (in tonnes) are: 500, 520, 510, 530, 525, 540, 535, 550.
We need to calculate the 5-year moving averages. A 5-year moving average is the average of five consecutive data points, centered at the middle year of the period.
Calculations:
1. Moving Average for the period covering the 1st to 5th data points (centered at the 3rd data point):
$\frac{500 + 520 + 510 + 530 + 525}{5} = \frac{2585}{5}$
$= 517$ tonnes
2. Moving Average for the period covering the 2nd to 6th data points (centered at the 4th data point):
$\frac{520 + 510 + 530 + 525 + 540}{5} = \frac{2625}{5}$
$= 525$ tonnes
3. Moving Average for the period covering the 3rd to 7th data points (centered at the 5th data point):
$\frac{510 + 530 + 525 + 540 + 535}{5} = \frac{2640}{5}$
$= 528$ tonnes
4. Moving Average for the period covering the 4th to 8th data points (centered at the 6th data point):
$\frac{530 + 525 + 540 + 535 + 550}{5} = \frac{2680}{5}$
$= 536$ tonnes
Summary Table:
Year/Period | Production (tonnes) | 5-Year Moving Total | 5-Year Moving Average (Centered) |
1 | 500 | - | - |
2 | 520 | - | - |
3 | 510 | 2585 (500+...+525) | 517 |
4 | 530 | 2625 (520+...+540) | 525 |
5 | 525 | 2640 (510+...+535) | 528 |
6 | 540 | 2680 (530+...+550) | 536 |
7 | 535 | - | - |
8 | 550 | - | - |
The 5-year moving averages for the given data are 517, 525, 528, and 536 tonnes, centered at the 3rd, 4th, 5th, and 6th data points respectively.
Question 30. The annual profits of a company (in $\textsf{₹}$ crores) for 5 years are: 10, 12, 15, 11, 14. Calculate the sum of squares of deviations from the linear trend line $Y_t = a + bt$ after fitting it using Least Squares (just set up the expression, no need to calculate $a, b$).
Answer:
In the Method of Least Squares, the sum of squares of deviations (or errors) is the quantity that is minimized to find the best-fitting trend line. The deviation for each data point is the difference between the actual observed value ($y_t$) and the estimated trend value ($Y_t$) from the fitted line at time $t$.
The formula for the sum of squares of deviations (SSD) for a linear trend line $Y_t = a + bt$ is given by:
$\text{SSD} = \sum_{t=1}^{n} (y_t - Y_t)^2 = \sum_{t=1}^{n} (y_t - (a + bt))^2$
where:
- $y_t$ is the actual profit in year $t$.
- $Y_t = a + bt$ is the estimated trend profit in year $t$.
- $a$ and $b$ are the coefficients of the trend line.
- $n$ is the number of years (data points).
Given Data:
The annual profits for 5 years are: 10, 12, 15, 11, 14.
Here, $n=5$. Let's assign the time index $t=1$ for the first year, $t=2$ for the second year, and so on.
- For $t=1$, $y_1 = 10$
- For $t=2$, $y_2 = 12$
- For $t=3$, $y_3 = 15$
- For $t=4$, $y_4 = 11$
- For $t=5$, $y_5 = 14$
Expression for the Sum of Squares of Deviations:
Substituting the given $y_t$ values and the trend equation $Y_t = a + bt$ into the SSD formula, we get:
$\text{SSD} = (10 - (a + b \times 1))^2 + (12 - (a + b \times 2))^2 + (15 - (a + b \times 3))^2 + (11 - (a + b \times 4))^2 + (14 - (a + b \times 5))^2$
This can be written as:
$\text{SSD} = (10 - a - b)^2 + (12 - a - 2b)^2 + (15 - a - 3b)^2 + (11 - a - 4b)^2 + (14 - a - 5b)^2$
This is the expression for the sum of squares of deviations from the linear trend line $Y_t = a + bt$ for the given data, which is minimized by the Method of Least Squares to find the values of $a$ and $b$.
Question 31. What is forecasting in the context of time series analysis?
Answer:
In the context of time series analysis, forecasting refers to the process of predicting future values of a time series variable based on the analysis of its historical data and the patterns identified within it.
Time series analysis methods are used to understand the behavior of the data over time by decomposing it into its underlying components (trend, seasonality, cyclical, and irregular). Forecasting then involves projecting these identified patterns, particularly the trend, seasonality, and sometimes cyclical components, forward into future time periods.
The primary objective of time series forecasting is to provide estimates of what the values of the variable are likely to be at specified future points or intervals in time. These forecasts are crucial for planning, decision-making, resource allocation, and strategic management in various fields like business, economics, finance, meteorology, and operations.
Question 32. How can graphical representation help in identifying the components of a time series?
Answer:
Graphical representation is a fundamental and often the first step in analyzing a time series. By plotting the data over time, it provides a visual overview that can help in identifying the presence and nature of the various time series components:
1. Trend: A plot of the time series can visually reveal a long-term upward or downward movement in the data. If the points generally slope upwards over the entire period, an upward trend is likely present. If they slope downwards, a downward trend is indicated. A relatively flat movement suggests little or no significant linear trend.
2. Seasonal Variation: If there are regular, repeating patterns within each year (or another fixed period), these stand out clearly on a graph, especially if the data is plotted with appropriate time intervals (e.g., monthly data plotted over several years). Peaks and troughs occurring consistently in the same months or quarters each year are visual evidence of seasonality.
3. Cyclical Variation: Longer-term swings or oscillations that span multiple years and do not have a fixed period are often visible on the graph. These cycles are typically smoother and broader than seasonal patterns. The graph helps in observing the approximate duration and amplitude of these upswings and downswings.
4. Irregular Variation: After observing the overall trend, seasonality, and major cycles, the remaining jaggedness or random spikes and dips in the plot often represent the irregular or random fluctuations. Unusual, sudden shifts in the data that don't fit the other patterns can be visually identified as potential irregular events.
In summary, graphical representation serves as an initial exploratory tool that allows analysts to visually inspect the data, get a sense of the patterns present, and make preliminary judgments about which components are significant before applying more formal statistical methods for decomposition and analysis.
Question 33. Which component of a time series is generally caused by long-term economic cycles?
Answer:
The component of a time series that is generally caused by long-term economic cycles (such as prosperity, recession, depression, and recovery, commonly referred to as the business cycle) is the Cyclical Variation.
Cyclical variations represent oscillatory movements in the data that occur over periods longer than one year and are often tied to macroeconomic fluctuations. These cycles do not necessarily follow a fixed pattern in terms of their length or amplitude, distinguishing them from the more regular seasonal variations.
Question 34. What is the main objective of fitting a trend line to a time series data?
Answer:
The main objective of fitting a trend line to a time series data is to identify, quantify, and represent the long-term underlying movement or direction of the series over time.
Essentially, fitting a trend line helps in:
- Capturing the Secular Trend: Isolating the consistent upward or downward sweep of the data, ignoring short-term fluctuations like seasonality and cycles.
- Understanding the Rate of Change: Quantifying the average increase or decrease in the variable per unit of time (especially with methods like Least Squares, where the slope 'b' represents this).
- Smoothing the Data: Providing a smoothed representation of the series that highlights the fundamental pattern.
- Basis for Decomposition: Serving as a foundation for removing the trend component from the data to analyze other components (seasonal, cyclical, irregular).
- Forecasting: Providing a mathematical or visual basis for projecting the long-term movement into the future, which is a key element in forecasting future values of the time series.
In summary, it's about extracting the fundamental, sustained movement of the time series from the more volatile short-term or medium-term patterns.
Question 35. If the Method of Least Squares is used to fit a linear trend, what is the sum of the deviations of the actual values from the trend line values?
Answer:
When a linear trend line $Y_t = a + bt$ is fitted to a time series using the Method of Least Squares, the sum of the deviations of the actual observed values ($y_t$) from the corresponding estimated trend values ($Y_t$) is always zero.
The deviation (or residual) at time $t$ is given by $e_t = y_t - Y_t = y_t - (a + bt)$.
The sum of the deviations is $\sum_{t=1}^{n} (y_t - Y_t)$.
The normal equations derived from minimizing the sum of squared deviations ($\sum (y_t - Y_t)^2$) are:
$\sum y_t = na + b \sum t$
$\sum t y_t = a \sum t + b \sum t^2$
(where $n$ is the number of data points)
Consider the first normal equation:
$\sum y_t = na + b \sum t$
Rearranging this equation, we get:
$\sum y_t - na - b \sum t = 0$
The sum of the deviations is:
$\sum (y_t - Y_t) = \sum (y_t - (a + bt)) = \sum y_t - \sum a - \sum bt$
Since $\sum a = na$ (summing constant $a$ $n$ times) and $\sum bt = b \sum t$ (constant $b$ factored out of the sum), the sum of deviations becomes:
$\sum (y_t - Y_t) = \sum y_t - na - b \sum t$
From the first normal equation, we know that $\sum y_t - na - b \sum t = 0$.
Therefore, the sum of the deviations of the actual values from the trend line values is always zero when the trend line is fitted using the Method of Least Squares.
Question 36. A city's monthly temperature data shows a peak in May-June and a low in December-January every year. Which component of time series is most evident here?
Answer:
The pattern described in the city's monthly temperature data, which shows a consistent peak in May-June and a low in December-January **every year**, is a classic example of a regular, repeating fluctuation within a fixed period (one year).
This type of variation is directly related to the seasons and calendar months and repeats with a predictable pattern annually.
Therefore, the component of time series that is most evident here is the Seasonal Variation.
Question 37. Give one disadvantage of the Method of Least Squares for measuring trend.
Answer:
One significant disadvantage of the Method of Least Squares for measuring trend, particularly when using a straight line, is that it is highly sensitive to extreme values or outliers.
Since the method minimizes the sum of the *squares* of the deviations, a large deviation (from an outlier) contributes much more significantly to the sum than smaller deviations. This causes the fitted line to be disproportionately pulled towards the outlier, potentially distorting the representation of the underlying trend for the majority of the data points.
While less of an issue when the data strictly follows a linear path or when using curved trends, for real-world time series with occasional abnormal observations, outliers can significantly affect the estimated slope and intercept of the linear trend line fitted by this method.
Question 38. Why is the length of the moving average period important?
Answer:
The length (or period) of the moving average is crucial because it directly determines the degree of smoothing applied to the time series and which components of the series are emphasized or removed.
Here's why the length is important:
1. Degree of Smoothing: A longer moving average period results in a smoother series, as each value is the average of more data points, which helps in dampening the effect of short-term fluctuations (like seasonal and irregular components) more effectively. A shorter period results in less smoothing.
2. Eliminating Specific Components: The choice of period is particularly important when trying to remove seasonal variation. If the period of the moving average is equal to the length of the seasonal cycle (e.g., 4 for quarterly data, 12 for monthly data), the seasonal fluctuations are largely averaged out, leaving behind the trend and cyclical components.
3. Revealing Underlying Patterns: By smoothing out shorter-term variations, the moving average helps to reveal the underlying trend and longer-term cyclical movements. The appropriate period length allows the analyst to focus on the desired underlying pattern.
4. Data Loss at Ends: A longer moving average period leads to the loss of more data points at the beginning and end of the time series for which a moving average cannot be calculated. This reduces the length of the smoothed series available for analysis or forecasting.
Choosing the wrong period length can either fail to adequately smooth the data (if too short) or over-smooth it, potentially distorting the underlying trend or cyclical patterns and losing valuable data points (if too long). Therefore, the period is selected based on the characteristics of the time series and the specific analytical objective (e.g., smoothing out seasonality, identifying the general trend).
Question 39. How does the Method of Moving Averages handle irregular variations?
Answer:
The Method of Moving Averages handles irregular variations by smoothing them out.
Irregular variations are random, unpredictable fluctuations in the time series. When a moving average is calculated, each value in the smoothed series is the average of several consecutive data points from the original series. By averaging these points, the impact of any single, extreme irregular value (a random high spike or low dip) is reduced because it is averaged together with the surrounding, less extreme values.
Essentially, the averaging process distributes the effect of the random shock over the period of the moving average, thereby reducing the magnitude of the fluctuations caused by irregular factors. The longer the period of the moving average, the more effectively it tends to smooth out the irregular variations.
The goal is that, after applying a moving average (especially one long enough to cover seasonality and cycles), the smoothed series primarily reflects the underlying trend and major cyclical movements, with the irregular component significantly minimized.
Question 40. If the annual profits (in $\textsf{₹}$ lakhs) for 7 years are 25, 28, 26, 30, 29, 32, 31, calculate the 3-year moving averages.
Answer:
Given Data:
The annual profits (in $\textsf{₹}$ lakhs) for 7 years are: 25, 28, 26, 30, 29, 32, 31.
We need to calculate the 3-year moving averages. A 3-year moving average is the average of three consecutive data points, centered at the middle year of the period.
Calculations:
Let the data points be $y_1, y_2, ..., y_7$. The 3-year moving average centered at year $t$ is $\frac{y_{t-1} + y_t + y_{t+1}}{3}$.
1. Moving Average centered at Year 2 (covering Years 1, 2, 3):
$\frac{25 + 28 + 26}{3} = \frac{79}{3}$
$\approx 26.33$ $\textsf{₹}$ lakhs
2. Moving Average centered at Year 3 (covering Years 2, 3, 4):
$\frac{28 + 26 + 30}{3} = \frac{84}{3}$
$= 28.00$ $\textsf{₹}$ lakhs
3. Moving Average centered at Year 4 (covering Years 3, 4, 5):
$\frac{26 + 30 + 29}{3} = \frac{85}{3}$
$\approx 28.33$ $\textsf{₹}$ lakhs
4. Moving Average centered at Year 5 (covering Years 4, 5, 6):
$\frac{30 + 29 + 32}{3} = \frac{91}{3}$
$\approx 30.33$ $\textsf{₹}$ lakhs
5. Moving Average centered at Year 6 (covering Years 5, 6, 7):
$\frac{29 + 32 + 31}{3} = \frac{92}{3}$
$\approx 30.67$ $\textsf{₹}$ lakhs
Summary Table:
Year | Profits ($\textsf{₹}$ lakhs) | 3-Year Moving Total | 3-Year Moving Average (Centered) ($\textsf{₹}$ lakhs) |
1 | 25 | - | - |
2 | 28 | 79 (25+28+26) | $\frac{79}{3} \approx 26.33$ |
3 | 26 | 84 (28+26+30) | $\frac{84}{3} = 28.00$ |
4 | 30 | 85 (26+30+29) | $\frac{85}{3} \approx 28.33$ |
5 | 29 | 91 (30+29+32) | $\frac{91}{3} \approx 30.33$ |
6 | 32 | 92 (29+32+31) | $\frac{92}{3} \approx 30.67$ |
7 | 31 | - | - |
The 3-year moving averages for the given data are approximately 26.33, 28.00, 28.33, 30.33, and 30.67 $\textsf{₹}$ lakhs, centered at Years 2, 3, 4, 5, and 6 respectively.
Question 41. When fitting a linear trend $Y_t = a + bt$ using Least Squares, what assumption is made about the relationship between the variable and time?
Answer:
When fitting a linear trend line $Y_t = a + bt$ to a time series using the Method of Least Squares, the primary assumption made about the relationship between the time series variable ($Y_t$) and time ($t$) is that the trend is linear.
This means it is assumed that the time series variable is, on average, either increasing or decreasing by a constant amount per unit of time. The relationship between the variable and time is modeled as a straight line.
The coefficient 'b' represents this constant rate of change (the slope), and 'a' represents the estimated value of the variable when the time index is zero (the y-intercept).
Question 42. Explain the concept of smoothing in time series analysis with respect to removing fluctuations.
Answer:
Smoothing in Time Series Analysis
Smoothing in time series analysis is a technique used to reduce or eliminate short-term fluctuations and noise from the data, thereby making the underlying pattern, such as the trend or longer-term cycles, more visible and easier to analyze.
Time series data often contains variations due to seasonal, irregular, and sometimes short cyclical movements. These fluctuations can obscure the overall direction or underlying structure of the series, making it difficult to identify the fundamental trend or make reliable forecasts.
The core idea of smoothing is to average values over a certain period. This averaging process has the effect of:
- Damping out Random Fluctuations: Extreme values caused by irregular events tend to be averaged with more typical values, reducing their impact on the smoothed series.
- Averaging out Periodic Fluctuations: If the smoothing period is chosen appropriately (e.g., equal to the length of the seasonal cycle), the regular ups and downs of seasonality are averaged out within each period, effectively removing the seasonal component.
The result of applying a smoothing technique is a new series that is "smoother" or less volatile than the original one. This smoothed series provides a clearer representation of the trend and possibly longer cyclical movements, free from the noise of irregular and seasonal variations. Common smoothing methods include Moving Averages and Exponential Smoothing.
Question 43. If the trend equation is $Y_t = 50 + 3t$, where $t$ is measured in years and $t=0$ is the base year 2010, what is the predicted value for 2025?
Answer:
Given:
The trend equation is $Y_t = 50 + 3t$.
The time index $t$ is measured in years.
The base year is 2010, where $t=0$.
To Find:
The predicted value for the year 2025.
Solution:
First, we need to find the value of $t$ that corresponds to the year 2025.
The time index $t$ is the number of years from the base year 2010 ($t=0$).
Value of $t$ for 2025 = Year - Base Year Index
Value of $t$ for 2025 = $2025 - 2010$
Value of $t$ for 2025 = $15$
So, for the year 2025, $t=15$.
Now, substitute $t=15$ into the given trend equation:
${Y_t} = 50 + 3t$
${Y_{15}} = 50 + 3(15)$
Calculate the value:
${Y_{15}} = 50 + 45$
${Y_{15}} = 95$
Answer:
The predicted value for the year 2025 is 95.
Question 44. How can you visually differentiate between Seasonal and Cyclical variations on a time series graph?
Answer:
You can visually differentiate between Seasonal and Cyclical variations on a time series graph primarily by observing their periodicity and duration:
1. Seasonal Variation: This component appears as regular, repeating peaks and troughs within a fixed, relatively short period, most commonly within a calendar year (e.g., monthly or quarterly data). On a graph, you would see a consistent pattern that repeats itself year after year. The time between consecutive peaks (or troughs) is uniform and equal to the length of the seasonal cycle (e.g., 12 months or 4 quarters).
Visually, it looks like a fixed wave pattern superimposed on the underlying trend and cyclical movement. The timing of the peaks and troughs is predictable each year.
2. Cyclical Variation: This component appears as longer-term, undulating swings or oscillations that extend over periods typically longer than one year. On a graph, you would observe broad waves representing phases of expansion and contraction. Unlike seasonal patterns, these cycles do not necessarily have a fixed or regular duration or amplitude. One cycle might last 3 years, the next 5 years, and their intensity (height of the wave) can also vary.
Visually, they look like smoother, larger waves compared to the sharper, annual seasonal patterns. The time between consecutive peaks (or troughs) is irregular.
In summary: Look for regular, short-term, fixed-period patterns (Seasonal) versus irregular, longer-term, varying-period swings (Cyclical) on the graph.
Question 45. Give an example of a time series where the trend might be non-linear.
Answer:
A time series where the trend might be non-linear is the adoption rate or sales of a new technology or product over its lifecycle.
Initially, when a new technology or product is introduced, sales or adoption might be slow as only early adopters or niche markets pick it up. This represents a relatively flat or slowly increasing trend.
As the product gains popularity and word spreads, adoption accelerates rapidly. This phase shows a steep, increasing trend, which is faster than the initial phase.
Finally, as the market becomes saturated and most potential customers who want the product have bought it, the growth in sales or adoption slows down, flattening out. It might even decline slightly as the market is only driven by replacements or late adopters.
When plotted over time, the trend for such a series would typically follow an S-shaped curve (sigmoid curve), which is clearly non-linear. A simple straight line would not accurately capture the varying rate of growth over the product's lifecycle.
Question 46. Why is the order of data important in time series analysis?
Answer:
The order of data is fundamentally important in time series analysis because time series data is, by definition, a sequence of observations collected over successive points or intervals in time. The temporal order is what distinguishes a time series from other types of data.
Here's why the order is crucial:
1. Temporal Dependencies: Many time series exhibit temporal dependencies, meaning the value at a given time point is related to the values at previous time points (autocorrelation). Analyzing these dependencies (e.g., using ARIMA models) requires the data to be in the correct sequence.
2. Identification of Components: The major components of a time series – Trend, Seasonal variation, Cyclical variation, and Irregular variation – are all defined and identified by their patterns *over time*. The order of the data reveals the long-term trend, the regular annual patterns of seasonality, the longer-term swings of cycles, and the timing of irregular events.
3. Forecasting: Forecasting involves predicting future values based on past patterns. These patterns are only meaningful when the historical data is ordered correctly. Projecting a trend or seasonality forward relies entirely on understanding how the series has evolved sequentially.
4. Causality and Dynamics: Analyzing the dynamics of a system (how it changes over time) and potential causal relationships over time requires preserving the sequence of events. Lagged effects, for instance, can only be studied with ordered data.
If the order of a time series were shuffled, it would lose its temporal structure, making it impossible to identify trends, seasonal patterns, cyclical movements, or autocorrelation, and consequently rendering most time series analytical techniques and forecasting methods invalid.
Question 47. Calculate the 3-year moving total for the production data (in units): 1000, 1100, 1050, 1200, 1150, 1300.
Answer:
Given Data:
The production data (in units) are: 1000, 1100, 1050, 1200, 1150, 1300.
We need to calculate the 3-year moving totals. A 3-year moving total is the sum of three consecutive data points.
Calculations:
1. Moving Total for the period covering the 1st, 2nd, and 3rd data points:
Sum = $1000 + 1100 + 1050$
Sum = $3150$ units
This total is centered at the 2nd data point.
2. Moving Total for the period covering the 2nd, 3rd, and 4th data points:
Sum = $1100 + 1050 + 1200$
Sum = $3350$ units
This total is centered at the 3rd data point.
3. Moving Total for the period covering the 3rd, 4th, and 5th data points:
Sum = $1050 + 1200 + 1150$
Sum = $3400$ units
This total is centered at the 4th data point.
4. Moving Total for the period covering the 4th, 5th, and 6th data points:
Sum = $1200 + 1150 + 1300$
Sum = $3650$ units
This total is centered at the 5th data point.
Summary Table:
Year/Period | Production (units) | 3-Year Moving Total |
1 | 1000 | - |
2 | 1100 | 3150 (1000+1100+1050) |
3 | 1050 | 3350 (1100+1050+1200) |
4 | 1200 | 3400 (1050+1200+1150) |
5 | 1150 | 3650 (1200+1150+1300) |
6 | 1300 | - |
Question 48. If the trend line equation is $Y_t = 80 - 2t$, where $t=0$ for the midpoint of the data, what is the estimated value when $t=4$ (if $t$ represents coded values)?
Answer:
Given:
The trend line equation is $Y_t = 80 - 2t$, where $t$ represents coded time values and $t=0$ is the midpoint of the data.
We need to find the estimated value when $t=4$.
To Find:
The estimated value when $t=4$.
Solution:
The trend equation is given by:
${Y_t} = 80 - 2t$
We need to find the estimated value when $t=4$.
Substitute $t=4$ into the equation:
${Y_4} = 80 - 2(4)$
Calculate the value:
${Y_4} = 80 - 8$
${Y_4} = 72$
Answer:
The estimated value when $t=4$ is 72.
Question 49. Which method of measuring trend provides an equation that can be directly used for forecasting?
Answer:
The method of measuring trend that provides a mathematical equation which can be directly used for forecasting is the Method of Least Squares.
This method fits a predefined mathematical function (like a straight line $Y_t = a + bt$, a parabola $Y_t = a + bt + ct^2$, or other curves) to the time series data. Once the parameters of this function (e.g., $a$ and $b$ for a linear trend) are determined, the resulting equation explicitly relates the trend value ($Y_t$) to the time period ($t$).
To forecast the trend value for a future time period, you simply substitute the value of that future time period into the derived trend equation. This is a direct mathematical extrapolation based on the fitted model.
Question 50. What is the purpose of isolating the seasonal component of a time series?
Answer:
The purpose of isolating the seasonal component of a time series is primarily to understand, quantify, and remove the regular, predictable fluctuations that occur within a fixed period, typically a year.
Key reasons for isolating the seasonal component include:
1. Understanding Seasonal Patterns: It helps in identifying the precise pattern (magnitude and timing) of the seasonal peaks and troughs, allowing businesses and analysts to understand how factors like weather, holidays, or social customs affect the variable at different times of the year.
2. Deseasonalizing the Data: By removing the seasonal component, the underlying trend and cyclical components become more evident. The resulting "deseasonalized" series is useful for analyzing long-term growth or decline and identifying cyclical swings without the distraction of regular seasonal ups and downs.
3. Improved Forecasting: Once the seasonal pattern is isolated and quantified (e.g., as seasonal indices), it can be used to improve forecasting accuracy. Forecasts of the non-seasonal components (trend and cycle) can be made first using the deseasonalized data, and then the seasonal pattern can be added back to produce a final, more accurate forecast that reflects the expected seasonal fluctuations.
4. Operational Planning: Understanding seasonal variations is vital for operational planning, such as managing inventory levels, scheduling production, planning marketing activities, and allocating staff resources to match anticipated demand variations throughout the year.
5. Meaningful Comparisons: Deseasonalizing data allows for "month-on-month" or "quarter-on-quarter" comparisons that are not distorted by typical seasonal variations. For instance, comparing current month's sales to last month's sales is more informative if both figures are deseasonalized.
In essence, isolating seasonality allows for a clearer view of the other components and provides crucial information for both analysis and practical decision-making.
Question 51. Describe a situation where identifying cyclical variation would be important for a business.
Answer:
Identifying cyclical variation is crucial for a business operating in industries that are highly sensitive to the overall state of the economy and the broader business cycle. These are often referred to as "cyclical industries".
Example Situation:
Consider a large manufacturing company that produces durable goods, such as automobiles, heavy machinery, or construction equipment.
Importance of Identifying Cyclical Variation:
Demand for durable goods is heavily influenced by the economic cycle. During periods of economic prosperity and expansion, businesses and consumers have more confidence and capital, leading to increased investment in machinery, infrastructure, and vehicles. This results in high sales and production for the manufacturing company.
Conversely, during economic recessions or depressions, confidence is low, capital is scarce, and businesses cut back significantly on major investments. This causes demand for durable goods to plummet sharply, leading to reduced sales, production cuts, and potentially losses for the company.
For such a company, identifying and understanding the cyclical variation in demand is vital for:
- Production Planning: Accurately forecasting demand based on the cycle helps in scaling production up or down, avoiding costly overproduction during downturns and missed opportunities during upturns.
- Inventory Management: Preventing the accumulation of excessive inventory during anticipated declines and ensuring sufficient stock during peaks.
- Financial Planning: Projecting revenues, cash flow, and profitability based on the expected phase of the economic cycle to manage budgets and secure financing.
- Capital Investment Decisions: Timing major investments in new plants or equipment to coincide with or precede cyclical upswings.
- Workforce Management: Planning for hiring and training during expansions and potentially managing layoffs or reduced hours during contractions.
Ignoring cyclical variation could lead to significant financial instability, inefficient operations, and poor strategic decisions for businesses in these sensitive sectors.
Question 52. If the Method of Moving Averages is used with a 5-year period, how many trend values will be lost at the beginning and end of a series with 12 years of data?
Answer:
In the Method of Moving Averages, we lose the ability to calculate trend values for some periods at the beginning and end of the time series. This is because each calculation requires a full set of data points corresponding to the chosen period.
Given:
Period of Moving Average (m) = 5 years (an odd period).
Total number of years in the data (N) = 12 years.
Explanation:
For a moving average with an odd period 'm', the trend value is placed at the center of the time period it represents. The number of trend values lost at the beginning and at the end is given by the formula:
Number of lost values at each end = $\frac{m - 1}{2}$
Values lost at the beginning:
The first 5-year moving average will be calculated from the data of years 1, 2, 3, 4, and 5. This average is centered on the middle year, which is Year 3. Therefore, we cannot calculate trend values for Year 1 and Year 2.
Using the formula:
Number of lost values at the beginning = $\frac{5 - 1}{2} = \frac{4}{2} = 2$.
Values lost at the end:
The last 5-year moving average will be calculated from the data of the last five years, which are years 8, 9, 10, 11, and 12. This average is centered on the middle year, which is Year 10. Therefore, we cannot calculate trend values for Year 11 and Year 12.
Using the formula:
Number of lost values at the end = $\frac{5 - 1}{2} = \frac{4}{2} = 2$.
Conclusion:
With a 5-year moving average period, 2 trend values will be lost at the beginning and 2 trend values will be lost at the end of the series.
Question 53. Explain why the Method of Moving Averages is simpler to compute than the Method of Least Squares.
Answer:
The Method of Moving Averages is considered simpler to compute than the Method of Least Squares primarily because it relies on basic arithmetic operations, whereas the Method of Least Squares involves more complex algebraic calculations to fit a mathematical trend line.
Method of Moving Averages
This method calculates the trend by smoothing out fluctuations in the data. The computation involves:
- Simple Arithmetic: The core of the method is calculating the average of a specific number of consecutive data points. This only requires addition and division.
- Repetitive Process: The same simple calculation is repeated by "moving" the period one step forward at a time. For example, for a 3-year moving average, you first average years 1, 2, 3; then years 2, 3, 4; and so on.
- No Equation Fitting: It does not attempt to define the trend with a mathematical equation. It simply provides smoothed trend values for the middle of each period.
Method of Least Squares
This method fits a formal mathematical trend line, most commonly a straight line, to the data. The goal is to find the "best fit" line that minimizes the sum of the squared vertical distances from each data point to the line. The computation involves:
- Algebraic Formulae: It requires fitting a trend equation, typically of the form:
$Y_t = a + bX$
[where $Y_t$ is the trend value and $X$ is time]
- Solving Simultaneous Equations: To find the values of the constants 'a' (the Y-intercept) and 'b' (the slope), one must solve two "normal equations":
$\sum Y = na + b \sum X$
…(i)
$\sum XY = a \sum X + b \sum X^2$
…(ii)
- Multiple Calculations: Before solving the equations, you must first calculate several sums from your data: $\sum X$, $\sum Y$, $\sum X^2$, and $\sum XY$. This is a multi-step, calculation-intensive process.
Summary of Differences in Complexity
Aspect | Method of Moving Averages | Method of Least Squares |
Computational Nature | Simple arithmetic (addition, division) | Complex algebra (summation of products, solving simultaneous equations) |
Mathematical Skill | Basic arithmetic skills are sufficient. | Requires knowledge of algebra and solving equations. |
Trend Representation | Provides a set of smoothed values, not a mathematical function. | Provides a mathematical equation ($Y_t = a + bX$) that defines the entire trend. |
Process | Straightforward and repetitive averaging. | Involves several preliminary calculations ($\sum X$, $\sum Y$, etc.) before solving for the trend line parameters. |
Conclusion:
In essence, the simplicity of the Moving Averages method lies in its direct, arithmetic approach to smoothing data. In contrast, the Method of Least Squares is computationally more demanding because it involves an indirect, algebraic approach to derive a precise mathematical model for the trend, requiring the solution of simultaneous equations.
Question 54. What kind of trend is represented by the equation $Y_t = a \cdot b^t$ (exponential trend)? (Assuming linear trend is the main focus, but this might be a conceptual SA question).
Answer:
The equation $Y_t = a \cdot b^t$ represents an exponential trend, also known as a geometric trend or a trend with a constant rate of change.
Characteristics of an Exponential Trend:
An exponential trend is fundamentally different from a linear trend ($Y_t = a + bt$). While a linear trend describes a situation where the variable changes by a constant amount in each time period, an exponential trend describes a situation where the variable changes by a constant percentage or ratio in each time period.
In the equation $Y_t = a \cdot b^t$:
- $Y_t$ is the trend value at time $t$.
- $a$ is the initial value of the trend, i.e., the value of $Y_t$ when $t=0$.
- $b$ is the constant ratio of change (the growth or decay factor). It is equal to $1 + r$, where $r$ is the constant rate of growth or decay.
- $t$ is the time variable.
The nature of the trend depends on the value of $b$:
- If $b > 1$, the trend shows exponential growth. The value of $Y_t$ increases by a constant percentage in each period. For example, if $b=1.05$, the trend value increases by 5% each year. The graph of this trend curves upward.
- If $0 < b < 1$, the trend shows exponential decay. The value of $Y_t$ decreases by a constant percentage in each period. For example, if $b=0.90$, the trend value decreases by 10% each year. The graph of this trend curves downward, approaching zero.
Comparison with a Linear Trend:
Feature | Linear Trend ($Y_t = a + bt$) | Exponential Trend ($Y_t = a \cdot b^t$) |
Type of Change | The trend value changes by a constant amount ($b$) in each time period. (Additive change) | The trend value changes by a constant ratio or percentage ($b$) in each time period. (Multiplicative change) |
Example | Sales increase by $\textsf{₹}1,000$ every year. | Sales increase by 10% every year. |
Graphical Representation | Forms a straight line on a standard arithmetic scale graph. | Forms a curve on a standard graph, but becomes a straight line on a semi-logarithmic graph (where the Y-axis is a log scale). |
Conclusion:
In summary, the equation $Y_t = a \cdot b^t$ represents a trend where the variable is compounded over time, leading to either accelerating growth or decelerating decay, which is characteristic of many real-world phenomena like population growth, compound interest, or radioactive decay.
Long Answer Type Questions
Question 1. Calculate the 5-year moving averages for the following data on the yield of a crop (in quintals per acre):
Year | Yield |
2010 | 18 |
2011 | 20 |
2012 | 22 |
2013 | 25 |
2014 | 23 |
2015 | 26 |
2016 | 28 |
2017 | 27 |
2018 | 30 |
2019 | 29 |
2020 | 32 |
2021 | 31 |
Answer:
Calculation of 5-Year Moving Averages
To calculate the 5-year moving averages, we will sum the yield values for five consecutive years and then divide the sum by 5. Since the period is an odd number (5), the resulting average, which is the trend value, is placed at the center of the time period (i.e., the third year of the five-year block).
The first trend value will correspond to the year 2012, calculated from the data of 2010 through 2014. The last trend value will correspond to the year 2019, calculated from the data of 2017 through 2021. As a result, we will lose the trend values for the first two years (2010, 2011) and the last two years (2020, 2021).
The detailed calculations are presented in the table below.
Year | Yield (in quintals) | 5-Year Moving Total | 5-Year Moving Average (Trend) |
2010 | 18 | - | - |
2011 | 20 | - | - |
2012 | 22 | $18+20+22+25+23 = 108$ | $108 \div 5 = 21.6$ |
2013 | 25 | $20+22+25+23+26 = 116$ | $116 \div 5 = 23.2$ |
2014 | 23 | $22+25+23+26+28 = 124$ | $124 \div 5 = 24.8$ |
2015 | 26 | $25+23+26+28+27 = 129$ | $129 \div 5 = 25.8$ |
2016 | 28 | $23+26+28+27+30 = 134$ | $134 \div 5 = 26.8$ |
2017 | 27 | $26+28+27+30+29 = 140$ | $140 \div 5 = 28.0$ |
2018 | 30 | $28+27+30+29+32 = 146$ | $146 \div 5 = 29.2$ |
2019 | 29 | $27+30+29+32+31 = 149$ | $149 \div 5 = 29.8$ |
2020 | 32 | - | - |
2021 | 31 | - | - |
Plotting the Data and Trend Values
To visualize the results, we plot both the original yield data and the calculated moving average trend values on a single graph.
- X-Axis: Year (from 2010 to 2021)
- Y-Axis: Yield (in quintals per acre)
Two lines will be drawn on the graph:
- Original Data Line: This line connects the actual yield values for each year. It will show more fluctuations as it represents the raw data. The points for this line are (2010, 18), (2011, 20), (2012, 22), ..., (2021, 31).
- Moving Average Trend Line: This line connects the calculated 5-year moving average values. It will appear much smoother than the original data line, as the averaging process removes short-term fluctuations and reveals the underlying trend. The points for this line are (2012, 21.6), (2013, 23.2), (2014, 24.8), ..., (2019, 29.8).
The resulting graph would clearly show a generally increasing trend in the crop yield over the period, with the trend line providing a smoothed representation of this growth.
Question 2. Calculate the 4-year centered moving averages for the following production data (in thousand units):
Year | Production |
2015 | 80 |
2016 | 85 |
2017 | 92 |
2018 | 88 |
2019 | 95 |
2020 | 90 |
2021 | 98 |
2022 | 94 |
2023 | 100 |
2024 | 96 |
Answer:
Calculation of 4-Year Centered Moving Averages
When the period of the moving average is an even number, such as 4 years, the calculated average falls between two time periods. To address this, a "centering" process is required to align the trend value with a specific year. The procedure involves two main steps:
- Calculate 4-Year Moving Averages (Uncentered): First, we calculate the sum of production for four consecutive years and divide by 4. This average is placed at the midpoint between the second and third years of the 4-year block.
- Center the Averages: Next, we take the average of two consecutive uncentered moving averages. This new average becomes the centered trend value and is aligned with the year that falls between the two uncentered averages.
This two-step process means we will lose the trend values for the first two years (2015, 2016) and the last two years (2023, 2024). The calculations are detailed in the table below.
Year | Production (in '000 units) | 4-Year Moving Total | 4-Year Moving Average (Uncentered) | 4-Year Centered Moving Average (Trend) |
2015 | 80 | - | - | - |
2016 | 85 | $80+85+92+88 = 345$ | $345 \div 4 = 86.25$ | - |
2017 | 92 | $(86.25 + 90.00) \div 2 = \mathbf{88.125}$ | ||
2018 | 88 | $85+92+88+95 = 360$ | $360 \div 4 = 90.00$ | $(90.00 + 91.25) \div 2 = \mathbf{90.625}$ |
2019 | 95 | $92+88+95+90 = 365$ | $365 \div 4 = 91.25$ | $(91.25 + 92.75) \div 2 = \mathbf{92.000}$ |
2020 | 90 | $88+95+90+98 = 371$ | $371 \div 4 = 92.75$ | $(92.75 + 94.25) \div 2 = \mathbf{93.500}$ |
2021 | 98 | $95+90+98+94 = 377$ | $377 \div 4 = 94.25$ | $(94.25 + 95.50) \div 2 = \mathbf{94.875}$ |
2022 | 94 | $90+98+94+100 = 382$ | $382 \div 4 = 95.50$ | $(95.50 + 97.00) \div 2 = \mathbf{96.250}$ |
2023 | 100 | $98+94+100+96 = 388$ | $388 \div 4 = 97.00$ | - |
2024 | 96 | - | - | - |
Plotting the Data and Trend Values
To plot the data, a graph should be created with the 'Year' on the horizontal axis (x-axis) and 'Production (in thousand units)' on the vertical axis (y-axis). Two lines will be drawn on this graph:
- Original Data Line: This line connects the points representing the actual production data for each year from 2015 to 2024. The points are (2015, 80), (2016, 85), (2017, 92), and so on. This line will show the year-to-year fluctuations.
- Centered Moving Average Trend Line: This line connects the calculated centered trend values. It will be a shorter and smoother line, running from 2017 to 2022. The points for this line are (2017, 88.125), (2018, 90.625), (2019, 92.000), etc.
The trend line will reveal the underlying, long-term movement in production by smoothing out the short-term variability present in the original data.
Question 3. Fit a linear trend line $Y_t = a + bt$ using the Method of Least Squares for the following sales data (in $\textsf{₹}$ lakhs):
Year | Sales |
2018 | 50 |
2019 | 55 |
2020 | 62 |
2021 | 60 |
2022 | 68 |
Answer:
Solution: Fitting a Linear Trend Line by Method of Least Squares
The goal is to fit a linear trend line of the form $Y_t = a + bt$ to the given sales data. The constants 'a' and 'b' are determined by solving the following two normal equations:
$\sum Y = na + b \sum X$
…(i)
$\sum XY = a \sum X + b \sum X^2$
…(ii)
Here, $n$ is the number of years. Since $n=5$ (an odd number), we can simplify the calculations by choosing the middle year (2020) as the origin. This makes the sum of the time variable, $\sum X$, equal to zero.
Calculation Table
We set up a table to compute the necessary sums: $\sum Y$, $\sum X$, $\sum X^2$, and $\sum XY$.
Year | Sales (Y) (in $\textsf{₹}$ lakhs) | Time Deviation (X) (Origin: 2020) | $X^2$ | $XY$ |
2018 | 50 | -2 | 4 | -100 |
2019 | 55 | -1 | 1 | -55 |
2020 | 62 | 0 | 0 | 0 |
2021 | 60 | 1 | 1 | 60 |
2022 | 68 | 2 | 4 | 136 |
$n=5$ | $\sum Y = 295$ | $\sum X = 0$ | $\sum X^2 = 10$ | $\sum XY = 41$ |
Since $\sum X = 0$, the normal equations simplify to:
$a = \frac{\sum Y}{n}$
$b = \frac{\sum XY}{\sum X^2}$
Substituting the values from the table:
$a = \frac{295}{5} = 59$
$b = \frac{41}{10} = 4.1$
Therefore, the fitted linear trend line equation is:
$Y_t = 59 + 4.1X$
(Origin: 2020, X unit: 1 year, Y unit: in $\textsf{₹}$ lakhs)
Interpretation of the Values of $a$ and $b$
- Value of $a$ (Y-intercept): The value $a = 59$ represents the trend value of sales for the origin year, which is 2020. This means the estimated sales for the year 2020, according to the trend line, is $\textsf{₹}59$ lakhs.
- Value of $b$ (Slope): The value $b = 4.1$ represents the average annual rate of change in sales. Since $b$ is positive, it indicates an increasing trend. On average, the sales are increasing by $\textsf{₹}4.1$ lakhs per year.
Estimation of Sales for the Year 2025
To estimate the sales for the year 2025, we first need to find the corresponding value of $X$. Since the origin is 2020:
$X$ for 2025 = $2025 - 2020 = 5$
Now, we substitute $X=5$ into our trend equation:
$Y_{2025} = 59 + 4.1(5)$
$Y_{2025} = 59 + 20.5$
$Y_{2025} = 79.5$
Thus, the estimated sales for the year 2025 are $\textsf{₹}79.5$ lakhs.
Question 4. Fit a linear trend line $Y_t = a + bt$ using the Method of Least Squares for the following production data (in thousand units):
Year | Production |
2019 | 120 |
2020 | 125 |
2021 | 118 |
2022 | 130 |
2023 | 128 |
2024 | 135 |
Answer:
Solution: Fitting a Linear Trend Line for Even Years
We need to fit a linear trend line of the form $Y_t = a + bt$ to the given production data using the Method of Least Squares. The constants 'a' and 'b' are found by solving the two normal equations:
$\sum Y = na + b \sum X$
…(i)
$\sum XY = a \sum X + b \sum X^2$
…(ii)
Since the number of years ($n=6$) is even, we must use coded time values for $X$ to make the sum $\sum X = 0$. The origin is set at the midpoint of the two central years, which is $(2021 + 2022) \div 2 = 2021.5$. The time deviations are then calculated in half-year units, resulting in coded values of ..., -3, -1, 1, 3, ...
Calculation Table
We construct a table to find the required sums: $\sum Y$, $\sum X$, $\sum X^2$, and $\sum XY$.
Year | Production (Y) (in '000 units) | Coded Time (X) (Origin: 2021.5, unit: 0.5 year) | $X^2$ | $XY$ |
2019 | 120 | -5 | 25 | -600 |
2020 | 125 | -3 | 9 | -375 |
2021 | 118 | -1 | 1 | -118 |
2022 | 130 | 1 | 1 | 130 |
2023 | 128 | 3 | 9 | 384 |
2024 | 135 | 5 | 25 | 675 |
$n=6$ | $\sum Y = 756$ | $\sum X = 0$ | $\sum X^2 = 70$ | $\sum XY = 96$ |
With $\sum X = 0$, the normal equations are simplified to:
$a = \frac{\sum Y}{n}$
$b = \frac{\sum XY}{\sum X^2}$
Substituting the sum values from the table:
$a = \frac{756}{6} = 126$
$b = \frac{96}{70} \approx 1.3714$
Thus, the fitted linear trend line equation is:
$Y_t = 126 + 1.3714X$
(Origin: 2021.5, X unit: half-year, Y unit: in thousand units)
Interpretation of the Values of $a$ and $b$
- Value of $a$ (Y-intercept): The value $a = 126$ is the trend value of production at the origin (X=0), which is the midpoint between 2021 and 2022. This means the estimated production for mid-2021 is 126 thousand units.
- Value of $b$ (Slope): The value $b \approx 1.3714$ is the rate of change in production per unit of X. Since one unit of X is a half-year, this means the production is increasing by approximately 1.3714 thousand units every six months. The average annual increase is $2 \times b = 2 \times 1.3714 = 2.7428$ thousand units.
Prediction of Production for the Year 2026
To predict the production for 2026, we first determine its coded time value ($X$).
$X \text{ for 2026} = \frac{\text{Year} - \text{Origin}}{\text{Time Unit}} = \frac{2026 - 2021.5}{0.5} = \frac{4.5}{0.5} = 9$
Now, we substitute $X=9$ into the trend equation:
$Y_{2026} = 126 + 1.3714(9)$
$Y_{2026} = 126 + 12.3426$
$Y_{2026} \approx 138.34$
Therefore, the predicted production for the year 2026 is approximately 138.34 thousand units.
Question 5. Explain in detail the four components of a time series: Secular Trend, Seasonal Variation, Cyclical Variation, and Irregular Variation. For each component, provide examples of phenomena in the Indian context that would exhibit this type of variation.
Answer:
A time series is a sequence of data points collected over a period of time. Analyzing a time series often involves decomposing it into its constituent components, which help in understanding the underlying patterns and forecasting future values. The four primary components of a time series are:
1. Secular Trend (T):
The secular trend represents the long-term movement or general direction of a time series over an extended period, typically years or decades. It reflects the underlying growth or decline in the data, unaffected by short-term fluctuations. This trend can be upward, downward, or horizontal (static).
Characteristics:
- Occurs over a long period.
- Represents the overall tendency of the data.
- Can be linear or non-linear.
Examples in the Indian context:
- Population Growth: India's population has shown a consistent upward secular trend over the past several decades due to factors like improved healthcare, increased life expectancy, and sustained birth rates.
- Economic Growth (GDP): The Gross Domestic Product (GDP) of India has generally exhibited an upward secular trend, indicating long-term economic expansion, despite short-term fluctuations.
- Technological Adoption: The increasing penetration of smartphones and internet usage in India demonstrates a clear upward secular trend over the last decade.
2. Seasonal Variation (S):
Seasonal variation refers to the regular and predictable patterns of fluctuation that occur within a specific period, usually a year or less (e.g., daily, weekly, monthly, quarterly). These variations are often caused by weather conditions, holidays, or customs.
Characteristics:
- Occurs within a fixed period (e.g., annually, quarterly).
- Is repetitive and predictable.
- Caused by natural or man-made factors related to the calendar.
Examples in the Indian context:
- Retail Sales: Sales of certain goods, like apparel or electronics, tend to surge during festival seasons such as Diwali or Christmas, showing a clear seasonal pattern.
- Agricultural Output: The production of crops in India is highly seasonal, with distinct planting and harvesting seasons influenced by the monsoon and other climatic factors. For example, the demand for agricultural machinery is highest during the pre-sowing and post-harvesting periods.
- Tourism: Tourist arrivals in popular destinations in India, such as hill stations or beaches, typically peak during specific months due to favorable weather conditions (e.g., summer for hill stations, winter for beaches).
3. Cyclical Variation (C):
Cyclical variation refers to the fluctuations in a time series that occur over periods longer than a year but are not necessarily of a fixed duration. These cycles are often associated with business cycles (upswings and downswings in economic activity) or longer-term patterns influenced by economic, social, or political factors.
Characteristics:
- Occurs over periods longer than one year.
- Are not of fixed duration and can vary in length and amplitude.
- Often related to economic booms and busts.
Examples in the Indian context:
- Stock Market Indices: The performance of Indian stock markets, like the BSE Sensex or NSE Nifty, often exhibits cyclical patterns reflecting periods of economic expansion and recession, investor sentiment, and global economic trends.
- Real Estate Prices: Property prices in major Indian cities can experience cyclical booms and busts driven by factors like interest rates, demand-supply dynamics, and economic growth.
- Investment in Infrastructure: Government spending and private investment in large-scale infrastructure projects can follow a cyclical pattern, with periods of significant investment followed by periods of consolidation.
4. Irregular Variation (I) or Random Variation:
Irregular variation, also known as random or residual variation, represents the erratic, unpredictable, and short-term fluctuations in a time series that cannot be explained by the other three components. These variations are due to unforeseen events, random shocks, or measurement errors.
Characteristics:
- Unpredictable and random.
- Short-lived and erratic.
- Can be positive or negative.
Examples in the Indian context:
- Natural Disasters: The impact of unexpected natural disasters such as floods, earthquakes, or cyclones can cause sudden, irregular dips in economic activity, agricultural output, or infrastructure damage.
- Sudden Policy Changes: Unforeseen government policy changes, like a sudden demonetization or a shift in import-export regulations, can lead to temporary, irregular disruptions in economic data.
- Global Events: Major global events, such as the COVID-19 pandemic, caused unprecedented and irregular shocks to supply chains, consumer behavior, and economic activity worldwide, including in India.
In summary, understanding these four components is crucial for analyzing time series data, identifying underlying patterns, and making informed predictions about future trends.
Question 6. Describe the Method of Moving Averages for determining trend. Discuss its advantages and disadvantages. Illustrate the smoothing effect of a 3-year moving average on a hypothetical time series with some fluctuations.
Answer:
Method of Moving Averages for Determining Trend
The Method of Moving Averages is a statistical technique used to analyze time series data to identify underlying trends by smoothing out short-term fluctuations and cyclical variations. It involves calculating the average of consecutive subsets of the data over a specified period (the "moving" period).
How it works:
For a given period of length 'n', the moving average is calculated by summing up 'n' consecutive data points and dividing by 'n'. This average is then assigned to the middle point of that period (for odd 'n') or to the end of the period (for even 'n', often requiring a further adjustment). The process is then repeated by shifting the window one period forward, effectively "moving" the average across the time series.
Illustrative Example: 3-Year Moving Average
Let's consider a hypothetical time series of annual sales over 7 years:
Year | Sales (in thousands) | 3-Year Moving Total | 3-Year Moving Average (Trend) |
2017 | 50 | ||
2018 | 55 | $50 + 55 + 60 = 165$ | $165 / 3 = 55$ (Trend for 2018) |
2019 | 60 | $55 + 60 + 58 = 173$ | $173 / 3 = 57.67$ (Trend for 2019) |
2020 | 58 | $60 + 58 + 65 = 183$ | $183 / 3 = 61$ (Trend for 2020) |
2021 | 65 | $58 + 65 + 70 = 193$ | $193 / 3 = 64.33$ (Trend for 2021) |
2022 | 70 | $65 + 70 + 72 = 207$ | $207 / 3 = 69$ (Trend for 2022) |
2023 | 72 |
In this table, the '3-Year Moving Total' sums the sales for three consecutive years. The '3-Year Moving Average' is then calculated by dividing this total by 3. The resulting moving averages represent the smoothed trend of the sales data, with the fluctuations due to other factors reduced.
Advantages of the Method of Moving Averages:
- Simplicity: It is relatively easy to understand and compute, making it accessible for basic trend analysis.
- Smoothing Effect: It effectively smooths out random fluctuations and seasonal variations, making the underlying trend more apparent.
- Identification of Trend: It helps in visualizing and identifying the general direction of the time series (upward, downward, or horizontal).
- Forecasting (Short-term): The smoothed trend can be used as a basis for short-term forecasting, assuming the trend continues.
Disadvantages of the Method of Moving Averages:
- Loss of Data Points: The first and last few data points of the time series cannot have a moving average calculated for them, leading to a loss of data at the beginning and end. For an 'n'-period moving average, n-1/2 data points are lost at each end.
- Lagging Indicator: Moving averages are lagging indicators. They are based on past data, so they tend to smooth out peaks and troughs and may not accurately reflect the current trend if there are sudden changes.
- Choice of Period (n): The effectiveness of the moving average is highly dependent on the chosen period 'n'. A short period will result in less smoothing, while a long period might over-smooth the data and mask important turning points.
- Doesn't Account for Seasonality/Cyclicality Explicitly: While it smooths them, it doesn't explicitly isolate or measure seasonal or cyclical components.
- Not Suitable for Rapidly Changing Trends: If the trend itself is changing rapidly, the lagging nature of moving averages can lead to inaccurate estimations.
Smoothing Effect Illustration:
The table above illustrates the smoothing effect. Compare the original 'Sales' data with the '3-Year Moving Average'. Notice how the moving average values are less volatile. For instance, the sales in 2018 were 55, and in 2019 were 60, but the trend value for 2018 (calculated using 2017-2019 data) is 55, and for 2019 (using 2018-2020 data) is 57.67. The fluctuations between individual years are reduced, revealing a more consistent upward trend.
If we were to plot both the original sales data and the 3-year moving average, the moving average line would appear much smoother than the original data line, effectively 'moving' through the fluctuations to reveal the underlying trend.
Question 7. Explain the Method of Least Squares for fitting a linear trend line. Discuss its advantages and disadvantages compared to the Method of Moving Averages. When would you prefer to use Least Squares over Moving Averages?
Answer:
Method of Least Squares for Fitting a Linear Trend Line
The Method of Least Squares is a mathematical technique used to find the best-fitting straight line (or curve) through a set of data points. For a linear trend, the goal is to determine the equation of a line, typically represented as $y = a + bx$, where:
- $y$ is the dependent variable (e.g., sales, stock price)
- $x$ is the independent variable (e.g., time)
- $a$ is the y-intercept (the value of y when x is 0)
- $b$ is the slope of the line (representing the rate of change of y with respect to x)
The method works by minimizing the sum of the squares of the vertical distances (residuals) between the actual data points and the points on the fitted line. These distances represent the errors or deviations of the observed values from the predicted values by the line. The line that results in the smallest sum of squared errors is considered the "best-fit" line.
The formulas derived from the least squares method to calculate $a$ and $b$ are:
$b = \frac{n \sum(xy) - \sum x \sum y}{n \sum(x^2) - (\sum x)^2}$
$a = \frac{\sum y - b \sum x}{n}$
Where:
- $n$ is the number of data points.
- $\sum(xy)$ is the sum of the products of x and y.
- $\sum x$ is the sum of the x values.
- $\sum y$ is the sum of the y values.
- $\sum(x^2)$ is the sum of the squares of the x values.
- $(\sum x)^2$ is the square of the sum of the x values.
Comparison with Method of Moving Averages:
Advantages of Least Squares over Moving Averages:
- Objective and Precise: Least squares provides a mathematically precise and objective way to determine the trend line, based on all data points. Moving averages are more subjective in the choice of the period.
- Uses All Data Points: Unlike moving averages, which discard data at the ends, the least squares method utilizes all available data points to fit the trend line. This results in a more robust trend estimation.
- No Lag: The trend line fitted by least squares represents the overall trend of the entire series, and it doesn't inherently lag behind the data like moving averages do.
- Provides a Formulaic Trend: It gives an explicit equation ($y = a + bx$) for the trend, which can be easily used for forecasting.
- Measures Trend Magnitude: The slope ($b$) directly quantifies the average rate of change (trend) per unit of the independent variable.
Disadvantages of Least Squares compared to Moving Averages:
- Assumption of Linearity: It strictly assumes a linear relationship. If the underlying trend is curved or non-linear, a linear least squares line will not fit the data well.
- Sensitivity to Outliers: The squaring of residuals makes the least squares method highly sensitive to outliers. A single extreme data point can significantly skew the fitted line.
- More Complex Calculation: While software makes it easy, the manual calculation involves more steps and understanding of statistical formulas compared to simple moving averages.
- Less Intuitive Smoothing: It doesn't "smooth" in the same visual sense as moving averages. While it minimizes errors, it might still show a relatively jagged trend if the original data is very volatile around the line.
When to Prefer Least Squares over Moving Averages:
You would prefer to use the Method of Least Squares over Moving Averages in the following scenarios:
- When a Linear Trend is Expected: If you have reason to believe that the underlying trend in your data is approximately linear (e.g., steady growth or decline over time).
- When Maximum Data Utilization is Required: If you need to incorporate all available data points into the trend estimation and avoid the loss of data at the beginning and end of the series.
- For Accurate Forecasting: When precise forecasting based on a well-defined trend equation is needed. The equation from least squares is a powerful tool for this.
- When Objective and Robust Estimation is Crucial: If you need a statistically sound method that minimizes errors across the entire dataset, provided outliers are managed or not a significant concern.
- To Understand the Rate of Change: When you need to quantify the average rate of increase or decrease in the dependent variable over time (i.e., the slope).
In essence, Least Squares is preferred when a more rigorous, mathematically derived, and comprehensive trend analysis is required, particularly when a linear relationship is a reasonable assumption.
Question 8. The quarterly sales data (in $\textsf{₹}$ thousands) for a retail store for 3 years is given below:
Year | Quarter | Sales |
2022 | Q1 | 80 |
2022 | Q2 | 100 |
2022 | Q3 | 120 |
2022 | Q4 | 90 |
2023 | Q1 | 85 |
2023 | Q2 | 105 |
2023 | Q3 | 125 |
2023 | Q4 | 95 |
2024 | Q1 | 90 |
2024 | Q2 | 110 |
2024 | Q3 | 130 |
2024 | Q4 | 100 |
Answer:
Calculation of 4-Quarter Centered Moving Average
To calculate the 4-quarter centered moving average, we first find the 4-quarter moving totals. Since we have an even number of periods (4 quarters), we need to perform a second step of averaging to center the moving average.
Step 1: Calculate 4-Quarter Moving Totals
We sum up sales for consecutive four quarters. The first total will be for Q1 2022 to Q4 2022. The next total will be for Q2 2022 to Q1 2023, and so on.
Step 2: Calculate 4-Quarter Centered Moving Averages (2 x 4 Moving Average)
For each 4-quarter moving total, we calculate a 2-period moving average. This is done by summing two consecutive 4-quarter moving totals and dividing by 2 (effectively summing 8 quarters and dividing by 8). This average is then centered to the middle of the 8-quarter period, which corresponds to the midpoint between the two centered quarters of the original 4-quarter totals.
Year | Quarter | Sales (₹ thousands) | 4-Quarter Moving Total | 2 x 4 Moving Total (for centering) | 4-Quarter Centered Moving Average (Trend) |
2022 | Q1 | 80 | |||
2022 | Q2 | 100 | |||
2022 | Q3 | 120 | $80+100+120+90 = 390$ | ||
2022 | Q4 | 90 | $100+120+90+85 = 395$ | $390 + 395 = 785$ | $785 / 2 = 392.5$ (Trend for Mid-2023) |
2023 | Q1 | 85 | $120+90+85+105 = 400$ | $395 + 400 = 795$ | $795 / 2 = 397.5$ (Trend for Mid-2023) |
2023 | Q2 | 105 | $90+85+105+125 = 405$ | $400 + 405 = 805$ | $805 / 2 = 402.5$ (Trend for Mid-2023) |
2023 | Q3 | 125 | $85+105+125+95 = 410$ | $405 + 410 = 815$ | $815 / 2 = 407.5$ (Trend for Mid-2023) |
2023 | Q4 | 95 | $105+125+95+110 = 435$ | $410 + 435 = 845$ | $845 / 2 = 422.5$ (Trend for Mid-2023) |
2024 | Q1 | 90 | $125+95+110+130 = 460$ | $435 + 460 = 895$ | $895 / 2 = 447.5$ (Trend for Mid-2023) |
2024 | Q2 | 110 | $95+110+130+100 = 435$ | $460 + 435 = 895$ | $895 / 2 = 447.5$ (Trend for Mid-2023) |
2024 | Q3 | 130 | $110+130+100 = 340$ (Incomplete 4-quarter sum) | ||
2024 | Q4 | 100 |
Explanation of Centering:
A 4-quarter moving average for a period, say Q1 2022 to Q4 2022, would naturally be centered at the midpoint of this period, which falls between Q2 2022 and Q3 2022. By taking a further 2-period average of consecutive 4-quarter moving totals, we essentially align the average to a more precise point in time. For example, the first centered average (392.5) is derived from the totals of (Q1'22-Q4'22) and (Q2'22-Q1'23). The midpoint of this combined 8-quarter period is the middle of Q2'22 and Q3'22, which is effectively the middle of Q3 2022. Therefore, the trend values are associated with the middle of the quarters.
The trend values are often listed against the middle quarter of the original 4-quarter sum. For example, the first calculated trend value (392.5) corresponds to the period from Q1 2022 to Q4 2022. The center of this period is between Q2 and Q3 2022. The next trend value (397.5) is for the period Q2 2022 to Q1 2023, which centers between Q3 2022 and Q4 2022. The average of these two is then assigned to the middle of Q3 2022.
Note: The first and last three quarters do not have complete 4-quarter moving averages, so trend values cannot be calculated for them.
Question 9. A company's annual profit (in $\textsf{₹}$ crores) over 8 years is given:
Year | Profit |
2015 | 3.5 |
2016 | 3.8 |
2017 | 4.0 |
2018 | 4.2 |
2019 | 4.5 |
2020 | 4.3 |
2021 | 4.6 |
2022 | 4.8 |
Answer:
Fitting a Linear Trend Line using the Method of Least Squares
We are fitting a linear trend line of the form $y = a + bx$, where $y$ is the profit and $x$ is the year. To simplify calculations, we can represent the years by assigning a numerical value to them. Let's set the base year 2015 as $x=1$. Then the years and corresponding $x$ values are:
Year | Profit (y) | x | $x^2$ | xy |
2015 | 3.5 | 1 | 1 | 3.5 |
2016 | 3.8 | 2 | 4 | 7.6 |
2017 | 4.0 | 3 | 9 | 12.0 |
2018 | 4.2 | 4 | 16 | 16.8 |
2019 | 4.5 | 5 | 25 | 22.5 |
2020 | 4.3 | 6 | 36 | 25.8 |
2021 | 4.6 | 7 | 49 | 32.2 |
2022 | 4.8 | 8 | 64 | 38.4 |
Totals | 33.7 | 36 | 204 | 158.8 |
Here, $n = 8$ (number of years).
Using the formulas for least squares:
$b = \frac{n \sum(xy) - \sum x \sum y}{n \sum(x^2) - (\sum x)^2}$
$b = \frac{8 \times 158.8 - 36 \times 33.7}{8 \times 204 - (36)^2}$
$b = \frac{1270.4 - 1213.2}{1632 - 1296}$
$b = \frac{57.2}{336}$
$b \approx 0.1702$
$a = \frac{\sum y - b \sum x}{n}$
$a = \frac{33.7 - (0.1702 \times 36)}{8}$
$a = \frac{33.7 - 6.1272}{8}$
$a = \frac{27.5728}{8}$
$a \approx 3.4466$
Therefore, the linear trend line is approximately:
$y = 3.4466 + 0.1702x$
Predicting Profit for the Year 2027
The year 2027 corresponds to $x = 2027 - 2015 + 1 = 13$.
Substitute $x = 13$ into the trend line equation:
$y_{2027} = 3.4466 + 0.1702 \times 13$
$y_{2027} = 3.4466 + 2.2126$
$y_{2027} \approx 5.6592$
The predicted profit for the year 2027 is approximately $\textsf{₹}$ 5.66 crores.
Question 10. Discuss various applications of Time Series analysis in different fields like Economics, Business, and Meteorology, providing specific examples for each in the Indian context.
Answer:
Time Series Analysis is a powerful statistical method used to analyze data points collected over time. It helps in understanding past patterns, identifying trends, seasonality, and cyclical behavior, and making predictions about future values. Its applications are vast and span across numerous fields.
1. Economics
In economics, time series analysis is crucial for understanding economic behavior, forecasting economic indicators, and formulating policy. It helps in analyzing trends in GDP, inflation, unemployment, interest rates, and stock markets.
- Forecasting GDP Growth: Agencies like the Reserve Bank of India (RBI) and the National Statistical Office (NSO) use time series models (e.g., ARIMA, VAR) to forecast India's Gross Domestic Product (GDP) growth rate. This helps in economic planning and policy decisions. For example, analyzing the historical GDP data alongside other economic indicators like industrial production and inflation helps in predicting future economic performance.
- Inflation Rate Prediction: Predicting the Consumer Price Index (CPI) or Wholesale Price Index (WPI) is vital for monetary policy. Models are built using historical inflation data, commodity prices, and other economic factors to forecast future inflation, enabling the RBI to make informed decisions on interest rates.
- Unemployment Rate Forecasting: Analyzing historical unemployment data helps government bodies understand labor market trends and forecast future unemployment rates. This information is critical for designing employment generation schemes and social welfare programs.
- Stock Market Analysis: Financial institutions and individual investors use time series techniques to analyze the movement of stock prices on exchanges like the NSE and BSE. Models can predict future stock prices, although the inherent volatility of financial markets makes this challenging.
2. Business
Businesses utilize time series analysis for demand forecasting, inventory management, sales prediction, and understanding market trends. This helps in optimizing operations, resource allocation, and strategic planning.
- Sales Forecasting: Companies like Hindustan Unilever (HUL) or Tata Motors use historical sales data to forecast future sales of their products. This helps in production planning, inventory management, and marketing strategies. For instance, predicting seasonal sales of ice cream in summer months or demand for specific vehicle models.
- Inventory Management: Retailers use time series analysis to predict the demand for various products. This helps them maintain optimal inventory levels, reducing the risk of stockouts or excessive holding costs. For example, predicting the demand for essential medicines in a pharmacy chain.
- Website Traffic Prediction: E-commerce companies like Flipkart and Amazon analyze historical website traffic data to forecast user visits. This helps in managing server capacity and planning promotional activities.
- Resource Planning: Utility companies analyze historical data on electricity or water consumption to forecast future demand, enabling them to plan resource generation and distribution efficiently. For example, predicting peak electricity demand during festivals or extreme weather conditions.
3. Meteorology
In meteorology, time series analysis is fundamental for weather forecasting, climate modeling, and understanding weather patterns. It helps in predicting temperature, rainfall, wind speed, and other atmospheric conditions.
- Rainfall Prediction for Agriculture: The India Meteorological Department (IMD) uses extensive time series data of past rainfall patterns, temperature, and atmospheric pressure to forecast monsoon patterns. This is critically important for India's agricultural sector, helping farmers decide on crop cycles and irrigation needs.
- Temperature Forecasting: Predicting daily, weekly, or monthly temperature variations helps in managing energy demand (for heating and cooling) and planning outdoor activities. IMD forecasts daily temperatures for various cities across India.
- Weather Event Forecasting: Time series analysis aids in predicting the likelihood and intensity of weather events like cyclones (e.g., along the Bay of Bengal or Arabian Sea coast), heatwaves, or heavy rainfall. Historical track data of cyclones is analyzed to predict future paths and impact.
- Climate Change Analysis: Long-term time series data of temperature, precipitation, and greenhouse gas concentrations are analyzed to understand climate change trends and their potential impacts on regions like the Indian subcontinent.
Question 11. Explain the concept of forecasting using time series analysis. Describe how the trend component, once isolated, can be used for making predictions about future values. What are the inherent limitations of such forecasts?
Answer:
Forecasting using Time Series Analysis
Forecasting using time series analysis involves using historical data to predict future values of a variable. The core idea is that patterns observed in the past (trends, seasonality, cycles) are likely to continue into the future. Time series forecasting methods aim to identify and model these patterns to extrapolate them forward.
A typical time series can be decomposed into several components:
- Trend (T): The long-term upward or downward movement in the data.
- Seasonality (S): Regular, predictable patterns that repeat over a fixed period (e.g., daily, weekly, monthly, quarterly).
- Cyclical (C): Fluctuations that are not of a fixed period, often related to business cycles or other longer-term influences.
- Irregular/Random (I): Unpredictable variations or noise in the data.
These components can be combined additively ($Y_t = T_t + S_t + C_t + I_t$) or multiplicatively ($Y_t = T_t \times S_t \times C_t \times I_t$), where $Y_t$ is the value of the time series at time $t$.
Using the Isolated Trend Component for Prediction
Once the trend component ($T_t$) is isolated from the time series, it represents the underlying long-term direction of the data. This isolated trend can be used for prediction in the following ways:
- Extrapolation of the Trend Line: If the trend is estimated using a method like Least Squares to fit a line ($y = a + bx$), future values can be predicted by simply plugging in future values of $x$ (time) into this equation. For example, if the trend is upward and linear, it's assumed to continue rising at a constant rate.
- Forecasting the Smoothed Series: If methods like moving averages are used, the smoothed values themselves represent the trend. These smoothed values can be extended into the future, assuming the pattern of smoothing continues.
- Re-introducing Other Components (Advanced): In more sophisticated forecasting, after predicting the trend, seasonal and cyclical components are often re-estimated and added (or multiplied) back to the predicted trend to create a more comprehensive forecast that accounts for these regular patterns. However, if the focus is solely on the trend's predictive power, it's the extrapolated trend line that is used.
For instance, if we fit a trend line to annual sales data, the line $y = 3.4466 + 0.1702x$ predicted a profit of $\textsf{₹}$ 5.66 crores for 2027 by extrapolating the historical linear growth.
Inherent Limitations of Trend-Based Forecasts
Forecasts based solely on isolating and extrapolating the trend component have several inherent limitations:
- Assumption of Trend Continuation: The most significant limitation is the assumption that the historical trend will continue unchanged into the future. Economic, business, or environmental conditions can change, causing the trend to shift, accelerate, decelerate, or even reverse.
- Ignoring Other Components: Forecasts that rely only on the trend component ignore the impact of seasonality, cyclical patterns, and random fluctuations. This can lead to inaccurate predictions, especially for data with strong seasonal or cyclical influences. For example, predicting annual sales based only on trend would miss the impact of quarterly sales peaks or troughs.
- Sensitivity to Outliers and Volatility: If the trend estimation method is sensitive to outliers (like Least Squares), a single extreme data point in the historical data can significantly distort the trend line, leading to flawed future predictions.
- Lagging Nature of Trend Estimation: Methods that estimate trends (like moving averages) can lag behind actual turning points in the data, meaning the estimated trend might not reflect the most recent changes.
- Non-Linear Trends: If the underlying trend is not linear but is approximated by a linear trend line, the forecast will be inaccurate, especially for predictions far into the future.
- Lack of Causal Factors: Time series forecasting based on historical patterns does not inherently incorporate external causal factors (e.g., new government policies, competitor actions, technological disruptions) that could significantly impact future values.
- Unpredictability of Random Component: The random (irregular) component is, by definition, unpredictable. While methods aim to minimize its impact on the trend, its presence means that no forecast will ever be perfectly accurate.
Therefore, while trend analysis provides a baseline for understanding long-term direction, it's often combined with other methods and expert judgment for more robust forecasting.
Question 12. Calculate the 3-year and 5-year moving averages for the following sales data (in thousand units). Plot the original data and both moving average series on the same graph and comment on the smoothing effect of different periods.
Year | Sales |
2015 | 10 |
2016 | 12 |
2017 | 11 |
2018 | 15 |
2019 | 14 |
2020 | 18 |
2021 | 17 |
2022 | 20 |
2023 | 19 |
2024 | 22 |
Answer:
Calculation of 3-Year and 5-Year Moving Averages
We will calculate the moving averages for the given sales data. For odd-numbered moving averages (like 3-year and 5-year), the average is centered on the middle year of the period.
3-Year Moving Average (MA) Calculation:
This involves summing sales for three consecutive years and dividing by 3. The average is centered on the middle year.
5-Year Moving Average (MA) Calculation:
This involves summing sales for five consecutive years and dividing by 5. The average is centered on the middle year.
Year | Sales | 3-Year Moving Total | 3-Year Moving Average (Trend) | 5-Year Moving Total | 5-Year Moving Average (Trend) |
2015 | 10 | ||||
2016 | 12 | $10 + 12 + 11 = 33$ | $33 / 3 = 11.0$ (for 2016) | ||
2017 | 11 | $12 + 11 + 15 = 38$ | $38 / 3 = 12.7$ (for 2017) | $10+12+11+15+14 = 62$ | $62 / 5 = 12.4$ (for 2017) |
2018 | 15 | $11 + 15 + 14 = 40$ | $40 / 3 = 13.3$ (for 2018) | $12+11+15+14+18 = 70$ | $70 / 5 = 14.0$ (for 2018) |
2019 | 14 | $15 + 14 + 18 = 47$ | $47 / 3 = 15.7$ (for 2019) | $11+15+14+18+17 = 75$ | $75 / 5 = 15.0$ (for 2019) |
2020 | 18 | $14 + 18 + 17 = 49$ | $49 / 3 = 16.3$ (for 2020) | $15+14+18+17+20 = 84$ | $84 / 5 = 16.8$ (for 2020) |
2021 | 17 | $18 + 17 + 20 = 55$ | $55 / 3 = 18.3$ (for 2021) | $14+18+17+20+19 = 88$ | $88 / 5 = 17.6$ (for 2021) |
2022 | 20 | $17 + 20 + 19 = 56$ | $56 / 3 = 18.7$ (for 2022) | $18+17+20+19+22 = 96$ | $96 / 5 = 19.2$ (for 2022) |
2023 | 19 | $20 + 19 + 22 = 61$ | $61 / 3 = 20.3$ (for 2023) | $17+20+19+22 = 78$ (Incomplete 5-year sum) | |
2024 | 22 |
Plotting the Data and Moving Averages
To plot the data, you would create a graph with 'Year' on the x-axis and 'Sales' on the y-axis. Then, plot the original sales data points. You would also plot the calculated 3-year moving averages (centered on the middle year) and the 5-year moving averages (centered on the middle year) on the same graph.
*(Please note: A visual plot cannot be generated directly in this text-based format. However, the data points for plotting are provided in the table above. You would typically use spreadsheet software or charting tools to create this visual representation.)*
Key points for plotting:
- Original Data: Plot points for each year (e.g., (2015, 10), (2016, 12), ... (2024, 22)).
- 3-Year MA: Plot points like (2016, 11.0), (2017, 12.7), (2018, 13.3), ..., (2023, 20.3).
- 5-Year MA: Plot points like (2017, 12.4), (2018, 14.0), (2019, 15.0), ..., (2022, 19.2).
Comment on the Smoothing Effect of Different Periods
By visually inspecting the plotted lines (or comparing the calculated values), we can observe the smoothing effect:
- Original Data: The original sales data shows more fluctuations. For example, there's a dip from 11 in 2017 to 15 in 2018, then a dip to 14 in 2019, followed by a rise to 18 in 2020. These upswings and downswings represent short-term variations.
- 3-Year Moving Average: The 3-year moving average is smoother than the original data. It still shows the general upward trend but dampens some of the more immediate fluctuations. For instance, the sharp rise from 11 to 15 in the original data is smoothed out in the 3-year MA values for those years.
- 5-Year Moving Average: The 5-year moving average is significantly smoother than both the original data and the 3-year moving average. It captures the long-term upward trend more clearly by averaging over a longer period, effectively filtering out more of the short-term ups and downs. You can see that the values for the 5-year MA are less volatile and follow a more consistent upward path.
Conclusion on Smoothing: A longer moving average period (like 5 years) provides a greater degree of smoothing compared to a shorter period (like 3 years). This is because it averages over more data points, thus diluting the impact of any single period's extreme value. The trade-off is that a longer period results in a trend line that lags more behind the actual data and might obscure short-term turning points.
Question 13. Fit a linear trend line using the Method of Least Squares for the following population data (in lakhs):
Year | Population |
2010 | 180 |
2012 | 190 |
2014 | 205 |
2016 | 215 |
2018 | 230 |
2020 | 245 |
Answer:
Fitting a Linear Trend Line using the Method of Least Squares with Coded Time Values
We are fitting a linear trend line of the form $y = a + bx$, where $y$ is the population and $x$ is the coded time. Since the data is given for every two years, we can use coded time values that reflect this interval. Let's set the middle year of the data (2016) as $x=0$. This is a common practice to simplify calculations when data points are equally spaced.
The years and their corresponding coded time values ($x$) are:
- 2010: $x = -3$ (since 2010 is 6 years before 2016, and each interval is 2 years)
- 2012: $x = -2$
- 2014: $x = -1$
- 2016: $x = 0$
- 2018: $x = 1$
- 2020: $x = 2$
Now, we set up the table with the given population data and the coded time values:
Year | Population (y) | Coded Time (x) | $x^2$ | xy |
2010 | 180 | -3 | 9 | -540 |
2012 | 190 | -2 | 4 | -380 |
2014 | 205 | -1 | 1 | -205 |
2016 | 215 | 0 | 0 | 0 |
2018 | 230 | 1 | 1 | 230 |
2020 | 245 | 2 | 4 | 490 |
Totals | 1265 | -3 | 19 | -405 |
Here, $n = 6$ (number of data points).
Using the formulas for least squares:
$b = \frac{n \sum(xy) - \sum x \sum y}{n \sum(x^2) - (\sum x)^2}$
$b = \frac{6 \times (-405) - (-3) \times 1265}{6 \times 19 - (-3)^2}$
$b = \frac{-2430 - (-3795)}{114 - 9}$
$b = \frac{-2430 + 3795}{105}$
$b = \frac{1365}{105}$
$b = 13$
$a = \frac{\sum y - b \sum x}{n}$
$a = \frac{1265 - (13 \times -3)}{6}$
$a = \frac{1265 - (-39)}{6}$
$a = \frac{1265 + 39}{6}$
$a = \frac{1304}{6}$
$a \approx 217.33$
The linear trend line is approximately:
$y = 217.33 + 13x$
Where $x$ is the coded time value, with $x=0$ corresponding to the year 2016.
Estimating Population for the Year 2028
The year 2028 is 12 years after 2016. Since each coded time unit ($x$) represents 2 years, the coded time value for 2028 will be:
$x = \frac{2028 - 2016}{2} = \frac{12}{2} = 6$
Now, substitute $x = 6$ into the trend line equation:
$y_{2028} = 217.33 + 13 \times 6$
$y_{2028} = 217.33 + 78$
$y_{2028} = 295.33$
The estimated population for the year 2028 is approximately 295.33 lakhs.
Question 14. Explain the process of isolating the trend component from a time series using the Method of Moving Averages. Discuss how the choice of the moving average period affects the resulting trend line.
Answer:
Process of Isolating Trend using the Method of Moving Averages
The Method of Moving Averages is a simple yet effective technique to isolate the trend component from a time series. The fundamental idea is to smooth out the short-term fluctuations (seasonal, cyclical, and irregular components) by averaging consecutive data points. The average value over a period is considered a representation of the underlying trend during that period.
The process involves the following steps:
- Determine the Moving Average Period (n): The first step is to decide on the length of the period over which to calculate the average. This period should ideally be long enough to smooth out the dominant cyclical or seasonal patterns but not so long that it eliminates the trend itself.
- Calculate Moving Totals: For a chosen period 'n', calculate the sum of 'n' consecutive data points.
- Calculate Moving Averages: Divide each moving total by 'n' to get the moving average.
- Center the Moving Averages:
- Odd Period (e.g., 3-year, 5-year MA): When 'n' is odd, the moving average naturally falls on the middle data point of the period. For example, a 3-year MA for years 1, 2, and 3 is centered on year 2. This average directly represents the trend for that middle year.
- Even Period (e.g., 4-quarter, 12-month MA): When 'n' is even, the moving average falls between two data points. For example, a 4-quarter MA for Q1, Q2, Q3, Q4 of a year falls between Q2 and Q3. To center this average at a specific time point (usually the midpoint of the period), a further step is required: calculate a 2-period moving average of the 'n'-period moving averages. This is often referred to as a "2 x n moving average" (e.g., 2 x 4 moving average). For example, summing the moving average for (Q1-Q4) and (Q2-Q1 of next year) and dividing by 2 centers the value appropriately.
- The Centered Moving Averages represent the Trend: The calculated and centered moving averages at each time point (where calculable) are considered estimates of the trend component of the original time series.
Effect of the Choice of Moving Average Period on the Trend Line
The length of the moving average period ('n') significantly influences the resulting trend line:
- Shorter Period (e.g., 3-year MA):
- Effect: A shorter period results in a trend line that is more sensitive to short-term fluctuations. It will follow the original data more closely.
- Pros: It can capture turning points in the trend more quickly and is less likely to mask subtle changes.
- Cons: It provides less smoothing. If the data has significant random noise or short cycles, these may still appear in the trend line, making it less clear.
- Longer Period (e.g., 5-year, 10-year MA):
- Effect: A longer period results in a trend line that is much smoother and less responsive to short-term variations. It effectively filters out more noise and short cycles.
- Pros: It provides a clearer picture of the long-term underlying trend.
- Cons: It lags significantly behind the actual data. It might over-smooth, potentially obscuring important medium-term cyclical movements or failing to capture recent shifts in the trend. The first and last $(n-1)/2$ or $n/2$ points cannot be calculated.
- Cyclical Component Consideration: When choosing the period 'n', analysts often consider the dominant cycle length in the data. For instance, if there's a known business cycle of roughly 4 years, a moving average period longer than that (e.g., 5 years) might be chosen to smooth out the cycle and reveal a longer-term trend. If seasonality is a concern (e.g., quarterly data), a period related to the seasonality (e.g., 4-period for quarterly) is used, often with centering.
In summary, the choice of 'n' is a trade-off between responsiveness to trend changes and the degree of smoothing. A shorter period is more responsive but less smooth, while a longer period is smoother but lags more and might miss shorter-term trend shifts.
Question 15. The annual imports (in $\textsf{₹}$ thousand crores) of a country over 7 years are given:
Year | Imports |
2017 | 80 |
2018 | 85 |
2019 | 88 |
2020 | 82 |
2021 | 90 |
2022 | 93 |
2023 | 95 |
Answer:
Fitting a Linear Trend Line using the Method of Least Squares
We need to fit a linear trend line of the form $y = a + bx$, where $y$ represents the annual imports and $x$ represents the year. To simplify calculations, we'll use coded time values. Let the middle year, 2020, be coded as $x=0$. Since the data is annual, each subsequent year will have an increment of 1.
The years and their corresponding coded time values ($x$) are:
- 2017: $x = -3$
- 2018: $x = -2$
- 2019: $x = -1$
- 2020: $x = 0$
- 2021: $x = 1$
- 2022: $x = 2$
- 2023: $x = 3$
Now, we construct a table to calculate the necessary sums for the least squares formulas:
Year | Imports (y) | Coded Time (x) | $x^2$ | xy |
2017 | 80 | -3 | 9 | -240 |
2018 | 85 | -2 | 4 | -170 |
2019 | 88 | -1 | 1 | -88 |
2020 | 82 | 0 | 0 | 0 |
2021 | 90 | 1 | 1 | 90 |
2022 | 93 | 2 | 4 | 186 |
2023 | 95 | 3 | 9 | 285 |
Totals | 613 | 0 | 28 | 203 |
Here, $n = 7$ (the number of data points).
Using the formulas for least squares:
$b = \frac{n \sum(xy) - \sum x \sum y}{n \sum(x^2) - (\sum x)^2}$
$b = \frac{7 \times 203 - 0 \times 613}{7 \times 28 - (0)^2}$
$b = \frac{1421 - 0}{196 - 0}$
$b = \frac{1421}{196}$
$b \approx 7.25$
$a = \frac{\sum y - b \sum x}{n}$
$a = \frac{613 - (7.25 \times 0)}{7}$
$a = \frac{613}{7}$
$a \approx 87.57$
The linear trend line is approximately:
$y = 87.57 + 7.25x$
Where $y$ is the imports in $\textsf{₹}$ thousand crores and $x$ is the coded time value, with $x=0$ corresponding to the year 2020.
Interpretation of Slope and Intercept
- Intercept ($a \approx 87.57$): The intercept represents the estimated imports when the coded time value is zero. In this context, $x=0$ corresponds to the year 2020. Thus, the estimated imports for the year 2020, based on this trend line, are approximately $\textsf{₹}$ 87.57 thousand crores.
- Slope ($b \approx 7.25$): The slope represents the average annual change in imports. In this context, it means that, on average, the country's annual imports have been increasing by approximately $\textsf{₹}$ 7.25 thousand crores per year during the period 2017-2023.
Predicting Imports for the Year 2025
The year 2025 corresponds to a coded time value of $x = 2025 - 2020 = 5$.
Substitute $x = 5$ into the trend line equation:
$y_{2025} = 87.57 + 7.25 \times 5$
$y_{2025} = 87.57 + 36.25$
$y_{2025} = 123.82$
The predicted imports for the year 2025 are approximately $\textsf{₹}$ 123.82 thousand crores.
Question 16. Discuss the importance of identifying and understanding the Seasonal and Cyclical components of a time series for businesses and policymakers. Provide specific examples of how this understanding can influence decision-making.
Answer:
Importance of Seasonal and Cyclical Components in Time Series Analysis
Identifying and understanding the seasonal and cyclical components of a time series is crucial for businesses and policymakers because these components represent recurring patterns that significantly impact economic and operational activities. While the trend component shows the long-term direction, seasonality and cyclicality explain the more frequent ups and downs within that trend.
1. Importance for Businesses
For businesses, understanding these components is vital for effective planning, resource allocation, and strategy development.
- Sales and Demand Forecasting:
- Seasonal: Many products have predictable seasonal demand. For example, ice cream sales surge in summer, while warm clothing sales peak in winter. Understanding this seasonality allows businesses to stock appropriate inventory, plan production levels, and tailor marketing campaigns. For instance, a beverage company would ramp up production and marketing for cold drinks before and during the summer months in India.
- Cyclical: Business cycles affect demand for durables and luxury goods. During economic booms (upward phase of the cycle), demand for cars or high-end electronics increases. During recessions (downward phase), demand for such goods may fall. A car manufacturer needs to adjust production and sales targets based on the anticipated phase of the economic cycle.
- Inventory Management: Seasonal demand necessitates proactive inventory adjustments. A retailer must stock up on festive goods before major festivals like Diwali or Christmas. Understanding cyclical patterns helps manage inventory for longer-term economic trends.
- Pricing Strategies: Businesses can adjust prices based on seasonal demand. For example, airlines and hotels often charge higher prices during peak tourist seasons and offer discounts during off-peak periods.
- Resource Allocation: Understanding seasonal peaks in demand helps businesses plan staffing, operational capacity, and marketing budgets effectively. A retail store might hire temporary staff during holiday seasons.
- Financial Planning: Businesses can create more accurate financial forecasts by accounting for predictable seasonal cash flow variations, aiding in budgeting and investment decisions.
2. Importance for Policymakers
For policymakers, analyzing these components provides insights into economic health, helps in formulating effective fiscal and monetary policies, and guides resource management.
- Economic Stabilization Policies:
- Seasonal: Seasonal patterns in employment or consumer spending can distort short-term economic indicators. Policymakers use seasonal adjustment techniques to "remove" seasonality and get a clearer view of underlying economic trends, enabling better policy responses. For instance, adjusting unemployment figures to account for summer hiring or winter layoffs.
- Cyclical: Understanding the business cycle is paramount for macroeconomic policy. During a recession (downturn), policymakers might implement expansionary fiscal policies (e.g., increased government spending, tax cuts) or accommodative monetary policies (e.g., lower interest rates) to stimulate demand. Conversely, during an inflationary boom, contractionary policies might be used. The Reserve Bank of India (RBI) closely monitors cyclical indicators to adjust the repo rate and manage inflation.
- Resource Management:
- Seasonal: Sectors like agriculture and energy are heavily influenced by seasonality. Understanding seasonal rainfall patterns helps the government plan for water management, irrigation, and drought mitigation. Similarly, predictable seasonal increases in electricity demand (e.g., summer cooling, winter heating) require proactive energy generation planning.
- Cyclical: Policymakers need to anticipate cyclical downturns to implement counter-cyclical measures to prevent severe economic contraction and job losses.
- Infrastructure Planning: Understanding long-term cyclical trends in population growth, urbanization, and economic activity helps governments plan for infrastructure development (e.g., transportation, housing, utilities).
- Employment and Social Welfare: Policymakers need to understand seasonal employment patterns to design targeted employment generation programs or unemployment support, especially in sectors prone to seasonal job losses.
Examples of Decision-Making Influence:
- Retail Business: A clothing retailer seeing a seasonal peak in winter wear demand would increase orders for woolens and coats in the autumn and plan targeted advertising campaigns for the winter season.
- Automobile Manufacturer: An automobile company, anticipating a downturn in the economic cycle, might delay the launch of a new luxury car model or offer attractive financing schemes to boost sales of existing models.
- Government (Monetary Policy): If the RBI observes through time series analysis that inflation shows a strong seasonal pattern but the underlying cyclical trend is stable, it might adjust its policy based on the cyclical trend rather than just the seasonal peak. However, if cyclical inflation is rising, it might raise interest rates to curb demand.
- Government (Agricultural Policy): The Ministry of Agriculture, relying on IMD's rainfall time series analysis, might advise farmers on crop choices based on predicted monsoon strength or release water from reservoirs strategically during dry spells.
In essence, by dissecting a time series into its trend, seasonal, and cyclical components, businesses and policymakers gain a deeper, more nuanced understanding of the forces driving their data, enabling them to make more informed, proactive, and effective decisions.
Question 17. Calculate the 4-year centered moving averages for the following price data (in $\textsf{₹}$ per unit):
Year | Price |
2018 | 120 |
2019 | 125 |
2020 | 118 |
2021 | 130 |
2022 | 128 |
2023 | 135 |
2024 | 132 |
2025 | 140 |
Answer:
Calculation of 4-Year Centered Moving Averages
To calculate the 4-year centered moving average, we need to perform two steps:
- Calculate the 4-year moving totals.
- Calculate the 2-period moving average of these 4-year totals (this is the 2 x 4 moving average, which is centered).
Step 1: Calculate 4-Year Moving Totals
Sum the prices for consecutive 4-year periods.
Step 2: Calculate 2 x 4 Moving Averages (Centered)
Sum two consecutive 4-year moving totals and divide by 2. This average is then centered.
Year | Price (₹ per unit) | 4-Year Moving Total | 2 x 4 Moving Total (for centering) | 4-Year Centered Moving Average (Trend) |
2018 | 120 | |||
2019 | 125 | $120 + 125 + 118 + 130 = 493$ | ||
2020 | 118 | $125 + 118 + 130 + 128 = 501$ | $493 + 501 = 994$ | $994 / 2 = 497$ (Centered for mid-2020) |
2021 | 130 | $118 + 130 + 128 + 135 = 511$ | $501 + 511 = 1012$ | $1012 / 2 = 506$ (Centered for mid-2021) |
2022 | 128 | $130 + 128 + 135 + 132 = 525$ | $511 + 525 = 1036$ | $1036 / 2 = 518$ (Centered for mid-2022) |
2023 | 135 | $128 + 135 + 132 + 140 = 535$ | $525 + 535 = 1060$ | $1060 / 2 = 530$ (Centered for mid-2023) |
2024 | 132 | $135 + 132 + 140 = 407$ (Incomplete 4-year sum) | $535 + 407 = 942$ | $942 / 2 = 471$ (Centered for mid-2024) |
2025 | 140 |
Explanation of Centering:
A 4-year moving average for, say, 2018-2021 (total 493) is naturally centered between 2019 and 2020. The next 4-year moving average for 2019-2022 (total 501) is centered between 2020 and 2021. To get a centered average for a specific point in time, we average these two moving averages. The average of 493 and 501 is 497. This value of 497 is assigned to the midpoint of the entire 8-year period (2018-2025), which effectively corresponds to the middle of the first two periods being averaged, aligning it to the midpoint between Q2 and Q3 of 2020.
The centered moving averages are typically associated with the midpoint of the period they represent. For a 4-year moving average, the first calculable centered average corresponds to the midpoint between the second and third year of the first 4-year sum.
Note: The first and last three years do not have complete 4-year moving averages, so centered moving averages cannot be calculated for them.
Question 18. Fit a linear trend line using the Method of Least Squares for the following data on area under cultivation (in hectares):
Year | Area |
2017 | 500 |
2018 | 520 |
2019 | 510 |
2020 | 530 |
2021 | 525 |
2022 | 540 |
Answer:
Fitting a Linear Trend Line using the Method of Least Squares
We need to fit a linear trend line of the form $y = a + bx$, where $y$ represents the area under cultivation (in hectares) and $x$ represents the year. To simplify calculations, we'll use coded time values. Let the middle year, 2019.5 (midpoint between 2019 and 2020), be our reference point. Since the data is annual, we can code the years as follows:
Let $x = 0$ for the midpoint between 2019 and 2020. This means:
- 2017: $x = -2.5$
- 2018: $x = -1.5$
- 2019: $x = -0.5$
- 2020: $x = 0.5$
- 2021: $x = 1.5$
- 2022: $x = 2.5$
Alternatively, to avoid decimals, we can code the years sequentially starting from $x=1$ for 2017, or $x=0$ for 2017. Using $x=1$ for 2017:
- 2017: $x = 1$
- 2018: $x = 2$
- 2019: $x = 3$
- 2020: $x = 4$
- 2021: $x = 5$
- 2022: $x = 6$
Let's use the second coding method (starting $x=1$ for 2017) as it's more common and avoids decimals in this case.
Now, we construct a table to calculate the necessary sums for the least squares formulas:
Year | Area (y) | Time (x) | $x^2$ | xy |
2017 | 500 | 1 | 1 | 500 |
2018 | 520 | 2 | 4 | 1040 |
2019 | 510 | 3 | 9 | 1530 |
2020 | 530 | 4 | 16 | 2120 |
2021 | 525 | 5 | 25 | 2625 |
2022 | 540 | 6 | 36 | 3240 |
Totals | 3125 | 21 | 91 | 11055 |
Here, $n = 6$ (the number of data points).
Using the formulas for least squares:
$b = \frac{n \sum(xy) - \sum x \sum y}{n \sum(x^2) - (\sum x)^2}$
$b = \frac{6 \times 11055 - 21 \times 3125}{6 \times 91 - (21)^2}$
$b = \frac{66330 - 65625}{546 - 441}$
$b = \frac{705}{105}$
$b = 6.714$
$a = \frac{\sum y - b \sum x}{n}$
$a = \frac{3125 - (6.714 \times 21)}{6}$
$a = \frac{3125 - 141.00}{6}$
$a = \frac{2984}{6}$
$a \approx 497.33$
The linear trend line is approximately:
$y = 497.33 + 6.714x$
Where $y$ is the area in hectares and $x$ is the coded time value, with $x=1$ corresponding to the year 2017.
Estimating Area under Cultivation for the Year 2028
The year 2028 corresponds to a coded time value of $x = 2028 - 2017 + 1 = 11 + 1 = 12$.
Substitute $x = 12$ into the trend line equation:
$y_{2028} = 497.33 + 6.714 \times 12$
$y_{2028} = 497.33 + 80.568$
$y_{2028} = 577.898$
The estimated area under cultivation for the year 2028 is approximately 577.90 hectares.
Question 19. Explain the concept of univariate time series analysis. Discuss how identifying and modeling the trend component helps in understanding the long-term behaviour of the series and in making future projections.
Answer:
Concept of Univariate Time Series Analysis
Univariate time series analysis is a statistical method that deals with time-ordered sequences of observations for a single variable (hence, "univariate"). The goal is to understand the past behavior of this single variable, identify patterns within its history, and use this understanding to make forecasts or predictions about its future values. In essence, it assumes that the future behavior of the variable can be reasonably inferred from its past behavior.
Key characteristics of univariate time series analysis include:
- Single Variable: It focuses exclusively on the historical data of one variable (e.g., a country's GDP, a company's stock price, a city's temperature).
- Time Dependency: The observations are ordered chronologically, and the order matters. The value at any given time point is often dependent on previous values.
- Pattern Identification: Analysis involves identifying components like trend, seasonality, cyclicality, and randomness.
- Forecasting: The primary application is to predict future values of the variable based on the identified patterns.
Common techniques include Moving Averages, Exponential Smoothing, ARIMA (AutoRegressive Integrated Moving Average) models, and Decomposition methods.
Role of the Trend Component in Understanding Long-Term Behaviour and Future Projections
The trend component is arguably the most fundamental aspect of a time series, representing the long-term direction or underlying movement of the data over an extended period, devoid of short-term fluctuations. Identifying and modeling it is crucial for several reasons:
- Understanding Long-Term Behaviour:
- Directional Insight: It reveals whether the variable is generally increasing, decreasing, or remaining stable over the long haul. For example, analyzing the trend in a country's GDP shows its long-term economic growth trajectory. A consistently upward trend indicates economic expansion, while a downward trend might signal recessionary pressures.
- Stability Assessment: A stable trend (close to flat) suggests little long-term growth or decline, while a steep trend indicates rapid change. For instance, understanding the trend in a company's sales can indicate its market position and growth potential over the years.
- Base for Further Analysis: Isolating the trend allows for the separate analysis of other components like seasonality and cyclicality. By removing the trend, patterns that might otherwise be masked become more apparent.
- Making Future Projections:
- Extrapolation: Once a trend is identified and modeled (e.g., as a linear line $y = a + bx$ or a curve), it can be extrapolated into the future. This provides a baseline projection of where the variable is likely headed in the long term, assuming the identified trend continues.
- Strategic Planning: Businesses and policymakers use trend projections for long-term strategic planning. For example, a utility company projects the long-term trend in electricity demand to plan future power generation capacity. A government might use demographic trends (population growth) to plan infrastructure and social services.
- Setting Targets: Trend projections can help in setting realistic long-term targets. For instance, a company might set a sales growth target that aligns with its historical trend.
- Baseline for Forecasting: Even when more complex models are used that account for seasonality and cycles, the trend component often forms the foundational layer of the forecast.
For example, if a time series analysis shows a long-term upward trend in a country's population, this projection can inform government policies related to education, healthcare, and housing for decades to come. Similarly, if a company's profit trend is declining, it signals a need for strategic intervention to reverse this long-term behavior.
Question 20. Calculate the 3-year and 4-year centered moving averages for the following data on the number of vehicles sold (in hundreds):
Year | Sales |
2016 | 25 |
2017 | 28 |
2018 | 26 |
2019 | 30 |
2020 | 29 |
2021 | 32 |
2022 | 31 |
2023 | 35 |
Answer:
Calculation of 3-Year and 4-Year Centered Moving Averages
We will calculate both the 3-year and 4-year centered moving averages for the given sales data.
3-Year Moving Average (MA) Calculation:
Sum sales for three consecutive years and divide by 3. The average is centered on the middle year.
4-Year Centered Moving Average Calculation:
This requires a two-step process: first, calculate 4-year moving totals, and then calculate the 2-period moving average of these totals (2 x 4 moving average) to center the value.
Year | Sales (Hundreds) | 3-Year Moving Total | 3-Year Moving Average (Trend) | 4-Year Moving Total | 2 x 4 Moving Total (for centering) | 4-Year Centered Moving Average (Trend) |
2016 | 25 | |||||
2017 | 28 | $25 + 28 + 26 = 79$ | $79 / 3 = 26.33$ (for 2017) | |||
2018 | 26 | $28 + 26 + 30 = 84$ | $84 / 3 = 28.00$ (for 2018) | $25+28+26+30 = 109$ | ||
2019 | 30 | $26 + 30 + 29 = 85$ | $85 / 3 = 28.33$ (for 2019) | $28+26+30+29 = 113$ | $109 + 113 = 222$ | $222 / 2 = 111.00$ (Centered for mid-2018.5) |
2020 | 29 | $30 + 29 + 32 = 91$ | $91 / 3 = 30.33$ (for 2020) | $26+30+29+32 = 117$ | $113 + 117 = 230$ | $230 / 2 = 115.00$ (Centered for mid-2019.5) |
2021 | 32 | $29 + 32 + 31 = 92$ | $92 / 3 = 30.67$ (for 2021) | $30+29+32+31 = 122$ | $117 + 122 = 239$ | $239 / 2 = 119.50$ (Centered for mid-2020.5) |
2022 | 31 | $32 + 31 + 35 = 98$ | $98 / 3 = 32.67$ (for 2022) | $29+32+31+35 = 127$ | $122 + 127 = 249$ | $249 / 2 = 124.50$ (Centered for mid-2021.5) |
2023 | 35 | $31 + 35 = 66$ (Incomplete 3-year sum) | $32+31+35 = 98$ (Incomplete 4-year sum) | $127 + 98 = 225$ | $225 / 2 = 112.50$ (Centered for mid-2022.5) |
Comparison of Resulting Trend Values
Let's compare the trend values obtained from the 3-year and 4-year centered moving averages:
- 3-Year Moving Average Trend: The trend values are: 26.33 (2017), 28.00 (2018), 28.33 (2019), 30.33 (2020), 30.67 (2021), 32.67 (2022), 34.67 (2023). This series shows a generally increasing trend, mirroring the original data but smoothing out some of the year-to-year fluctuations.
- 4-Year Centered Moving Average Trend: The trend values are approximately: 111.00 (mid-2018.5), 115.00 (mid-2019.5), 119.50 (mid-2020.5), 124.50 (mid-2021.5), 112.50 (mid-2022.5). Note: The dates are mid-year because of the centering. These values are quite different from the 3-year MA because the 4-year MA captures longer-term movements and is influenced by a wider range of data points, thus smoothing more aggressively.
Key Differences and Observations:
- Smoothing Effect: The 4-year centered moving average is expected to be smoother than the 3-year moving average because it averages over a longer period. This means it will filter out more short-term variations and highlight the longer-term trend more clearly.
- Lag: The 4-year MA will generally lag more behind the original data than the 3-year MA.
- Sensitivity to Fluctuations: The 3-year MA will be more sensitive to individual year fluctuations compared to the 4-year MA, which is less affected by single outlier years.
- Applicability: The 3-year MA is centered on specific years (2017, 2018, etc.), making it easier to directly compare with the original annual data. The 4-year centered MA is centered between years, making direct comparison less straightforward, but it's better for isolating longer-term trends.
In summary, while both methods aim to smooth the data to reveal the trend, the 3-year MA provides a trend estimate that is more responsive to year-to-year changes, whereas the 4-year centered MA offers a smoother depiction of the longer-term underlying movement.
Question 21. Fit a linear trend line using the Method of Least Squares for the following profit data (in $\textsf{₹}$ crores):
Year | Profit |
2019 | 5.2 |
2020 | 5.5 |
2021 | 5.1 |
2022 | 5.7 |
2023 | 5.6 |
2024 | 5.9 |
Answer:
Fitting a Linear Trend Line using the Method of Least Squares
We will fit a linear trend line of the form $y = a + bx$, where $y$ represents the profit (in $\textsf{₹}$ crores) and $x$ represents the year. To simplify calculations, we'll use coded time values. Let's assign $x=1$ to the first year, 2019.
The years and their corresponding coded time values ($x$) are:
- 2019: $x = 1$
- 2020: $x = 2$
- 2021: $x = 3$
- 2022: $x = 4$
- 2023: $x = 5$
- 2024: $x = 6$
Now, we construct a table to calculate the necessary sums for the least squares formulas:
Year | Profit (y) | Time (x) | $x^2$ | xy |
2019 | 5.2 | 1 | 1 | 5.2 |
2020 | 5.5 | 2 | 4 | 11.0 |
2021 | 5.1 | 3 | 9 | 15.3 |
2022 | 5.7 | 4 | 16 | 22.8 |
2023 | 5.6 | 5 | 25 | 28.0 |
2024 | 5.9 | 6 | 36 | 35.4 |
Totals | 33.0 | 21 | 91 | 117.7 |
Here, $n = 6$ (the number of data points).
Using the formulas for least squares:
$b = \frac{n \sum(xy) - \sum x \sum y}{n \sum(x^2) - (\sum x)^2}$
$b = \frac{6 \times 117.7 - 21 \times 33.0}{6 \times 91 - (21)^2}$
$b = \frac{706.2 - 693.0}{546 - 441}$
$b = \frac{13.2}{105}$
$b \approx 0.1257$
$a = \frac{\sum y - b \sum x}{n}$
$a = \frac{33.0 - (0.1257 \times 21)}{6}$
$a = \frac{33.0 - 2.6397}{6}$
$a = \frac{30.3603}{6}$
$a \approx 5.060$
The linear trend line is approximately:
$y = 5.060 + 0.1257x$
Where $y$ is the profit in $\textsf{₹}$ crores and $x$ is the coded time value, with $x=1$ corresponding to the year 2019.
Predicting Profit for the Year 2029
The year 2029 corresponds to a coded time value of $x = 2029 - 2019 + 1 = 10 + 1 = 11$.
Substitute $x = 11$ into the trend line equation:
$y_{2029} = 5.060 + 0.1257 \times 11$
$y_{2029} = 5.060 + 1.3827$
$y_{2029} = 6.4427$
The predicted profit for the year 2029 is approximately $\textsf{₹}$ 6.44 crores.
Plotting the Original Data and the Trend Line
To plot the original data and the trend line, we would typically use a scatter plot:
- Original Data: Plot the actual profit values against their corresponding years (or coded time values). Points would be (2019, 5.2), (2020, 5.5), (2021, 5.1), (2022, 5.7), (2023, 5.6), (2024, 5.9).
- Trend Line: Calculate the trend values for each year using the fitted equation $y = 5.060 + 0.1257x$ and plot these points. For example:
- 2019 (x=1): $y = 5.060 + 0.1257(1) = 5.1857$
- 2020 (x=2): $y = 5.060 + 0.1257(2) = 5.3114$
- 2021 (x=3): $y = 5.060 + 0.1257(3) = 5.4371$
- 2022 (x=4): $y = 5.060 + 0.1257(4) = 5.5628$
- 2023 (x=5): $y = 5.060 + 0.1257(5) = 5.6885$
- 2024 (x=6): $y = 5.060 + 0.1257(6) = 5.8142$
The trend line will visually represent the average annual growth in profit, smoothing out the year-to-year variations observed in the original data.
Question 22. Discuss the various methods of measuring trend in time series analysis. Explain the steps involved in each method and their underlying principles. (Focus on Moving Averages and Least Squares).
Answer:
Methods of Measuring Trend in Time Series Analysis
Measuring trend is a fundamental step in time series analysis, aiming to capture the long-term, underlying direction of the data. Various methods exist, but we will focus on two commonly used techniques: the Method of Moving Averages and the Method of Least Squares.
1. Method of Moving Averages
Underlying Principle: This method works on the principle of smoothing out short-term fluctuations (seasonal, cyclical, and irregular components) by calculating averages over consecutive periods. The resulting average is considered a representation of the trend at the midpoint of the averaging period.
Steps Involved:
- Choose the Period (n): Decide on the length of the period (e.g., 3 years, 4 quarters, 12 months) over which to calculate the average. The choice depends on the nature of the data and the desired smoothing.
- Calculate Moving Totals: Sum up the values of the time series for 'n' consecutive periods.
- Calculate Moving Averages: Divide each moving total by 'n'. This gives the moving average.
- Center the Moving Averages:
- For Odd 'n' (e.g., 3, 5): The moving average naturally falls on the middle period. This value is assigned to the middle period's time point.
- For Even 'n' (e.g., 4, 12): The moving average falls between two periods. To center it, a second moving average is calculated (a 2-period MA of the 'n'-period MAs). This is often called a '2 x n' moving average. The result is assigned to the midpoint of the original 'n' periods.
- The resulting centered moving averages represent the trend component.
Example: For a 3-year moving average, we sum Y1+Y2+Y3 and divide by 3, assigning this to Year 2. Then Y2+Y3+Y4 divided by 3, assigned to Year 3, and so on.
2. Method of Least Squares
Underlying Principle: This is a more mathematically rigorous method that fits a straight line (or a curve) to the time series data by minimizing the sum of the squares of the vertical deviations (residuals) between the actual data points and the fitted line. The goal is to find the line that best represents the overall trend in the data.
Steps Involved for a Linear Trend ($y = a + bx$):
- Assign Time Values (x): Convert the years into numerical values. This can be done by sequential numbering (e.g., 1, 2, 3, ...) or by coding time values around a central point (e.g., using 0 for the middle year, -1, -2 for preceding years, and 1, 2 for succeeding years, especially if data is equally spaced).
- Set up the Equations: The least squares method aims to find the values of 'a' (intercept) and 'b' (slope) that minimize the sum of squared errors ($\sum(y_i - (a+bx_i))^2$). This leads to two normal equations:
$\sum y = na + b \sum x$
$\sum xy = a \sum x + b \sum x^2$
- Calculate Sums: Compute the following sums from the data: $n$, $\sum x$, $\sum y$, $\sum x^2$, and $\sum xy$.
- Solve for 'a' and 'b': Solve the system of two normal equations to find the values of 'a' and 'b'. The formulas are:
$b = \frac{n \sum(xy) - \sum x \sum y}{n \sum(x^2) - (\sum x)^2}$
$a = \frac{\sum y - b \sum x}{n}$
- The resulting equation $y = a + bx$ defines the linear trend line, where 'a' and 'b' are calculated from the historical data.
The values of $y$ obtained from this equation for each $x$ represent the trend values.
Comparison of Principles:
- Moving Averages: Focuses on local averages to smooth out data. It is intuitive but can be arbitrary in the choice of period and results in loss of data at the ends.
- Least Squares: A global fitting method that uses all data points to find the "best" line based on minimizing squared errors. It's statistically sound for linear trends but assumes linearity and can be sensitive to outliers.
Question 23. The annual imports data for a commodity (in tonnes) over 5 years are:
Year | Imports |
2018 | 1000 |
2019 | 1100 |
2020 | 1050 |
2021 | 1200 |
2022 | 1150 |
Answer:
Fitting a Linear Trend Line using the Method of Least Squares
We need to fit a linear trend line of the form $y = a + bx$, where $y$ represents the annual imports (in tonnes) and $x$ represents the year. To simplify calculations, we'll use coded time values. Let's assign $x=1$ to the first year, 2018.
The years and their corresponding coded time values ($x$) are:
- 2018: $x = 1$
- 2019: $x = 2$
- 2020: $x = 3$
- 2021: $x = 4$
- 2022: $x = 5$
Now, we construct a table to calculate the necessary sums for the least squares formulas:
Year | Imports (y) | Time (x) | $x^2$ | xy |
2018 | 1000 | 1 | 1 | 1000 |
2019 | 1100 | 2 | 4 | 2200 |
2020 | 1050 | 3 | 9 | 3150 |
2021 | 1200 | 4 | 16 | 4800 |
2022 | 1150 | 5 | 25 | 5750 |
Totals | 5500 | 15 | 55 | 16900 |
Here, $n = 5$ (the number of data points).
Using the formulas for least squares:
$b = \frac{n \sum(xy) - \sum x \sum y}{n \sum(x^2) - (\sum x)^2}$
$b = \frac{5 \times 16900 - 15 \times 5500}{5 \times 55 - (15)^2}$
$b = \frac{84500 - 82500}{275 - 225}$
$b = \frac{2000}{50}$
$b = 40$
$a = \frac{\sum y - b \sum x}{n}$
$a = \frac{5500 - (40 \times 15)}{5}$
$a = \frac{5500 - 600}{5}$
$a = \frac{4900}{5}$
$a = 980$
The linear trend line is:
$y = 980 + 40x$
Where $y$ is the imports in tonnes and $x$ is the coded time value, with $x=1$ corresponding to the year 2018.
Calculating Trend Values and Residuals
Now we calculate the trend values ($ \hat{y} = 980 + 40x $) and residuals ($e = y - \hat{y}$) for each year:
Year | Imports (y) | Time (x) | Trend Value ($\hat{y}$) | Residual (e) = y - $\hat{y}$ |
2018 | 1000 | 1 | $980 + 40(1) = 1020$ | $1000 - 1020 = -20$ |
2019 | 1100 | 2 | $980 + 40(2) = 1060$ | $1100 - 1060 = 40$ |
2020 | 1050 | 3 | $980 + 40(3) = 1100$ | $1050 - 1100 = -50$ |
2021 | 1200 | 4 | $980 + 40(4) = 1140$ | $1200 - 1140 = 60$ |
2022 | 1150 | 5 | $980 + 40(5) = 1180$ | $1150 - 1180 = -30$ |
Sum of Residuals | -20 + 40 - 50 + 60 - 30 = 0 |
Comment on the Goodness of Fit based on Residuals
The sum of the residuals is 0, which is a property of the least squares method. To comment on the goodness of fit, we look at the magnitude and pattern of the residuals:
- Magnitude: The residuals range from -50 to 60. These values represent the deviations of the actual imports from the predicted trend. The absolute values of these residuals (20, 40, 50, 60, 30) indicate the typical size of the errors. A common measure of fit is the Root Mean Square Error (RMSE), which would give a single value representing the average magnitude of the residuals.
- Pattern: Ideally, for a good linear fit, the residuals should appear to be randomly scattered around zero, with no discernible pattern. In this case, the residuals are: -20, 40, -50, 60, -30. There isn't a clear systematic pattern (like all positive then all negative, or a curve), suggesting that a linear trend is a reasonable approximation for this data. The residuals fluctuate above and below zero, indicating that the line captures the general direction but doesn't perfectly predict each year's value.
Conclusion on Goodness of Fit: Based on the residuals, the linear trend line provides a moderate fit. The residuals are relatively small compared to the magnitude of the data (imports ranging from 1000 to 1200), and they don't exhibit a strong systematic pattern, suggesting that the linear trend captures the underlying growth reasonably well.
Question 24. Explain how understanding the trend and seasonal components of sales data can help a manufacturing company plan its production schedule and inventory levels effectively in the Indian market, where festivals and weather patterns play a significant role.
Answer:
Impact of Trend and Seasonal Components on Production and Inventory Planning in India
For a manufacturing company operating in the diverse Indian market, understanding the trend and seasonal components of its sales data is critical for efficient production scheduling and optimal inventory management. The Indian context, with its distinct festivals, varied weather patterns, and regional consumption differences, makes this analysis even more vital.
1. Understanding the Trend Component:
The trend represents the long-term upward or downward movement in sales. It indicates the overall growth or decline of the product's market over years.
- Production Planning:
- Long-term Capacity: By analyzing the sales trend, a company can forecast its long-term demand. This helps in deciding whether to increase or decrease production capacity, invest in new machinery, or expand facilities. For instance, a company observing a consistent upward trend in demand for its air conditioners in India might plan to expand its manufacturing plants.
- Resource Allocation: A stable or growing trend suggests a need for consistent resource allocation (raw materials, labor, capital) to meet projected sales volumes. A declining trend might necessitate a review of product strategy or cost reduction measures.
- Inventory Management:
- Baseline Inventory: The trend helps establish a baseline inventory level needed to meet average demand. Companies can set safety stock levels based on the expected long-term sales trajectory.
- Strategic Stocking: If the trend is upward, a company might strategically build inventory in anticipation of future demand growth.
2. Understanding the Seasonal Component:
Seasonality refers to predictable, recurring patterns in sales that occur within a year, often driven by festivals, weather, or holidays.
- Production Scheduling:
- Peak Season Production: Manufacturing companies must ramp up production well in advance of anticipated seasonal peaks. In India, this is crucial for products related to major festivals like Diwali, Holi, or Eid, or for seasonal goods influenced by weather. For example, a fan and cooler manufacturer would boost production from early spring to meet summer demand, while an air conditioner company would do the same. Similarly, a packaged food company would increase production for sweets and snacks before Diwali.
- Off-Season Adjustments: During periods of lower seasonal demand, companies can adjust production schedules downwards to avoid overstocking and reduce carrying costs. They might also use this period for maintenance, R&D, or focusing on products with different seasonal demand patterns.
- Weather Sensitivity: Products like umbrellas, raincoats, or specific agricultural inputs have demand highly correlated with weather patterns (monsoon, winters). Production schedules must align with these weather-driven seasons.
- Inventory Levels:
- Anticipatory Stocking: Companies need to build inventory before peak demand seasons begin. For example, a toy manufacturer will increase production and inventory levels months before Diwali and Christmas to meet the surge in demand.
- Just-in-Time (JIT) vs. Anticipatory Stock: While JIT can be efficient, for highly seasonal products, it's often more practical to hold higher inventory levels in anticipation of demand, rather than relying on immediate production during peak times, which might not be feasible or cost-effective.
- Managing Excess Inventory: After a peak season, companies must manage any leftover inventory to avoid obsolescence or high holding costs. Understanding the pattern helps in planning clearance sales or alternative market strategies.
Integration of Trend and Seasonality for Effective Planning:
The real power comes from combining the understanding of both trend and seasonality.
- Example 1: Air Conditioner Manufacturer
- Trend: Observing a long-term upward trend in AC sales due to rising disposable incomes and hotter summers in India.
- Seasonality: Sales peak sharply in summer months (April-June) and are low in winter.
- Planning: The company plans for overall capacity expansion based on the upward trend. For production scheduling, it will ramp up manufacturing significantly from February/March to build inventory for the summer sales surge. Inventory levels will be highest in April/May and then gradually reduced after July. Marketing efforts will be concentrated during the pre-summer and summer periods.
- Example 2: Two-Wheeler Manufacturer
- Trend: A moderate upward trend in sales due to increasing urbanization and demand for personal mobility.
- Seasonality: Sales might see a slight uptick during festive seasons like Diwali when purchases are considered auspicious, and possibly during wedding seasons. Demand might also be influenced by monsoon (lower sales during heavy rains) and then pick up post-monsoon.
- Planning: Based on the trend, they plan for gradual capacity increases. For production, they might focus on building inventory before Diwali and potentially adjust production downwards during the peak monsoon months if data suggests a significant dip. Marketing campaigns will be timed around festivals and auspicious periods.
In conclusion, by analyzing sales data to understand both the long-term trend and the recurring seasonal patterns influenced by India's unique festivals and weather, manufacturing companies can make informed decisions about production volumes, resource allocation, inventory levels, and marketing strategies, leading to greater efficiency, reduced costs, and improved customer satisfaction.
Question 25. Calculate the 3-year and 5-year moving averages for the following data on the price of gold (in $\textsf{₹}$ per 10 grams):
Year | Price |
2015 | 25000 |
2016 | 26500 |
2017 | 28000 |
2018 | 30000 |
2019 | 32500 |
2020 | 35000 |
2021 | 38000 |
2022 | 41000 |
2023 | 45000 |
2024 | 50000 |
Answer:
Calculation of 3-Year and 5-Year Moving Averages
We will calculate the 3-year and 5-year moving averages for the given gold price data. For odd-numbered moving averages, the average is centered on the middle year of the period.
3-Year Moving Average (MA) Calculation:
Sum sales for three consecutive years and divide by 3. The average is centered on the middle year.
5-Year Moving Average (MA) Calculation:
Sum sales for five consecutive years and divide by 5. The average is centered on the middle year.
Year | Price (₹ per 10g) | 3-Year Moving Total | 3-Year Moving Average (Trend) | 5-Year Moving Total | 5-Year Moving Average (Trend) |
2015 | 25000 | ||||
2016 | 26500 | $25000 + 26500 + 28000 = 79500$ | $79500 / 3 = 26500$ (for 2016) | ||
2017 | 28000 | $26500 + 28000 + 30000 = 84500$ | $84500 / 3 = 28166.67$ (for 2017) | $25000+26500+28000+30000+32500 = 142000$ | $142000 / 5 = 28400$ (for 2017) |
2018 | 30000 | $28000 + 30000 + 32500 = 90500$ | $90500 / 3 = 30166.67$ (for 2018) | $26500+28000+30000+32500+35000 = 152000$ | $152000 / 5 = 30400$ (for 2018) |
2019 | 32500 | $30000 + 32500 + 35000 = 97500$ | $97500 / 3 = 32500$ (for 2019) | $28000+30000+32500+35000+38000 = 163500$ | $163500 / 5 = 32700$ (for 2019) |
2020 | 35000 | $32500 + 35000 + 38000 = 105500$ | $105500 / 3 = 35166.67$ (for 2020) | $30000+32500+35000+38000+41000 = 176500$ | $176500 / 5 = 35300$ (for 2020) |
2021 | 38000 | $35000 + 38000 + 41000 = 114000$ | $114000 / 3 = 38000$ (for 2021) | $32500+35000+38000+41000+45000 = 191500$ | $191500 / 5 = 38300$ (for 2021) |
2022 | 41000 | $38000 + 41000 + 45000 = 124000$ | $124000 / 3 = 41333.33$ (for 2022) | $35000+38000+41000+45000+50000 = 209000$ | $209000 / 5 = 41800$ (for 2022) |
2023 | 45000 | $41000 + 45000 + 50000 = 136000$ | $136000 / 3 = 45333.33$ (for 2023) | $38000+41000+45000+50000 = 174000$ (Incomplete 5-year sum) | |
2024 | 50000 |
Plotting the Data and Moving Averages
A graph would be created with 'Year' on the x-axis and 'Price' on the y-axis. The original gold price data points would be plotted. Then, the calculated 3-year moving average trend values and the 5-year moving average trend values would be plotted against their respective centered years.
*(Visual plotting is not possible in this format. The table provides the data points for the graph.)*
Key plotting points:
- Original Data: (2015, 25000), (2016, 26500), ..., (2024, 50000)
- 3-Year MA Trend: (2016, 26500), (2017, 28166.67), ..., (2023, 45333.33)
- 5-Year MA Trend: (2017, 28400), (2018, 30400), ..., (2022, 41800)
Comment on Smoother Representation of the Trend
By observing the plotted lines or comparing the trend values:
- Original Data: The gold price data shows a strong and consistent upward trend, but there are year-to-year fluctuations. For instance, the price increase from 2015 to 2016 is 1500, then 1500 to 2017, then 2000 to 2018, then 2500 to 2019, showing accelerating growth but with some variations in the rate of increase.
- 3-Year Moving Average: This series is smoother than the original data. It captures the general upward trend but dampens the year-to-year variability. The trend values closely follow the overall upward movement.
- 5-Year Moving Average: This series is considerably smoother than both the original data and the 3-year moving average. It filters out more of the year-to-year price changes, providing a clearer view of the long-term trend in gold prices. The differences between consecutive 5-year MA values are smaller than the differences between consecutive 3-year MA values, indicating greater smoothing. For example, the 5-year MA shows a steadier increase from 28400 in 2017 to 41800 in 2022.
Conclusion: The 5-year moving average provides a smoother representation of the trend in this gold price data. This is because averaging over a longer period effectively filters out more of the short-term fluctuations, highlighting the underlying long-term direction more effectively than the 3-year moving average.
Question 26. Fit a linear trend line $Y_t = a + bt$ using the Method of Least Squares for the following data on the number of tourists visiting a state (in lakhs):
Year | Tourists |
2016 | 15 |
2017 | 18 |
2018 | 20 |
2019 | 22 |
2020 | 16 |
2021 | 19 |
2022 | 23 |
Answer:
Fitting a Linear Trend Line using the Method of Least Squares
We need to fit a linear trend line of the form $Y_t = a + bt$, where $Y_t$ represents the number of tourists (in lakhs) and $t$ represents the year. To simplify calculations, we'll use coded time values. Let's assign $t=1$ to the first year, 2016.
The years and their corresponding coded time values ($t$) are:
- 2016: $t = 1$
- 2017: $t = 2$
- 2018: $t = 3$
- 2019: $t = 4$
- 2020: $t = 5$
- 2021: $t = 6$
- 2022: $t = 7$
Now, we construct a table to calculate the necessary sums for the least squares formulas:
Year | Tourists (Y) | Time (t) | $t^2$ | tY |
2016 | 15 | 1 | 1 | 15 |
2017 | 18 | 2 | 4 | 36 |
2018 | 20 | 3 | 9 | 60 |
2019 | 22 | 4 | 16 | 88 |
2020 | 16 | 5 | 25 | 80 |
2021 | 19 | 6 | 36 | 114 |
2022 | 23 | 7 | 49 | 161 |
Totals | 133 | 28 | 140 | 554 |
Here, $n = 7$ (the number of data points).
Using the formulas for least squares:
$b = \frac{n \sum(tY) - \sum t \sum Y}{n \sum(t^2) - (\sum t)^2}$
$b = \frac{7 \times 554 - 28 \times 133}{7 \times 140 - (28)^2}$
$b = \frac{3878 - 3724}{980 - 784}$
$b = \frac{154}{196}$
$b \approx 0.786$
$a = \frac{\sum Y - b \sum t}{n}$
$a = \frac{133 - (0.786 \times 28)}{7}$
$a = \frac{133 - 22.008}{7}$
$a = \frac{110.992}{7}$
$a \approx 15.856$
The linear trend line is approximately:
$Y_t = 15.856 + 0.786t$
Where $Y_t$ is the number of tourists in lakhs and $t$ is the coded time value, with $t=1$ corresponding to the year 2016.
Calculating Trend Value for 2020 and Deviation
The year 2020 corresponds to a coded time value of $t=5$.
Calculate the trend value for 2020 using the fitted line:
Trend Value for 2020 ($\hat{Y}_{2020}$) = $15.856 + 0.786 \times 5$
$\hat{Y}_{2020} = 15.856 + 3.93$
$\hat{Y}_{2020} = 19.786$
The actual number of tourists in 2020 was 16 lakhs.
The deviation (residual) for 2020 is calculated as:
Deviation = Actual Value - Trend Value
Deviation for 2020 = $16 - 19.786$
Deviation for 2020 = $-3.786$
The magnitude of the deviation is the absolute value of this difference.
Magnitude of Deviation = $|-3.786| \approx 3.786$ lakhs.
Comment: The year 2020 shows a significant deviation because the actual tourist number (16 lakhs) is considerably lower than the trend value predicted by the linear model (19.786 lakhs). This suggests that some factor, like the COVID-19 pandemic, severely impacted tourism in 2020, causing a sharp drop below the otherwise increasing trend.
Question 27. Describe the process of fitting a linear trend using the Method of Least Squares when the number of time periods is even. Explain why coding of the time variable is necessary in this case. Illustrate with the normal equations and how they change with coding.
Answer:
Fitting a Linear Trend using Least Squares with Even Time Periods
When fitting a linear trend line $Y_t = a + bt$ using the Method of Least Squares, the goal is to find the values of 'a' (intercept) and 'b' (slope) that minimize the sum of the squared errors ($\sum (Y_t - (a + bt))^2$). The normal equations derived from this minimization process are:
$\sum Y = na + b \sum t$
$\sum tY = a \sum t + b \sum t^2$
Where $n$ is the number of observations, $Y$ is the dependent variable, and $t$ is the time variable.
The Challenge with Even Time Periods and the Necessity of Coding
When the number of time periods ($n$) is even, a critical issue arises with simple sequential coding of time (e.g., $t=1, 2, 3, \dots, n$). If we use such coding, the sum of time values ($\sum t$) and the sum of squared time values ($\sum t^2$) can lead to more complex calculations and, more importantly, the interpretation of the intercept 'a' becomes less straightforward. Specifically, the intercept 'a' would represent the trend value at $t=0$, which would correspond to a hypothetical time point before the first observation, not necessarily a meaningful midpoint.
Why Coding is Necessary:
Coding the time variable is essential for simplifying calculations and making the interpretation of the intercept 'a' more meaningful, especially when $n$ is even.
- Simplification of $\sum t$ and $\sum t^2$: By coding the time variable appropriately, we can make $\sum t = 0$. This significantly simplifies the normal equations, making them easier to solve.
- Meaningful Intercept: When $\sum t = 0$, the first normal equation becomes $\sum Y = na + b(0)$, which simplifies to $\sum Y = na$, or $a = \frac{\sum Y}{n} = \bar{Y}$ (the mean of Y). In this scenario, 'a' represents the trend value at the central point of the time series (which is the mean of the coded 't' values). This central point is a more meaningful reference for interpreting the trend.
- Handling of Even 'n': For an even number of periods, there isn't a single middle period. Coding around a central point (which might be a half-period) ensures that the trend line is centered correctly with respect to the entire time span of the data.
Process with Coding (Centering the Time Variable):
The most common and effective coding strategy when $n$ is even is to center the time variable around a point that is the mean of the time values. This is typically achieved by:
- If $n$ is even, let the sequence of time values be $1, 2, \dots, n$. The mean is $\frac{n+1}{2}$.
- To center, we can subtract this mean from each time value. However, to avoid decimals in time values, a common practice is to use a sequence of consecutive integers that sum to zero.
- For an even $n$, say $n=6$, the sequence is $1, 2, 3, 4, 5, 6$. The mean is $\frac{6+1}{2} = 3.5$.
- We can code the time values as: $-2.5, -1.5, -0.5, 0.5, 1.5, 2.5$. (Subtracting 3.5 from each time value). Sum of these $t$ values is 0.
- Alternatively, to avoid decimals, we can use a sequence of integers that are symmetric around zero, like for $n=6$: $-5, -3, -1, 1, 3, 5$. Here the common difference is 2, which represents the interval between observations. Sum of these $t$ values is 0. This is often preferred when observations are equally spaced.
Let's illustrate using the sequence $\{- (n-1), -(n-3), \dots, -1, 1, \dots, (n-3), (n-1)\}$ where the common difference is 2. This is valid for any $n \ge 2$. If $n$ is even, this sequence sums to 0.
How Normal Equations Change with Coding ($\sum t = 0$):
If we choose a coding scheme such that $\sum t = 0$, the normal equations simplify:
Original Eq 1: $\sum Y = na + b \sum t$
With $\sum t = 0$: $\sum Y = na + b(0) \implies \sum Y = na$
Simplified Eq 1: $a = \frac{\sum Y}{n} = \bar{Y}$
Original Eq 2: $\sum tY = a \sum t + b \sum t^2$
With $\sum t = 0$: $\sum tY = a(0) + b \sum t^2 \implies \sum tY = b \sum t^2$
Simplified Eq 2: $b = \frac{\sum tY}{\sum t^2}$
These simplified equations are much easier to solve, and the intercept 'a' directly gives the average value of Y at the central point of the time series (where $t=0$ in the coded system).
Question 28. The following table shows the annual production of steel (in million tonnes) in a country:
Year | Production |
2015 | 8 |
2016 | 9 |
2017 | 9.5 |
2018 | 10 |
2019 | 10.5 |
2020 | 11 |
2021 | 11.5 |
2022 | 12 |
Answer:
Part 1: Calculation of 4-Year Centered Moving Average for Trend
To calculate the 4-year centered moving average, we perform two steps:
- Calculate the 4-year moving totals.
- Calculate the 2-period moving average of these totals (2 x 4 moving average) to center the value.
Year | Production (Million Tonnes) | 4-Year Moving Total | 2 x 4 Moving Total (for centering) | 4-Year Centered Moving Average (Trend) |
2015 | 8 | |||
2016 | 9 | $8 + 9 + 9.5 + 10 = 36.5$ | ||
2017 | 9.5 | $9 + 9.5 + 10 + 10.5 = 39$ | $36.5 + 39 = 75.5$ | $75.5 / 2 = 37.75$ (Centered for mid-2017.5) |
2018 | 10 | $9.5 + 10 + 10.5 + 11 = 41$ | $39 + 41 = 80$ | $80 / 2 = 40.00$ (Centered for mid-2018.5) |
2019 | 10.5 | $10 + 10.5 + 11 + 11.5 = 43$ | $41 + 43 = 84$ | $84 / 2 = 42.00$ (Centered for mid-2019.5) |
2020 | 11 | $10.5 + 11 + 11.5 + 12 = 45$ | $43 + 45 = 88$ | $88 / 2 = 44.00$ (Centered for mid-2020.5) |
2021 | 11.5 | $11 + 11.5 + 12 = 34.5$ (Incomplete 4-year sum) | $45 + 34.5 = 79.5$ | $79.5 / 2 = 39.75$ (Centered for mid-2021.5) |
2022 | 12 |
Note: The first and last three years do not have complete 4-year moving averages for centering.
Part 2: Fitting a Linear Trend using the Method of Least Squares
We fit a linear trend line $Y_t = a + bt$. Since the number of periods is 8 (even), we need to code the time variable to simplify calculations and have a meaningful intercept. Let's use a coding where time is centered around a midpoint.
The years are 2015 to 2022. The midpoint is between 2018 and 2019.
Let's code time $t$ using the sequence of odd integers centered around 0:
- 2015: $t = -7$
- 2016: $t = -5$
- 2017: $t = -3$
- 2018: $t = -1$
- 2019: $t = 1$
- 2020: $t = 3$
- 2021: $t = 5$
- 2022: $t = 7$
This coding ensures $\sum t = 0$, simplifying the normal equations.
Now, construct the table for calculations:
Year | Production (Y) | Time (t) | $t^2$ | tY |
2015 | 8 | -7 | 49 | -56 |
2016 | 9 | -5 | 25 | -45 |
2017 | 9.5 | -3 | 9 | -28.5 |
2018 | 10 | -1 | 1 | -10 |
2019 | 10.5 | 1 | 1 | 10.5 |
2020 | 11 | 3 | 9 | 33 |
2021 | 11.5 | 5 | 25 | 57.5 |
2022 | 12 | 7 | 49 | 84 |
Totals | 81.5 | 0 | 168 | 145.5 |
Here, $n = 8$ (the number of data points).
Using the simplified normal equations since $\sum t = 0$:
$a = \frac{\sum Y}{n}$
$a = \frac{81.5}{8}$
$a = 10.1875$
$b = \frac{\sum tY}{\sum t^2}$
$b = \frac{145.5}{168}$
$b \approx 0.866$
The linear trend line equation is approximately:
$Y_t = 10.1875 + 0.866t$
Where $Y_t$ is the production in million tonnes and $t$ is the coded time value (with $t=0$ corresponding to the midpoint between 2018 and 2019).
Now, we calculate the trend values using this equation for the years where 4-year centered moving averages could be calculated:
Year | Coded Time (t) | Trend Value ($\hat{Y}$) = 10.1875 + 0.866t |
2017 | -3 | $10.1875 + 0.866(-3) = 10.1875 - 2.598 = 7.5895$ |
2018 | -1 | $10.1875 + 0.866(-1) = 10.1875 - 0.866 = 9.3215$ |
2019 | 1 | $10.1875 + 0.866(1) = 10.1875 + 0.866 = 11.0535$ |
2020 | 3 | $10.1875 + 0.866(3) = 10.1875 + 2.598 = 12.7855$ |
2021 | 5 | $10.1875 + 0.866(5) = 10.1875 + 4.33 = 14.5175$ |
2022 | 7 | $10.1875 + 0.866(7) = 10.1875 + 6.062 = 16.2495$ |
Part 3: Comparison of Trend Values
We compare the trend values from the 4-year centered moving average and the linear trend line (Least Squares) for the common years (2017, 2018, 2019, 2020, 2021, 2022).
Year | 4-Year Centered MA Trend | Least Squares Linear Trend | Difference (LS - MA) |
2017 | 37.75 | 7.59 | $-30.16$ |
2018 | 40.00 | 9.32 | $-30.68$ |
2019 | 42.00 | 11.05 | $-30.95$ |
2020 | 44.00 | 12.79 | $-31.21$ |
2021 | 39.75 | 14.52 | $-25.23$ |
2022 | N/A (Incomplete) | 16.25 |
Observations from Comparison:
- Difference in Scale: The most striking observation is the significant difference in the magnitude of the trend values themselves. The 4-year MA trend values are much lower than those from the linear trend line. This is because the 4-year centered moving average is a calculation based on the *original data values*, not a smoothed representation of the *entire* series' trend in a way that can be directly compared to a regression line. The values calculated for the 4-year MA in the table above are not representative of the production values; they are averages of production values that are themselves averages. A proper calculation of the 4-year moving average trend would be:
Year 4-Year Centered Moving Average (Production) 2017.5 (Mid-2017) $37.75 / 4 = 9.4375$ 2018.5 (Mid-2018) $40.00 / 4 = 10.0000$ 2019.5 (Mid-2019) $42.00 / 4 = 10.5000$ 2020.5 (Mid-2020) $44.00 / 4 = 11.0000$ 2021.5 (Mid-2021) $39.75 / 4 = 9.9375$ Year LS Trend Value 2017.5 $10.1875 + 0.866( -2.5) = 8.0225$ 2018.5 $10.1875 + 0.866( -1.5) = 8.8885$ 2019.5 $10.1875 + 0.866( -0.5) = 9.7475$ 2020.5 $10.1875 + 0.866( 0.5) = 10.6125$ 2021.5 $10.1875 + 0.866( 1.5) = 11.4775$ 2022.5 $10.1875 + 0.866( 2.5) = 12.3425$ Year (Midpoint) 4-Year Centered MA Trend LS Linear Trend Difference (LS - MA) 2017.5 9.4375 8.0225 $-1.4150$ 2018.5 10.0000 8.8885 $-1.1115$ 2019.5 10.5000 9.7475 $-0.7525$ 2020.5 11.0000 10.6125 $-0.3875$ 2021.5 9.9375 11.4775 $1.5400$ - Smoothing and Trend Capture: Both methods show an increasing trend. The linear trend line from least squares is a smooth representation of the overall movement across all data points. The moving average trend also indicates an upward movement, but the last value (2021.5) dips slightly, which is due to the relatively lower production in 2022 (12) compared to the preceding years in that moving average calculation, impacting the "average" more significantly with a longer span. This highlights how moving averages can sometimes be less responsive to genuine shifts or fluctuations compared to a regression line fitted to all points.
- Differences: The LS trend is calculated based on minimizing overall error, providing a single consistent slope. The moving average trend is more localized. The differences highlight that while both aim to find trend, they do so with different methodologies and sensitivities. The LS trend is generally preferred for its global optimization and clearer representation of a consistent long-term rate of change, especially when the trend is expected to be linear.